You are on page 1of 152

HL

Now with
interactive
e-book

PRACTICE
QUESTIONS
PEARSON
BACCALAUREATE

H IG H E R L E V E L

0DWKHPDWLFV 2012 edition

DEVELOPED SPECIFICALLY FOR THE


IB DIPLOMA

I B R A H I M WA Z I R • T I M G A R R Y
P E T E R A S H B O U R N E • PA U L B A R C L AY • P E T E R F LY N N • K E V I N F R E D E R I C K • M I K E WA K E F O R D
Chapter 2
Practice questions

1 f ( g ( x )) = f ( x 2 + 2x ) = x 2 + 2x − 3
a A radical function is not defined when the expression under the square root has a negative value, so:
x 2 + 2x − 3 < 0 ⇒ x 2 − x + 3x − 3 < 0 ⇒ x ( x − 1) + 3 ( x − 1) < 0 ⇒ ( x − 1) ( x + 3) < 0 ⇒ x ∈ ]−3, 1[
Therefore, a = −3, b = 1.
b

The range of the function is y  0.

Solution Paper 1 type

2 a g −1( 3) = a ⇒ g (a) = 3 ⇒ 2a − 7 = 3 ⇒ 2a = 3 + 7 ⇒ a = 5 ⇒ g −1( 3) = 5


b h ( g (6)) = h (2 × 6 − 7) = h (5) = 3 (2 − 5) = 3 × −3 = −9

Solution Paper 2 type

2 Firstly, we are going to input both functions in the Function menu.

a We have to solve the equation g(x) = 3, which can be solved in many different ways. Here we are going to use Solver.

b We are going to define the third function as a composition, y = g (h ( x )), and calculate its value for x = 6.

1
Chapter 2

Solution Paper 1 type

4−x 4− y
3 a g (x ) = ⇒x= ⇒ 3x = 4 − y ⇒ y = 4 − 3x ⇒ g−1 ( x ) = 4 − 3x
3 3
10 2 2
b f ( g−1 ( x )) = f ( 4 − 3x ) = 5 ( 4 − 3x ) − 2 = 20 − 15x − 2 = 18 − 15x ⇒ 18 − 15x = 8 ⇒ 10 = 15x ⇒ x = =
15 3 3

Solution Paper 2 type

3 This question can be solved using a CAS calculator, which, at present, is not permitted on the final exam by the IBO.
a–b

4 a g (h ( x )) = g (2x ) = 2x − 3

g( y ) = x ⇒ y − 3 = x ⇒ g−1 ( x ) = x + 3

 ⇒ g (14) + h (14) = 17 + 7 = 24
−1 −1
b x
h( y ) = x ⇒ 2 y = x ⇒ h ( x ) =
−1

2
5 a

b Since the function is transformed by applying a horizontal translation of one unit to the left and a vertical translation
 1
of half a unit down, the new coordinates of the maximum point are  −1, −  , whilst the new coordinates of the
 2
3
minimum point are  0, −  .
 2
1
6 There are four transformations defined by the graph y = − ( x + 5) + 3. The first is a reflection in the x-axis (the line
2

2
1. Then we have two translations: a horizontal translation of five units
y = 0). The second is a vertical stretch by scale factor __
2
to the left and a vertical translation of three units up. Therefore, the parameters are as follows:
1
a k= b p = −5 c q=3
2

2
7 Since we cannot use a calculator or apply any calculus knowledge, as this appears later in the textbook, we will simply
calculate the corresponding y-values for the given x-values from the domain. We also notice that this is an even function,
4 4
which means f (− x ) = = = f ( x ) , and geometrically its graph is symmetrical with respect to the
16 − (− x ) 16 − x 2
2

y-axis.
a Using a scientific calculator we obtain the following y-values:

x –3.5 –3 –2 –1 0 1 2 3 3. 5
y 2.07 1.51 1.15 1.03 1 1.03 1.15 1.51 2.07

From the table, we see that f(x) has a minimum value when the denominator is a maximum, i.e. when x = 0.
5 y

4
1
f(x) = √16 – x2
x

−4 −2 2 4

−1

b Since 16 − x > 0, the vertical asymptotes are x = −4 and x = 4.


2

c Looking at the table and the graph, we can see that the lowest possible y-value is 1 and the graph has two vertical
asymptotes. Therefore, when the values of x approach −4 or 4, the y-values tend to positive infinity.
R (f ) = { y ∈  : y  1}
8 a We are going to use a tabular form to show some points on the graph.

1 
– __ 1 
__
x –4 –3 –2 –1
2 2
1 2 3 4
1 
– __ 1 
– __ 1 
– __ 1 
__ 1 
__ 1 
__
y 4 3 2
–1 –2 2 1
2 3 4
3 y

1
2

g(x) = x
1

−4 −2 2 4

−1

−2

−3

b The horizontal translation of four units to the left means that instead of x we will have the expression (x + 4); whilst the
vertical translation of two units down means that we are going to subtract 2 at the end. So, the function h looks
1
like this: h ( x ) = − 2.
x+4
1 1 − 2x − 8 2x + 7
Note: This function can be written in a different form: h ( x ) = −2= =− .
x+4 x+4 x+4
This second form might be better for the calculations in part c.
2x + 7 7  7 
c i y=0⇒− = 0 ⇒ 2x + 7 = 0 ⇒ x = − ⇒  − , 0
x+4 2  2 
2×0+7 7  7
x=0⇒y=− = − ⇒  0, − 
0+4 4  4

3
Chapter 2

ii By looking at the first form, we see that the vertical asymptote has the equation x = −4, whilst the horizontal
asymptote has the equation y = −2. That result can also be found by looking at the asymptotes of the
1
original function, y = , which were the coordinate axes. By translating the original function four units to
x
the left, we were also translating the vertical asymptote. And again, by translating the function two units
down, we were translating the horizontal asymptote.
iii
3 y

−10 −8 −6 −4 −2 2

−1

−2

1 –2
h(x) =
−3

x+4 −4

−5

−6

9 a i f (8) = 8 + 3 = 11
ii f ( 46) = 46 + 3 = 49 = 7
iii f ( −3) = −3 + 3 = 0 = 0
b The function f is not defined for negative values under the square root, and therefore x + 3 < 0 ⇒ x < −3.
( ) ( )
2
c g (f ( x )) = g x+3 = x + 3 − 5 = x + 3 − 5 = x − 2 , x  −3
Notice here that the composition would not be represented by the whole straight line, but simply by a ray starting at
the point (23, 25).
a−8
10 a g −1 ( −2) = a ⇒ g (a) = −2 ⇒ = −2 ⇒ a − 8 = −4 ⇒ a = 4
2
An alternative method would be to find the inverse function first and then calculate its value at 22.
y−8
g ( y) = x ⇒ = x ⇒ y − 8 = 2x ⇒ g−1 ( x ) = 2x + 8
2
(This calculation is going to be used in the following part.)
g −1 ( −2) = 2 × ( −2) + 8 = 4
b g−1 (h ( x )) = g−1 ( x 2 − 1) = 2 × ( x 2 − 1) + 8 = 2x 2 − 2 + 8 = 2x 2 + 6
c g−1 (h ( x )) = 22 ⇒ 2 x 2 + 6 = 22 ⇒ 2 x 2 = 16 ⇒ x 2 = 8 ⇒ x = ± 8 = ±2 2
x +1
11 a f ( y ) = x ⇒ 3 y − 1 = x ⇒ 3 y = x + 1 ⇒ y =
3
 4  4 12
b f ( g ( x )) = f   = 3 ×   − 1 = −1
 x  x x
12 12 12
c f ( g ( y )) = x ⇒ = x + 1 ⇒ (f  g ( x )) =
−1
−1= x ⇒ , x ≠ −1
y y x +1
 4 4 x
d g ( g ( x )) = g   = = 4 × =x
 x 4 4
x
We notice that the function is self-inverse. That can be spotted from the graph, which is symmetrical with respect to
the line y = x .
12 a Even though the question suggests finding the parameters in a different order, we are going to use the form of the
function first and then the point.
ii Since the vertical line MN is a vertical asymptote, we can read its equation as x = 23. The function of the form
a
h (x ) = has a vertical asymptote when the denominator is equal to zero, so we can find the value of b: b = 23.
x−b

4
i To find parameter a, we will use point A(24, 28) which is on the graph.
a
−8 = ⇒ a = −8 × ( −1) = 8
−4 + 3
b y

10

A' (-4,8)

−20 −15 −10 −5 5 10 15 20

−5

A (-4,-8)
−10

The transformation is a reflection in the x-axis.


13 a

b To obtain the graph y = −f ( x ) − 1 we need to perform two transformations: a reflection in the x-axis and then a
vertical translation of one unit down. Therefore, point A(23, 1) will first be reflected to (23, 21) and then translated
vertically to A(23, 22).

A(23, 1)

A(23, 22)

14 The graph of the function y2 = f ( x − k ) is obtained by a horizontal translation of k units to the right. Since 0 < k < n − m,
the touching point with the x-axis is going to be translated horizontally and placed before the zero n.

5
Chapter 2

15 Firstly, we need to find the composite function and then we will find its inverse.
( f  g ) ( x ) = f ( g ( x )) = f ( x 3 ) = x 3 + 1 ⇒ x = y 3 + 1 ⇒ x − 1 = y 3 ⇒ (f  g) ( x ) =
−1 3
x −1
x x
 x  x +1 = x +1 x 1
16 a g ( x ) = ( f  f ) ( x ) = f (f ( x )) = f  = = , x ≠ −1, −
 x + 1 x x + x + 1 2x + 1 2
+1
x +1 x +1
2
 25 2
b ( g  g ) (2) = g ( g (2)) = g   = =
 52 9
2× +1
5
1 1 1 − 2x 2 2 2
17 a f ( x ) = −2 ⇒ 2 −20 ⇒  0 ⇒ 1 − 2x 2  0 ⇒ − x ,x ≠0
x 2
x x2 2 2
b Regarding the range, we notice that the sign before the square root is positive, and therefore f ( x )  0 .
2y + 1 x +1
18 x = ⇒ xy − x = 2 y + 1 ⇒ xy − 2 y = x + 1 ⇒ y ( x − 2) = x + 1 ⇒ f −1 ( x ) =
y −1 x−2
Since the denominator cannot be equal to zero, the domain of the inverse function is D (f −1 ) = {x ∈  x ≠ 2} .
2x − 1
19 a The rational function f ( x ) = , x > 0, has a horizontal asymptote at y = 2 because
x+2
0
1
2−
2x − 1 x = 2 = 2. On the other hand, since the restriction on the domain is to positive
lim f ( x ) = lim = lim
x →∞ x →∞ x + 2 x →∞ 2 1
1+
x

0

values of x, and the vertical asymptote without restriction would occur at x = −2, the minimum on the function takes
1
place at x = 0 ⇒ y = − .
2

{ 1
Therefore, the range is R (f ) = y ∈  −  y < 2 .
2 }
2y − 1 2x + 1
b x= ⇒ xy + 2x = 2 y − 1 ⇒ 2x + 1 = 2 y − xy ⇒ f −1 ( x ) = . Now, since there was a restriction on the
y+2 2−x
domain and range of the original function, we have a restriction on the domain of the inverse function:

{ 1
D (f −1 ) = x ∈  −  x < 2 .
2 }

20 a If f ( x ) = x 3 , then f ( g ( x )) = x + 1 ⇒ ( g ( x )) = x + 1 ⇒ g ( x ) =
3 3
x + 1.
b g (f ( x )) = x + 1 ⇒ g ( x 3 ) = x + 1 ⇒ g ( x ) = 3
x +1

21 a Since the surd expression is in the denominator, it cannot be equal to zero and hence:
{
3 − x 2 > 0 ⇒ D (f ) = x ∈  − 3 < x < 3 }

6
b The maximum value of the expression in the denominator is 3. Therefore, the range of the function is
 3
R (f ) =  y ∈  x  .
 3 

x +1
22 Given that ( f  g ) ( x ) = and g ( x ) = 2x − 1, we will firstly find f (x):
2
x +1 x +1
f ( g ( x )) = ⇒ f (2x − 1) =
2 2
t +1
To solve this type of functional equation, we will use a substitution, 2x − 1 = t ⇒ x = , and then transform the rule to
the new variable. 2
t +1 t + 1+ 2
+1
2 2 t+3 x −3+3 x
f (t ) = = = ⇒ f ( x − 3) = =
2 2 4 4 4

23 a The first transformation, y = f ( x − 4) , is a horizontal translation of four units to the right. Therefore,

A (−3, 25) → A ' (1, 25) ; B (0, 0) → B ' ( 4 , 0) ; C (3, −35) → C ' (7, −35); and D (6, 0) → D ' (10, 0) .

30 y
A A'
25

20

15

10

5
B B' D D' x

–9 –8 –7 –6 –5 –4 –3 –2 –1 1 2 3 4 5 6 7 8 9 10 11

–5

– 10

– 15

– 20

– 25

– 30

– 35

C C'
– 40

7
Chapter 2

b The second transformation, y = f (−3x ), actually consists of two transformations. One transformation is a reflection in
the y-axis, whilst the other is a horizontal shrinking by scale factor 3. So, the points will be mapped as follows:
A (−3, 25) → A " (1, 25) ; B (0, 0) → B " (0 , 0) ; C (3, −35) → C " (−1, −35) ; and D (6, 0) → D " ( −2, 0) .

30 y
A A"
25

20

15

10

5
D" B" B D x

−9 −8 −7 −6 −5 −4 −3 −2 −1 1 2 3 4 5 6 7 8 9 10 11

−5

−10

−15

−20

−25

−30

−35

C" C
−40

8
Chapter 3
Practice questions

1 x 2 − ( a + 3b )x + 3ab = 0
( a + 3b ) ± ( a + 3b )2 − 4 ⋅ 3ab ( a + 3b ) ± a2 + 6ab + 9b2 − 12ab
x1, 2 = =
2 2
( a + 3b ) ± a2 − 6ab + 9b2 ( a + 3b ) ± ( a − 3b )2 ( a + 3b ) ± ( a − 3b )
= = =
2 2 2
a + 3b + a − 3b a + 3b − a + 3b
x1 = = a, x2 = = 3b
2 2

2 3x − 2 4x − 1
+3 × 15
5 3
9 x − 6 + 45  20 x − 5
−11x  −44
x4
4 1
3 If, for parabola y = f ( x ) = 3x 2 − 8 x + c , the vertex is at  , −  , then:
3 3
2
 4 1  4  4 1 16 32 1 15
f   = − ⇒ 3⋅  − 8⋅  + c = − ⇒ − +c =− ⇒c = = 5.
 3 3  3   3  3 3 3 3 3

4 The quadratic function f ( x ) = a x 2 + b x + c :


i passes through ( 2, 4 ) ⇒ f ( 2) = 4
ii has a maximum value of 6 when x = 4 ⇒ f ( 4 ) = 6
iii has a zero of x = 4 + 2 3 ⇒ f ( 4 + 2 3 ) = 0
The zeros of the quadratic function are symmetrical about the axis of symmetry, i.e. x = 4, so the other zero is
x = 4 − 2 3. The function can be written as:
(
f ( x ) = a x − (4 + 2 3 )) (x − (4 − 2 3)) = a (x − 4 − 2 3) (x − 4 + 2 3)
( − (2 3 ) ) = a ( x − 8 x + 16 − 12) = a ( x − 8 x + 4)
2
= a ( x − 4)
2 2 2

1
Since f ( 2) = 4 ⇒ a ( 4 − 16 + 4) = 4 ⇒ −8a = 4 ⇒ a = −
2
1 2
The function is: f ( x ) = −
2
(x − 8 x + 4) = − 21 x 2 + 4 x − 2 ⇒ a = − 21 , b = 4 , c = −2.
5 The equation x 3 + 5x 2 + p x + q = 0 can be written in the factorized form as:
( x − ω ) ( x − 2ω ) ( x − ω − 3) = 0 ⇒ ( x 2 − 3ω x + 2ω 2 ) ( x − ω − 3) = 0 ⇒
x 3 − 3ω x 2 + 2ω 2 x − ω x 2 + 3ω 2 x − 2ω 3 − 3x 2 + 9ω x − 6ω 2 = 0
x 3 + (−4 ω − 3) x 2 + (5ω 2 + 9ω ) x + (−2ω 3 − 6ω 2 ) = 0
By comparing the corresponding coefficients, we obtain the following system of equations:
−4 ω − 3 = 5 ω = −2
 2 
5ω + 9 ω = p ⇒ p = 2
 
−2ω − 6ω = q q = −8
3 2

1
Chapter 3

6 The discriminant of the equation mx 2 − 2( m + 2)x + m + 2 = 0 is


∆ = (−2( m + 2)) − 4m( m + 2) = 4( m2 + 4m + 4 ) − 4m2 − 8m = 8m + 16.
2

a The equation has two real roots when ∆ > 0 ⇒ 8m + 16 > 0 ⇒ m > −2.
b The equation has one positive and one negative real root when ∆ > 0 and the first and the last coefficients are of
opposite sign. We have two possibilities:
m > 0 and m + 2 < 0 ⇒ m > 0 and m < −2, which is not possible; or
m < 0 and m + 2 > 0 ⇒ m < 0 and m < −2 , which gives us the solution −2 < m < 0.

7 For the polynomial f ( x ) = x 3 + a x 2 + b x + c , we are told that:


i x 2 1 is a factor ⇒ f (1) = 0
ii x 1 1 is a factor ⇒ f ( −1) = 0
iii division by x 2 2 gives a remainder of 12 ⇒ f ( 2) = 12
Therefore, we have the following system of equations:
1 + a + b + c = 0 a + b + c = −1
  −2b = 2 b = −1
−1 + a − b + c = 0 ⇒ a − b + c = 1 ⇒ ⇒ ⇒ −8 − 2 + c = −8 ⇒ c = 2
8 + 4 a + 2b + c = 0 4 a + 2b + c = −8 3a + b = −7 3a − 1 = −7 ⇒ a = −2
 

So: a = −2, b = −1, c = 2.

8 x <5 x−6
Since the expressions on both sides must be positive, we can square both sides and remove the absolute value signs.
x 2 < 25 ( x − 6)
2

x 2 < 25x 2 − 300 x + 900


24 x 2 − 300 x + 900 > 0
6 x 2 − 75x + 225 > 0
75 ± ( −75)2 − 4 ⋅ 6 ⋅ 225 75 ± 15
x1, 2 = =
2⋅6 12
90 15 60
x1 = = , x2 = =5
12 2 12
15
( x − 5)  x −  > 0
 2

We analyze the signs of both factors in a ‘sign chart’:


15
5 2
x
x−5 − 0 + +
15
x− − − 0 +
2
15
( x − 5)  x −  + 0 − 0 +
 2

15
The solution is: x < 5 or x > .
2

2
9 The equation 2x 2 + ( 3 − k )x + k + 3 = 0 has two imaginary solutions when its discriminant is negative.
∆ = ( 3 − k )2 − 4 ⋅ 2( k + 3) = 9 − 6k + k 2 − 8k − 24 = k 2 − 14 k − 15
k 2 − 14 k − 15 < 0
( k + 1)( k − 15) < 0

We analyze the signs of both factors in a ‘sign chart’:


21 15
x
k +1 − 0 + +

k − 15 − − 0 +

(k + 1) (k − 15) + 0 − 0 +

The solution is: −1 < k < 15.

2x 2 + 8 x + 7 2x 2 + 8 x + 10 − 3 2 ( x + 4 x + 5) − 3
2
10 a f (x) = = =
x2 + 4x + 5 x2 + 4x + 5 x2 + 4x + 5
3 3 3
=2− 2 =2− 2 =2−
x + 4x + 5 ( x + 4 x + 4) + 1 ( x + 2)2 + 1

b i lim f ( x ) = 2
x →+∞

ii lim f ( x ) = 2
x →−∞

3
c The minimum value occurs when is largest, that is, when x = −2.
( x + 2)2 + 1
3
f ( −2 ) = 2 − = −1
( −2 + 2)2 + 1
The minimum point is (−2, −1) .

11 The equation ( k − 2)x 2 + 4 x − 2k + 1 = 0 has two distinct real roots when its discriminant is positive.
∆ = 4 2 − 4 (k − 2) (−2k + 1) = 16 − 4 (−2k 2 − 4k − 2) = 8k 2 + 16k + 24
8k 2 + 16k + 24 > 0
k 2 + 2k + 3 > 0
In order to factorize this inequality, we try to solve the equation k 2 + 2k + 3 = 0 .
−2 ± 22 − 4 ⋅ 3 −2 ± −8
k1, 2 = =
2 2
Since this equation has no real solutions, the inequality is valid for all k ∈ ; so, the original equation has two distinct real
roots for all k ∈ .

12 When f ( x ) = 6 x 4 + 11x 3 − 22x 2 + a x + 6 is divided by ( x + 1) the remainder is −20 .


f ( −1) = −20 ⇒ 6 − 11 − 22 − a + 6 = −20 ⇒ a = −1

3
Chapter 3

13 When p( x ) = ( a x + b )3 is divided by ( x + 1) the remainder is −1 ⇒ p( −1) = −1 .


When divided by ( x − 2), the remainder is 27 ⇒ p( 2) = 27.
Therefore, we have the following system of equations:
(− a + b)3 = −1 − a + b = −1 4 1
 ⇒  ⇒ 3a = 4 ⇒ a = , b =
(2a + b) = 27
3
2 a + b = 3 3 3

14 When f ( x ) = x 3 + 3x 2 + a x + b is divided by ( x + 1) the remainder is the same as that when divided by ( x − 2).
f ( −1) = f ( 2)
−1 + 3 − a + b = 8 + 12 + 2a + b
2 − a = 20 + 2a
3a = −18
a = −6

15 When f ( x ) = x 4 + a x + 3 is divided by ( x − 1) the remainder is 8.


f (1) = 8 ⇒ 1 + a + 3 = 8 ⇒ a = 4

16 The polynomial g( x ) = x 3 + a x 2 − 3x + b is divisible by (x – 2) ⇒ g(2) = 0.


When divided by (x + 1) the remainder is 6 ⇒ g(–1) = 6.
We have the following system of equations:

8 + 4 a − 6 + b = 0  4 a + b = −2
 ⇒ ⇒ 3a = −6 ⇒ a = −2, b = 6
−1 + a + 3 + b = 6 a + b = 4

17 The polynomial g( x ) = x 2 − 4 x + 3 can be factorized as ( x − 1) ( x − 3), so if g( x ) is a factor of


f ( x ) = x 3 + ( a − 4 )x 2 + ( 3 − 4 a )x + 3, then both ( x − 1) and ( x − 3) are factors of f ( x ). We can use any of them in further
calculations:
f (1) = 0 ⇒ 1 + a − 4 + 3 − 4 a + 3 = 0 ⇒ −3a = −3 ⇒ a = 1

18 Given that (x + 2) is a factor of f ( x ) = x 3 − 2x 2 − 5x + k :


f ( −2) = 0 ⇒ −8 − 8 + 10 + k = 0 ⇒ k = 6

19 If 1+ ki is one zero of the polynomial z 2 + kz + 5, then 1− ki is the second, and the polynomial can be written as:
( z − (1 + ki )) ( z − (1 − ki )) = (( z − 1) − ki ) (( z − 1) + ki ) = ( z − 1)2 − ( ki )2 = z 2 − 2 z + 1 + k 2
By comparing the coefficients, we see that k = −2 and k 2 + 1 = 5.

20 The equation k x 2 − 3x + ( k + 2) = 0 has two distinct real roots when its discriminant is positive.
∆ = ( −3)2 − 4k ( k + 2) = 9 − 4k 2 − 8k
9 − 4 k 2 − 8k > 0
4 k 2 + 8k − 9 < 0
−8 ± 82 − 4 ⋅ 4 ⋅ ( −9 ) −8 ± 208 −8 ± 4 13 −2 ± 13
k1, 2 = = = =
2⋅4 8 8 4
k1 ≈ −2.80, k2 ≈ 0.803
(k + 2.8) (k − 0.803) < 0

4
We analyze the signs of both factors in a ‘sign chart’:
22.80 0.803
x

k + 2.80 − 0 + +

k − 0.803 − − 0 +

(k + 2.80) (k − 0.803) + 0 − 0 +

The solution is: −2.80 < k < 0.803.

21 The equation (1 + 2k )x 2 − 10 x + k − 2 = 0 has real roots when its discriminant is not negative.
∆ = ( −10 )2 − 4(1 + 2k )( k − 2) = 100 − 4( 2k 2 − 3k − 2) = −8k 2 + 12k + 108
−8k 2 + 12k + 108  0
2k 2 − 3k − 27  0
−27 ) 3 ± 15
3 ± ( −3)2 − 4 ⋅ 2 ⋅ (−
k1, 2 = =
2⋅2 4
9
k1 = −3, k2 =
2
9

2 (k + 3)  k −   0
 2
9
(k + 3)  k −   0
 2
We analyze the signs of both factors in a ‘sign chart’:
9
23 2
x

k+3 − 0 + +

9
k− − − 0 +
2
9
(k + 3)  k −  + 0 − 0 +
 2

9
The solution is: −3  k  .
2

22 To determine the range of values of m such that, for all real x, m(1 + x )  x 2 we transform the inequality into
x 2 − mx − m  0 and observe the function f ( x ) = x 2 − mx − m . The problem is now f ( x )  0. Since the leading
coefficient is 1, the graph of this quadratic function opens upwards; thus:
∆  0 ⇒ ( − m )2 − 4 ⋅ 1⋅ ( − m )  0 ⇒ m2 + 4m  0 ⇒ m( m + 4 )  0.
We analyze the signs of both factors in a ‘sign chart’:

5
Chapter 3

24 0
x
m+4 − 0 + +

m − − 0 +

m (m + 4) + 0 − 0 +

The solution is: −4  m  0.


23 5 − 3x  x + 1
Since the expressions on both sides must be positive, we can square both sides and remove the absolute value signs.
(5 − 3x )2  ( x + 1)2
25 − 30 x + 9 x 2  x 2 + 2x + 1
8 x 2 − 32x + 24  0
x2 − 4x + 3  0
( x − 1)( x − 3)  0
We analyze the signs of both factors in a ‘sign chart’:
1 3
x
x −1 − 0 + +

x−3 − − 0 +

( x − 1) ( x − 3) + 0 − 0 +

The solution is: 1  x  3.


3
24 x 2 − 4 + <0
x
3
x2 + <4
x
3
We will look for x ∈  so that the graph of the function f ( x ) = x 2 + is below the line y = 4.
x

For the right branch, we must change the window to see the intersections.

The solution is: −2.30 < x < 0 or 1 < x < 1.30 .

6
25 x − 2  2x + 1
Since the expressions on both sides must be positive, we can square both sides and remove the absolute value signs.
( x − 2)2  (2x + 1)2
x2 − 4x + 4  4x2 + 4x + 1
3x 2 + 8 x − 3  0
−8 ± 82 − 4 ⋅ 3 ⋅ ( −3) −8 ± 10
x1, 2 = =
2⋅3 6
2 1
x1 = −3, x 2 = =
6 3
1

3 ( x + 3)  x −   0
 3
1
( x + 3)  x −   0
 3
We analyze the signs of both factors in a ‘sign chart’:
1
23 3
x

x+3 − 0 + +
1
x− − − 0 +
3
1
( x + 3)  x −  + 0 − 0 +
 3
1
The solution is: −3  x  .
3
x+4 x−2
26 For f ( x ) = , x ≠ −1 and g( x ) = , x ≠ 4 we have:
x +1 x−4
f ( x )  g( x )
x+4 x−2

x +1 x − 4
x+4 x−2
− 0
x +1 x − 4
( x + 4) ( x − 4) − ( x − 2) ( x + 1)
0
( x + 1) ( x − 4)
x 2 − 16 − x 2 + x + 2
0
( x + 1) ( x − 4)
x − 14
0
( x + 1) ( x − 4)
The ‘sign chart’ for the inequality is: −1 4 14
x
x +1 − 0 + + +

x−4 − − 0 + +

x − 14 − − − 0 +
x − 14
− X + X − 0 +
( x + 1) ( x − 4)
The solution is: x < −1 or 4 < x  14.

7
Chapter 3

Solution Paper 1 type

27 x + 9  2
x−9
x+9
−2  2
x−9
We have two inequalities:

x+9 x+9
 −2 and 2
x−9 x−9
x+9 x+9
+20 −20
x−9 x−9
3x − 9 − x + 27
0 0
x−9 x−9
x−3 x − 27
0 0
x−9 x−9

We analyze the signs of the numerator and denominator for the first inequality in a ‘sign chart’:
3 9
x
x−3 − 0 + +

x−9 − − 0 +
x−3
+ 0 − X +
x−9

The first solution is: x  3 or x > 9.


For the second inequality:
9 27
x
x−9 − 0 + +

x − 27 − − 0 +

x − 27
+ X − 0 +
x−9

The second solution is: x < 9 or x  27.


The final solution is the intersection of the two solutions, i.e. x  3 or x  27.

Solution Paper 2 type

27

8
28 If x1 = 2 + i is the first root of the equation x 3 − 6 x 2 + 13x − 10 = 0, then x 2 = 2 − i is the second root, and the equation
can be factorized as:
( x − ( 2 + i )) ( x − ( 2 − i )) ( x − a) = 0
(( x − 2) − i ) (( x − 2) + i ) ( x − a) = 0
(( x − 2)2 − i 2 ) (x − a) = 0
(x 2 − 4 x + 5) (x − a) = 0
x 3 − a x 2 − 4 x 2 + 4 a x + 5x − 5a = 0
x 3 + ( − a − 4 )x 2 + ( 4 a + 5)x − 5a = 0
By comparing the coefficients with the coefficients of the original equation, we get:
− a − 4 = −6 ⇒ − a = −2 ⇒ a = 2
4 a + 5 = 13 ⇒ 4 a = 8 ⇒ a = 2
−5a = −10 ⇒ a = 2
We see that we get the solution a = 2 from all the equations, so the third root is x 3 = 2.

Solution Paper 1 type

2x
29 The inequality < 1 can be multiplied by the denominator since it is positive. Therefore: 2x < x − 1 , x ≠ 1 .
x −1
We have two possibilities:
If x − 1 > 0, i.e. x > 1 ⇒ x − 1 > 2x ⇒ x < −1, which is a contradiction.
1
If x − 1 < 0, i.e. x < 1 ⇒ x − 1 > −2x ⇒ 3x < 1 ⇒ x < , which is a solution.
3

Solution Paper 2 type

2x 2x
29 For the inequality < 1, we will observe where the graph of the function f ( x ) = is below the line y = 1.
x −1 x −1

1
The solution is: x < .
3

9
Chapter 4
Practice questions

1 a1 = 4
d We need to determine n so that bn  an . The easiest
a4 = 19
way of doing this is by entering n (as X), an (as Y1),
an = 99
and bn (as Y2) into Table in a GDC.
4 + 3d = 19 
 ⇒ d = 5 and n = 20
4 + ( n − 1)d = 99

2 A = 3000, r = 0.06, n = 4 , t = 6
nt 4⋅6
0.006 
A = P  1 +  ⇒ 3000 = P  1 +
r
 n  
4 
3000
P= 4⋅6 = 2098.63
 1 + 0.06 
 
4 
You should invest €2098.63 now.

3 Nicks’ studying hours form an arithmetic sequence with


first term a1 = 12 and common difference
d = 2. Charlottes’ studying hours form a geometric We see that in week 12 Nick will study for 34 hours
sequence with first term b1 = 12 and common ratio while Charlotte studies for 34.24 hours. So, Charlotte
r = 1.1. will catch up with Nick in week 12.
a a5 = a1 + 4 d = 12 + 4 ⋅ 2 = 20 4 Plan A forms an arithmetic sequence with first term
a1 = 1000 and common difference d = 80. Plan B forms
b5 = b1r 4 = 12 ⋅ 1.14 ≈ 17.57
a geometric sequence with first term b1 = 1000 and
In week 5, Nick studied for 20 hours and Charlotte common ratio r = 1.06.
studied for 17.57 hours. a b2 = 1000 ⋅ 1.06 = 1060 g
15 b3 = 1000 ⋅ 1.062 = 1123.6 g
b Sarithmetic15 = [2 ⋅ 12 + (15 − 1)2] = 390
2 b a12 = a1 + 11d = 1000 + 11⋅ 80 = 1880 g
1.115 − 1
Sgeometric15 = 12 ≈ 381.27 b12 = b1 ⋅ 1.0611 ≈ 1898.33 g
1.1 − 1 12
For the 15 weeks, Nick studied for a total of 390 hours c i SA12 = (2a1 + 11d ) = 6 (2 ⋅ 1000 + 11⋅ 80)
2
and Charlotte studied for a total of 381.27 hours. = 17 280 g
40 rn − 1 1.0612 − 1
log ii SB12 = b1 = 1000 ≈ 16 869.9 g
c n−1
bn > 40 ⇒ 12 ⋅ 1.1 > 40 ⇒ n > 12 + 1⇒ r −1 1.06 − 1
log 1.1
5 a The initial amount forms a geometric sequence with
n > 13.6
a1 = 500 and common ratio r = 1.06 (fixed rate 6%
Charlotte will exceed 40 hours of study per week in per annum).
the 14th week.
After 10 years it will be worth:
a11 = a1r 10 = 500 ⋅ 1.0610 = €895.42 = €895 to the
nearest euro.

1
Chapter 4

b The future value is a partial sum of a geometric 10 a1 = 25


sequence: a4 = 13 ⇒ a1 + 3d = 13 ⇒ 25 + 3d = 13 ⇒ d = −4
 r 11 − 1   1.0611 − 1  an = −11995 ⇒ a1 + (n − 1) d = −11995 ⇒
FV = a1  − 1 = 500  −1
 r −1   1.06 − 1  25 + (n − 1) ( −4) = −11995 ⇒ n = 3006
= €6985.82 = €6986 to the nearest euro. 2 2
 1  1 2 2
6 6, 9.5, 13, … is an arithmetic sequence with 11 a MN =   +   = =
 2  2 4 2
a1 = 6 and d = 3.5. 2
a a40 = a1 + 39d = 6 + 39 ⋅ 3.5 = 142.5  2 1
b AreaMNPQ =   =
103  2  2
b S103 = [2a1 + (103 − 1)d ]
2  1 2
2
 1 2
2
1 1 1
103 c i RS =  ⋅  + ⋅ = + =
= (2 ⋅ 6 + 102 ⋅ 3.5)  2 2   2 2  8 8 2
2 2
= 19 003.5  1 1
Area RSTU =   =
 2 4
7 For an = 3 8 − an3−1 : 1 1
1 1 1
a a1 = 1, a2 = 3 8 − 13 = 3 7 , a3 = 3 8 − ( 3 7 )3 = 1 ii 1, , , ... ⇒ r = 2 = 4 =
2 4 1 1 2
b a1 = 2, a2 = 3 8 − 23 = 0, a3 = 3 8 − 03 = 2 9
2
 1 1
d i Area10 = 1   =
 2 512
8 The training programme forms an arithmetic sequence
with a1 = 2 and d = 0.5. a1 1
ii S∞ = = =2
a an = 20 ⇒ 2 + ( n − 1) ⋅ 0.5 = 20 ⇒ n = 37 1− r 1− 1
2
She first runs a distance of 20 km on the 37th day of
12 Tim’s swimming programme forms an arithmetic
her training.
sequence with a1 = 200 and d = 20.
37
b S37 = (2 ⋅ 2 + 36 ⋅ 0.5) = 407 a a52 = a1 + 51d = 200 + 51⋅ 20 = 1220
2
The total distance run during 37 days of training Tim will swim 1220 metres in the final week.
would be 407 km. 52 52
b S52 = (2a1 + 51d ) = (2 ⋅ 200 + 51⋅ 20) = 36 920
2 2
2400 3600 3 Altogether, Tim swims 36 920 metres.
9 a r= = =
1600 2400 2 2
3
b The number of new participants in 2012 is the 13th 13 a Area A =   = 1
 3
term in the sequence. 2
 1 1
 3
12
AreaB =   =
a13 = a1r 12 = 1600 ⋅   = 207 594  3 9
 2
2
3
n−1  1 1
c an > 50 000 ⇒ 1600   > 50 000 ⇒ b AreaC =   =
 2  9 81
1 8
n−1 c Shaded area2 = 1 + 8 ⋅ = 1 +
 3 log 31.25 9 9
  > 31.25 ⇒ n > + 1⇒
2 log 1.5 8
Shaded area3 = Shaded area2 + 8 ⋅
n > 9.489 81
2
The 10th term of the sequence will be greater than 8  8
= 1+ + 
9  9
50 000; therefore, the number of new participants will
2
first exceed 50 000 in 2009. 8  8 1
d Shaded area∞ = 1 + +   + ... = =9
9  9 1−
8
r 13 − 1 1.513 − 1 9
d S13 = a1 = 1600 = 619 582
r −1 1.5 − 1 Unshaded area∞ = 9 − Shaded area∞ = 0
e This trend in growth would not continue due to
market saturation.

2
14 a i The series 2 + 22 + 222 + 2222 + ... is neither 16 The number of seats in each row forms an arithmetic
arithmetic nor geometric. sequence with a1 = 16 and d = 2.
4 8 16
ii The series 2 + + + + ... is geometric with a a24 = a1 + 23d = 16 + 23 ⋅ 2 = 62
3 9 27
2 24
r = < 1; thus converging. b S24 = 16 + 18 + ... + 62 = (16 + 62) = 936
3 2
iii The series 0.8 + 0.78 + 0.76 + 0.74 + ... is 17 The values of the investment after each year form a
arithmetic with d = 20.02. geometric sequence with a1 = 7000, r = 1.0525, and
8 32 128 an+1 represents the value of the investment after n years.
iv The series 2 + + + + ... is geometric
3 9 27
4 a Value of investment = 7000 ⋅ 1.0525t
with r = > 1; thus diverging.
3 10
2 b 7000 ⋅ 1.0525t = 10 000 ⇒ 1.0525t = ⇒
b For series ii we have: S∞ = = 6. 7
2
1− 10
3 log  
7
15 The Kell scheme forms an arithmetic sequence with t= = 6.97
log(1.0525)
a1 = 18 000 and d = 400. The IBO scheme forms a
The minimum number of years is 7.
geometric sequence with b1 = 17 000 and r = 1.07.
c If the rate of 5% is compounded quarterly, the value
a All answers are in euros.
of the investment over 7 years would be:
i Kell: a2 = 18 000 + 400 = 18 400, 7⋅ 4
 5 
a3 = a2 + 400 = 18 800 7000 ⋅  1 +  = 9911.95.
 4 ⋅ 100 
IBO: b2 = 17 000 ⋅ 1.07 = 18 190 , For 5.25% compounded annually, the value of the
b3 = b2 ⋅ 1.07 = 19 463.30 investment would be 7000 ⋅ 1.05257 = 10 015.04 .
10
ii Kell: S10 = ( 2 ⋅ 18 000 + 9 ⋅ 400 ) Therefore, the investment at 5.25% annually is better.
2
= 198 000 18 a S1 = 9 ⇒ a1 = 9
1.0710 − 1 S2 = 20 ⇒ a1 + a2 = 20 ⇒ 9 + a2 = 20 ⇒ a2 = 11
IBO: S10 = 17 000
1.07 − 1
= 234 879.62 b d = a2 − a1 = 11 − 9 = 2
iii Kell: a10 = 18 000 + 9 ⋅ 400 c a4 = a1 + 3d = 9 + 3 ⋅ 2 = 15
= 21600
19 a2 = a + d = 7
IBO: b10 = 17 000 ⋅ 1.079 
 4
= 31253.81 S4 = 2 [2a + 3d ] = 12
b i From a ii we can see that b3 > a3 , so Merijayne a + d = 7
will start earning more than Tim in the third year.  ⇒ a = 15, d = −8
2a + 3d = 6
ii We can compare their total earnings with 20 (1 + x ) (1 + a x )6 = 1 + b x + 10 x 2 + … + a 6 x 11
5

the help of a GDC. If X denotes the year, Y1


(1 + 5x + 10 x 2 + 10 x 3 + 5x 4 + x 5 )(1 + 6a x + 15a 2 x 2
represents total earnings for Tim and Y2 for
Merijayne, we have: + 20a 3 x 3 + 15a 4 x 4 + 6a 5 x 5 + a 6 x 6 )
= 1 + b x + 10 x 2 + … + a 6 x 11
1 + 5x + 6a x + 10 x 2 + 15a2 x 2 + 30a x 2 + … + a 6 x 11
= 1 + b x + 10 x 2 + … + a 6 x 11
By comparing the coefficients of the same powers of x,
we get:
5 + 6a = b
In the fourth year, Merijayne’s total earnings will 
10 + 15a + 30a = 10
2

exceed those of Tim. a( a + 2) = 0 ⇒ a = −2, b = −7

3
Chapter 4

 1 
7 7
   1 
i

21 a5 : a12 =
6
28  x + 2  =
 ax 
∑  7i  x 7−i
 2 
ax
 13 i =0
a1 ⋅ a3 = 32
7  1 
2
1 21x
a + 4d 6 i = 2 ⇒   x 7 − 2  2  = 21x 5 ⋅ 2 4 = 2
= ⇒ a = 2d  2  ax  ax a
a + 11d 13 21 7
a( a + 2d ) = 32 ⇒ 2d ⋅ 4 d = 32 ⇒ = ⇒ a2 = 9 ⇒ a = 3, a = −3
a2 3
d = 2 (all terms positive), a = 4
a 27 27(1 − r )
100 29 S∞ = = ⇒a=
S100 = (2 ⋅ 4 + 99 ⋅ 2) = 10 300 1− r 2 2
2
1− r3
22 22n − 3n − n = 18n = 9 ⋅ 2n which is clearly divisible by 9 S3 = a = 13
1− r
for n = 1, 2, ….
27 (1 − r ) 1 − r 3 1 1
23 a1 = 5 ⋅ = 13 ⇒ r 3 = ⇒r = ,a = 9
2 1− r 27 3
a2 = a1 + d = 13 ⇒ d = 8
 6
30 Student A can get 1 coin in   ways.
a an = a1 + ( n − 1)d = 5 + ( n − 1) ⋅ 8 = 8n − 3 1 
 6
b an < 400 ⇒ 8n − 3 < 400 ⇒ n < 50.375 Student A can get 2 coins in   ways.
 2
There are 50 terms which are less then 400.
 6
10
 10  10−i Student A can get 3 coins in   ways.
24 ( 2 + 3x )10 = ∑   3
 2 ( 3x )
i

i =0  i   6
Student A can get 4 coins in   ways.
 10  10−7  4
i =7⇒ 2 ( 3x )7 = 120 ⋅ 8 ⋅ 2187 x 7
 7   6
Student A can get 5 coins in   ways.
= 2 099 520 x 7  5
The coefficient of x 7 is 2 099 520. Altogether, there are
 6  6  6  6  6
25 Sn = 3n2 − 2n ⇒  1  +  2 +  3 +  4 +  5 = 6 + 15 + 20 + 15 + 6 = 62
S1 = u1 = 3 ⋅ 12 − 2 ⋅ 1 = 1 ways.
S2 = u1 + u2 = u1 + u1 + d = 3 ⋅ 22 − 2 ⋅ 2 = 8 ⇒
31 This is an infinite geometric series with:
2+d = 8⇒d = 6
2
un = u1 + ( n − 1)d = 1 + ( n − 1) ⋅ 6 = 6n − 5 a1 = −12, r = − . So:
3
26 Six people can be ordered in 6! ways, but, as they are a1 −12 −36
S∞ = = = .
seated around a circular table, all circular permutations 1− r 1+ 2 5
3
that come in groups of six (ABCDEF, BCDEFA, CDEFAB,…)
6! 32 For un = 3 ( 4 )n+1 , n ∈ + :
are equivalent, so there are actually ways. As Mr Black
6
and Mrs White should not sit together, we must subtract a u1 = 48, r = 4
all circular permutations in which this pair is regarded as 4n − 1
b Sn = 48 = 16( 4 n − 1)
one person, but multiplied by 2, because a male can be 4 −1
on the left or right of the female. Altogether, there are 33 For the infinite geometric series
6! 5!
− ⋅ 2 = 120 − 24 ⋅ 2 = 72 ways. 2x 2x
2
2x
3
6 5 1 +   +   +   + ... :
 3  3  3
27 Firstly, we must determine which is the last positive term 2x
in the sequence. a a1 = 1, r =
3
a1 = 85, d = −7 The series converges for
an = 85 + ( n − 1)( −7 ) > 0 ⇒ 92 − 7n > 0 ⇒ n < 13.14 2x 3 3
13 r < 1⇒ < 1⇒ − < x < .
For n = 13 ⇒ S13 = [2 ⋅ 85 + 12 ⋅ ( −7 )] = 559. 3 2 2
2

4
2 ⋅ 1.2 4 39 a S2 = a + ar = 15
b x = 1.2 ⇒ r = = 
3 5  a
1 S∞ = 1 − r = 27
S∞ = =5
4
1− a(1 + r ) = 15
5  ⇒
a = 27(1 − r )
34 There are 9999 − 999 = 9000 four-digit numbers.
27(1 − r )(1 + r ) = 15 ⇒ 27(1 − r 2 ) = 15 ⇒
Without digit 3 there are 8 ⋅ 9 ⋅ 9 ⋅ 9 = 5832 numbers. 4 2
r 2 = ⇒ r = (all term ms positive)
9 3
So, with at least one digit 3, there are
 2
9000 – 5832 = 3168 numbers. b a = 27 1 −  = 9
 3
35 For the arithmetic series we have: a1 = 2, d = 3. 40 For the arithmetic sequence 2, a − b , 2a + b + 7, a − 3b ,
n n( 3n + 1) the difference must be constant.
a Sn = [2 ⋅ 2 + ( n − 1) ⋅ 3] =
2 2 So, we have the following system of equations:
n( 3n + 1) ( a − b ) − 2 = ( 2a + b + 7 ) − ( a − b )
b Sn = 1365 ⇒ = 1365 ⇒ 
2 ( a − 3b ) − ( 2a + b + 7 ) = ( 2a + b + 7 ) − ( a − b )
3n2 + n − 2730 = 0 ⇒
a − b − 2 = a + 2b + 7
−1 ± 12 − 4 ⋅ 3 ⋅ ( −2730 ) 91 
n= = 30, − − a − 4 b − 7 = a + 2b + 7
2⋅3 3 3b = −9
 ⇒ a = 2, b = −3
 8   1 i
8 a + 3b = −7
36 1 8
(1 −
2
x) = ∑  i   − 2 x  41 Let A and B denote the two oldest children that cannot
i =0
both be chosen. Four children can be then be chosen as
 8   1 3  1 
i = 3 ⇒    − x  = 56  − x 3  = −7 x 3 4 out of 6 (without A and B), or as 3 out of 6 and 1 out of
 3 2   8 
2 (A or B). Altogether, there are
The coefficient of x 3 is −7.  6  6  2
 4 +  3  1  = 15 + 20 ⋅ 2 = 55 ways.
50 50 50
37 ∑ ln(2 ) =∑ r ln( 2) = ln 2∑ r
r
42 As a, 1, b form an arithmetic progression, the difference is
r =1 r =1 r =1

50 the same: 1 − a = b − 1 .
= (ln 2)  ( 2 + 49 )
2 
As 1, a, b form a geometric progression, the ratio is the
= 1275 ln 2 a b
same: = .
38 For u0 = 1, u1 = 2, un+1 = 3un − 2un−1 , n ∈ + : 1 a
We solve the following system of equations:
a u2 = 3u1 − 2u0 = 4 1 − a = b − 1

u3 = 3u2 − 2u1 = 8 a b
u4 = 3u3 − 2u2 = 16  1 = a

b i un = 2n a + b = 2
 2 ⇒ a2 + a − 2 = 0 ⇒ ( a − 1)( a + 2) = 0
a = b
ii 3un − 2un−1 = 3 ⋅ 2n − 2 ⋅ 2n−1
⇒ a = 1 or a = −2
= 3 ⋅ 2n − 2n As a = 1 gives b = 1, the solution is a = −2, b = 4.
= 2 ⋅ 2n 43 We can see that:
= 2n+1 = un+1 OB = OA = 1
OB1
cos θ = ⇒ OB1 = OA1 = OA cos θ = cos θ
OA
OB
cos θ = 2 ⇒ OB2 = OA2 = OA1 cos θ = cos2 θ
OA1
OB3
cos θ = ⇒ OB3 = OA3 = OA2 cos θ = cos3 θ
OA2

5
Chapter 4

The radii form the geometric sequence 47 Let S(n) be the statement: 5n + 9 n + 2 is divisible by 4, for
1, cos θ , cos2 θ , cos3 θ , ... n ∈ + .
As the length of the arc is equal to θ ⋅ radius, the sum of
Basis step:
the arc lengths is:
S(1):
AB + AB1 1 + A2B2 + A3B3 + ... =
Sn = 4 n2 − 2n
θ
θ + θ cos θ + θ cos2 θ + θ cos3 θ + ... =
1 − cos θ 51 + 91 + 2 = 16 which is divisible by 4.
+
44 For Sn = 2n − n, n ∈  , we have:
2
Inductive step:
a S1 = u1 = 2 ⋅ 12 − 1 = 1 Assume S(k) is true, i.e. assume that 5k + 9k + 2 is
S2 = u1 + u2 = 2 ⋅ 22 − 2 = 6 ⇒ divisible by 4. So, we assume that
u1 + u2 = 6 ⇒ 1 + u2 = 6 ⇒ u2 = 5 5k + 9k + 2 = 4 A, A ∈  (*) ⇒ 9k = 4 A − 5k − 2 .
S3 = u1 + u2 + u3 = 2 ⋅ 32 − 3 = 15 ⇒ Then S(k + 1):
u1 + u2 + u3 = 15 ⇒ 1 + 5 + u3 = 15 ⇒ u3 = 9 5k +1 + 9k +1 + 2 = 5k +1 + 9 ⋅ ( 4 A − 5k − 2) + 2
= 5 ⋅ 5k + 36 A − 9 ⋅ 5k − 18 + 2
b un = Sn − Sn−1 = 2n − n − 2( n − 1) − ( n − 1)
2 2

= 36 A − 4 ⋅ 5k − 16
= 2n − n − 2n − 4 n + 2 − n + 1
2 2
= 4( 9 A − 5k − 4 ),
= 4n − 3
i.e. 5k + 9k + 2 is divisible by 4.
5
 
45 a ( 2 + x )5 = ∑  5i  2 5− i
xi = This shows that S(k + 1) is true whenever S(k) is true.
i =0

 5  5−0 0  5  5−1 1  5  5−2 2  5  5−3 3 Therefore: 5n + 9 n + 2 is divisible by 4, for n ∈ + .


 0  2 x +  1  2 x +  2  x x +  3  2 x
48 For an arithmetic sequence with Sn = 4 n2 − 2n we have:
 5  5
+   2 5− 4 x 4 +   2 5− 5 x 5 u2 = S2 − S1 = ( 4 ⋅ 22 − 2 ⋅ 2) − ( 4 ⋅ 12 − 2 ⋅ 1)
 4  5
= 12 − 2 = 10
= 32 + 5 ⋅ 16 x + 10 ⋅ 8 x 2 + 10 ⋅ 4 x 3 + 5 ⋅ 2x 4 + x 5
um = Sm − Sm−1 = ( 4m2 − 2m) − 4( m − 1)2 − 2( m − 1)
= 32 + 80 x + 80 x 2 + 40 x 3 + 10 x 4 + x 5
= 4m2 − 2m − 4m2 − 8m + 4 − 2m + 2
b 2.01 = ( 2 + 0.01) = 32 + 80 ⋅ 0.01 + 80 ⋅ 0.01
5 5 2
= 8m − 6
+ 40 ⋅ 0.013 + 10 ⋅ 0.014 + 0.015 = 32 + 0.8 + 0.008 u32 = S32 − S31 = ( 4 ⋅ 3 = 22 − 2 ⋅ 32) − ( 4 ⋅ 312 − 2 ⋅ 31)
+ 0.000 04 + 0.000 0001 + 0.0000 000 0001 = 250
= 32.808 040 1001
As they are consecutive terms in a geometric sequence,
46 The interest is paid yearly. For n years, rate r = 0.063, and
the ratio is the same.
principal P = 5000 after n full years:
8m − 6 250
a A n = P (1 + r ) = 5000 (1.063)
n n = ⇒
10 8m − 6
b A 5 = 5000 (1.063) = 6786.35
5 ( 8m − 6 )2 = 2500 ⇒
8m − 6 = 50 ⇒ m = 7
c i 5000 (1.063) > 10 000
n

log 2
ii (1.063)n > 2 ⇒ n > = 11.35
log 1.063
The value will double after 12 full years.

6
Chapter 5
Practice questions

1 a To find the x-coordinate of P we have to solve the equation:

2 − log3 ( x + 1) = 0 ⇒ log3 ( x + 1) = 2 ⇒ x + 1 = 32 ⇒ x = 8. So, the point is P(8, 0) and it’s x-coordinate is 8.

b We will find the y-coordinate of Q by finding the value of the function for x = 0:

2 − log3 (0 + 1) = 0 ⇒ 2 − log3 1 = 2 ; hence, the y-coordinate of Q is 2.

The y-coordinate of R is 3. To find the x-coordinate of Q we have to solve the equation:


c
2 2
2 − log3 ( x + 1) = 3 ⇒ log3 ( x + 1) = −1 ⇒ x + 1 = 3−1 ⇒ x = − . Hence, the point is R  − , 3 .
3  3 
−0.0045⋅800 5
2 a We have to solve the equation A( 800 ) = 5 . Hence, 5 = A0 e ⇒ A0 = −3.6 ≈ 183 .
e

So, approximately 183 grams were present initially.


A
b We have to solve the equation A(t ) = 0 . Hence,
2 1
ln  
A0 −0.0045⋅t 1 −0.0045⋅t  1  2
= A0 e ⇒ =e ⇒ ln   = −0.0045t ⇒ t = ≈ 154.
2 2  2 −0.0045
So, the half-life of the substance is approximately 154 years.
183
Note: We could have started by using the result from a such that we would have to solve: = 183e −0.0045⋅t .
2
However, it is better, if possible, to use only the data given in the question text. In this way, possible mistakes made in
previous calculations are not carried through a problem.

3 a Using the properties of logarithms, we can write the terms of the sequence differently:
ln y + ln y 2 + ln y 3 + ... = ln y + 2 ln y + 3 ln y + .... Now, we can see that the sequence is arithmetic with first term
u1 = ln y , and common difference d = 2 ln y − ln y = ln y . Hence, un = ln y + ( n − 1) ln y. Using the property of
logarithms, we can simplify this expression: un = ln y + ( n − 1) ln y = 1 ⋅ ln y + ( n − 1) ln y = (1 + n − 1) ln y = n ln y .
n n n n n (n + 1)
Sum of Sn = (ln y + ln y n ) = ln ( yy n ) = ln y n+1 = (n + 1) ln y. So, Sn = ln y.
2 2 2 2 2
b Using the properties of logarithms, we can write the terms of the sequence differently:
ln ( xy ) + ln ( xy 2 ) + ln ( xy 3 ) + ... = (ln x + ln y ) + (ln x + 2 ln y ) + (ln x + 3 ln y ) + ....

We can see that each term of this sequence matches the corresponding term of the sequence in part a plus
n n n
n (n + 1)
ln x. Hence, un = ln x + ln y n , and the sum: Sn = ∑ (ln x + ln y i ) = ∑ ln x + ∑ ln y i = n ln x + ln y .
i =1 i =1 i =1 2
4 Using the properties of logarithms, we can transform the equation:
log2 (5x 2 − x − 2) = 2 + 2 log2 x ⇒ log2 (5x 2 − x − 2) = log2 4 + log2 x 2 ⇒ log2 (5x 2 − x − 2) = log2 4 x 2
5x 2 − x − 2 = 4 x 2 ⇒ x 2 − x − 2 = 0 ⇒ x1 = −1, x 2 = 2
Since, log2 x is not defined for −1, the only solution is x = 2.

1
Chapter 5

1 5
2+ 5
5 We can determine the value of x: x = log2 4 2 = log2 2 2
= log2 2 2 = .
2
From logz y = 4 we can conclude that: y = z 4 .
2
 5 5
From the third equation, we can conclude: z 4 = 4 ⋅   − 2 − 6 + z = 25 − 5 − 6 + z = 14 + z ⇒ z 4 − z − 14 = 0
 2 2
We will solve this equation using the PolySmlt/PolyRootFinder application:

From logz y = 4, we can conclude that the base of the logarithm should be positive; hence, z = 2, and thus y = z 4 = 16.

6 Transforming the equation: 2e 3t − 7e 2t + 7e t = 2 ⇒ 2e 3t − 7e 2t + 7e t − 2 = 0

Grouping the terms with the same coefficients: 2 (e 3t − 1) − 7 (e 2t − e t ) = 0 ⇒ 2 (e 3t − 1) − 7e t (e t − 1) = 0

Using the formula for the difference of cubes:

2 (e t − 1) (e 2t + e t + 1) − 7e t (e t − 1) = 0 ⇒ (e t − 1) (2e 2t + 2e t + 2 − 7e t ) = 0 ⇒ (e t − 1) (2e 2t − 5e t + 2) = 0

Hence, either e t − 1 = 0 ⇒ e t = 1 ⇒ t = 0, or 2e 2t − 5e t + 2 = 0 ⇒ 2 (e t ) − 5e t + 2 = 0 .
2

1
We can solve the second equation by using a substitution: e t = s ⇒ 2s 2 − 5s + 2 = 0 ⇒ s1 = , s2 = 2 ; therefore:
1 1 2
e t = ⇒ t = ln , or e t = 2 ⇒ t = ln 2 .
2 2
1 , ln 2.
Solutions are: t = 0, ln __
2
7 Using the substitution ln x = t:
−2e ± 4 e 2 + 32e 2 −2e ± 6e
8e 2 − 2et = t 2 ⇒ t 2 + 2et − 8e 2 ⇒ t1, 2 = = ⇒ t1 = −4 e , t2 = 2e
2 2
Hence:
ln x = −4 e ⇒ x = e −4 e

ln x = 2e ⇒ x = e 2 e
log3 3 1
8 a Using the change of base formula: log3 x − 4 + 3 = 0 ⇒ log3 x − 4 +3= 0
log3 x log3 x
Multiplying by log3 x : (log3 x )
2
− 4 + 3 log3 x = 0

Using the substitution log3 x = t : t 2 + 3t − 4 = 0 ⇒ t1 = −4 , t2 = 1

Therefore, the solutions are:


1
log3 x = −4 ⇒ x = 3−4 =
81
log3 x = 1 ⇒ x = 3
1
We can see from the original equation that x has to be positive and x ≠ 1, so x = and x = 3 are both solutions.
81
b Using the properties of logarithms:
log2 ( x − 5) + log2 ( x + 2) = 3 ⇒ log2 ( x − 5)( x + 2) = 3 ⇒ ( x − 5)( x + 2 ) = 23 ⇒ x 2 − 3x − 10 = 8

The solutions of the quadratic equation are 6 and −3. Since x has to be greater than 5 (for log2 ( x − 5) and log2 ( x + 2)
to be defined), the only solution is x = 6.

2
9 a First using the property k logb M = logb (M k ) and then the properties of the sum and difference of logarithms, we have:
a2 b3
2 log a + 3 log b − log c = log a2 + log b3 − log c = log (a2 b3 ) − log c = log .
c
b First using the property k logb M = logb (M k ) and then ln e = 1, and, finally, the properties of the sum and difference
of logarithms, we have:
1 1
ex3
3 ln x − ln y + 1 = ln x 3 − ln y 2 + ln e = (ln x 3 + ln e ) − ln y = ln (e x 3 ) − ln y = ln .
2 y
10 Given that A(t ) = 0.79 A0 , we have to solve the equation: 0.79 A0 = A0 e −0.000124 t . So, we have:
ln 0.79
0.79 = e −0.000124 t ⇒ ln 0.79 = −0.000 124t ⇒ t = ≈ 1900 years.
−0.000 124

11 c is solution of the equation: 0 = log3 (2 x − 3) − 4 ⇒ log3 (2 x − 3) = 4 ⇒ 2 x − 3 = 34 ⇒ 2 x = 81 + 3 ⇒ x = 42


So, c = 42.

12 a b
y y
y = b1–x
y=b –x
y = bx
y = bx

(0, b) (1, b)
(–1, b) (1, b)

(0, 1) (0, 1) (1, 1)

x x

t
 1 1600
13 a Since its half-life is 1600 years, the exponential decay model is: A(t ) = A0   . So, we have to solve:
t  2
 1 1600
A0   = A0 e − kt . Since the equation holds for all t, then:
 2
1 1
 1 1600  1 1600 1  1
  = e − k ⇒ ln   = −k ⇒ k = − ln   ≈ 0.000 4332 (4 s.f.)
2  2  1600  2 

3
Chapter 5

4000
1 1600
Ao  
A( 4000 )  2  1
2.50

b We have to find: = =  = 0.176 7766... ≈ 17.7%


A0 Ao  2
Note: We would get the same result if we used A(t ) = A0 e −0.0004332t . Using the result from a, we have:
e −0.0004332⋅4000 ≈ e −1.7328 ≈ 17.7%.

14 This equation cannot be solved exactly, so we will use a GDC. We transform the equation: e − x = x − 1. We know the
behaviour of both functions, so we can determine the number of solutions on the graph, and hence find all the solutions.

So, there is only one solution:

Solution is: x ≈ 1.28 .

Note: If we graph the original equation to find the zeros, we can’t be sure that our window is good enough to find all the
values.

In this case, we will find the solution:

15 This inequality cannot be solved exactly, so we will use a GDC.

We have to change the window to see all the regions where the logarithmic function is above the graph of the absolute
value. We will take into account that the logarithm is defined for positive values, and that the equation 0.1x 2 − 2x + 3 = 0
has two positive solutions ( x1 ≈ 1.64 , x 2 ≈ 18.36 ), so the absolute value function will decrease to zero once again.

4
So, the first interval is 1.52 < x < 1.79 and the second is 17.6 < x < 19.1.

Hence, the solution set is: ] 1.52, 1.79 [  ] 17.6, 19.1[


16 This inequality cannot be solved exactly, so we will use a GDC.

xex
The function f ( x ) = has vertical asymptotes x = −1, x = 1; so, we have to explore three regions. For x > 1, the
x2 − 1
graph of y = f ( x ) is above the line y = 1; hence, x > 1 is in the solution set.

For −1 < x < 1, we have:

So, for −1 < x < −0.800 , the graph of y = f ( x ) is above the line y = 1; hence, −1 < x < −0.800 is in the solution set.

For x < –1, the graph of y = f ( x ) is below the line y = 1; hence, there are no solutions for x < 21.

So, the solution set is: ]−1, −0.800[  ] 1, +∞ [


1
17 a Transforming the equation: 2 ( 4 x ) + 4 − x = 3 ⇒ 2 ( 4 x ) + = 3 ⇒ 2 (4 x ) − 3 (4 x ) + 1 = 0
2

4x
1
Using the substitution 4 x = t : 2t 2 − 3t + 1 = 0 ⇒ t1 = , t2 = 1
2

5
Chapter 5

Hence,
1
1 − 1
4x = ⇒ 4x = 4 2 ⇒ x = −
2 2
4x = 1 ⇒ x = 0
1
b i a x = e 2 x +1 ⇒ ln a x = 2x + 1 ⇒ x ln a − 2x = 1 ⇒ x (ln a − 2) = 1 ⇒ x =
ln a − 2
Note: Instead of taking ln from both sides of the equation, we can take loga . Then we will have:

a x = e 2 x +1 ⇒ x = loga e(2 x +1) ⇒ x = ( 2x + 1) loga e ⇒ x − 2x loga e = loga e

loga e
⇒ x (1 − 2 loga e ) = loga e ⇒ x =
1 − 2 loga e
loga e
ii The equation will not have a solution when is not defined. Hence,
1 − 2 loga e
1
1
1 − 2 loga e = 0 ⇒ loga e = ⇒ e = a 2 ⇒ e2 = a
2
18 Transforming the equation:

22 x + 3 = 2x +1 + 3 ⇒ 23 ⋅ 22 x − 2 ⋅ 2x − 3 = 0 ⇒ 8 ⋅ (2x ) − 2 ⋅ 2x − 3 = 0
2

1 3
Using the substitution 2x = t : 8 ⋅ t 2 − 2 ⋅ t − 3 = 0 ⇒ t1 = − , t3 =
2 4
Hence,
1
2x = − has no solution
2
x 3 3
2 = ⇒ x = log2 = log2 3 − log2 4 = log2 3 − 2 ; so, a = 22 and b = 3.
4 4
1
19 Using the substitution ln x = t : 2t 2 = 3t − 1 ⇒ 2t 2 − 3t + 1 = 0 ⇒ t1 = , t2 = 1. Hence, the solutions are:
1 2
1
ln x = ⇒ x = e 2 = e
2
ln x = 1 ⇒ x = e
20 a We use the exponential function for compound interest: V (t ) = 100 (1 + 0.05) . The value after 20 years is:
t

V ( 20 ) = 100 (1.05) = 265.33 (dollars).


20
t
 5 
 
b We use the exponential function for compound interest: A(t ) = 100 1 + 12  , where time is given in months. So,
100
 
we have to solve A(t ) > 265.33.
t t
 1   241 241 241
100  1 +  > 265.33 ⇒   = 2.6533 ⇒ t lo
og > log 2.6533. Dividing the inequality by log
 240   240  240 240
log 2.6533
(which is positive), we have: t > ≈ 234.68. So, the value of the investment will exceed V after 235 months.
241
log
240
Note: Since we are not looking for the exact solution, we can solve the inequality using a GDC. We are looking for the
smallest value, so we will use Math/Solver.

This process is much quicker than solving algebraically.

6
21 We will use the change of base formula:
log5 5 1 25
⇒ 9 (log5 x ) = 25 ⇒ (log5 x ) =
2 2
9 log5 x = 25 ⇒ 9 log5 x = 25
log5 x log5 x 9
Hence, the solutions are:
5
5 −
log5 x = − ⇒ x = 5 3
3
5
5
log5 x = ⇒ x = 53
3
22 From ln ( x + 3) = 1 we can conclude that either ln ( x + 3) = −1, or ln ( x + 3) = 1. Hence,
1
ln ( x + 3) = −1 ⇒ x + 3 = e −1 ⇒ x = e −1 − 3 = − 3
e
ln ( x + 3) = 1 ⇒ x + 3 = e1 ⇒ x = e − 3

23 Since the equation cannot be solved algebraically, we will use a GDC.

So, the solutions are: −2.50, −1.51, 0.440 .


1
Note: We can transform the equation and reduce the task by solving two equations graphically: e 2 x − = −2
x+2
1
and e 2 x − = 2.
x+2 t
24 Since the number of bacteria double every 20 minutes, the exponential growth model is: n = 650 (2)20 . So, we have
t 1 1 1 ln 2
to solve: 650 (2)20 = 650e kt . Since the equation holds for all t, then (2)20 = e k ⇒ ln (2)20 = k ⇒ k = ln 2 = .
20 20
25 a Using the property of the sum and difference of logarithms, we have:
x ( x − 2)
f ( x ) = ln x + ln( x − 2) − ln( x 2 − 4 ) = ln ( x ( x − 2)) − ln ( x 2 − 4) = ln 2 .
(x − 4)
x ( x − 2) x
Applying the formula for the difference of squares, we have: ln = ln .
( x − 2)( x + 2) x+2
Note: We can apply the formula for the difference of squares immediately and then apply the formula for the
logarithm of a product:
f ( x ) = ln x + ln( x − 2) − ln (( x − 2)( x + 2)) = ln x + ln( x − 2) − ln ( x − 2) − ln ( x + 2) = ln x − ln( x + 2) . Finally, we apply
x
the formula for the difference of logarithms: f ( x ) = ln x − ln( x + 2) = ln .
x+2
y
b Switching the variables, x = ln , we have:
y+2
y −2 e x −2 e x
ex = ⇒ e x ( y + 2) = y ⇒ y e x − y = −2e x ⇒ y (e x − 1) = −2e x ⇒ y = x . Hence, f −1( x ) = x .
y+2 e −1 e −1
Note: If we transfer the y-values to the other side, or if we multiply both the numerator and denominator
2e x
in the result by 21, the formula will be: f −1( x ) = .
1− ex

7
Chapter 5

26 a i

From the data, we can conclude that the minimum value of f ( x ) is 0.

ii From part i, we can conclude that f ( x )  0 for all x; hence, e x − 1 − x  0 ⇒ e x  1 + x for all x.
 1  1  1
b Let P(n) be the proposition: (1 + 1)  1 +   1 +    1 +  = n + 1.
 2   3  n 
Basis step:
P(1) is true since (1 + 1) = 2 and, for n = 1, n + 1 = 2.

Inductive step:
 1  1  1
Assume P(k) is true for some integer k  1, that is: (1 + 1)  1 +   1 +    1 +  = k + 1.
 2   3  k 
1 1 1 1  1  k + 1+ 1
Then: (1 + 1)  1 +   1 +    1 +   1 + 
= (k + 1) 1 + = (k + 1) = (k + 1) + 1
 2   3  k   k + 1  k + 1 k +1
Thus, using the fact that P(k) is true, we establish that P(k + 1) is true, and so P(n) is true for all integers n  1.
1 1 1 1 1 1
1+ + + ... +
c First transform: e 2 3 n
= e1e 2 e 3  e n

Now we will apply a ii:


1 1 1 1 1 1
1+ + + ... +  1  1  1
e 2 3
= e1e 2 e 3  e n  (1 + 1)  1 +   1 +
n
  1 +  = n + 1 > n
 2  3  n 
And then b:
1 1 1 1 1 1
1+ + + ... +  1  1  1
e 2 3 n = e1e 2 e 3  e n  (1 + 1)  1 +   1 +   1 +  = n + 1 > n
 2  3  n 
1 1 1
1 1 1 1+ + + ... +
d 1+ + + ... + > 100 ⇒ e 2 3 n > e100
2 3 n
Using the result from part c, we can see that any integer greater than e100 is a solution.

8
Chapter 6
Practice questions

2x 3
1 det( A) = = 2x ⋅ x − 3( −4 x ) = 2x 2 + 12x
−4 x x
det( A) = 14 ⇒ 2x 2 + 12x = 14 ⇒ x 2 + 6 x − 7 = 0 ⇒ x1 = −7, x 2 = 1
So, x = −7 or x = 1.
2
 a 2   a 2   a 2   a 2 + 4 2a − 2 
2 a M2 =  = =

 2 −1   2 −1   2 −1   2a − 2 5 

 5 −4   a 2 + 4 2 a − 2   5 −4  a 2 + 4 = 5
b M2 =  ⇒  2a − 2 = ⇒ 
 −4 5   5   −4 5  2a − 2 = −4
The solution a = −1 satisfies both equations.
 −1 2 
For a = 21, we have: 
 2 −1 
1  −1 −2  1  −1 −2 
M −1 =   =− 
det( m )  −2 −1  3  −2 −1 
The system of equations can be written as:
 x   −3   x  −3   x 1  −1 −2   −3  1  −3   1 
M  =  ⇒   = M −1   ⇒  =−     =−  = .
y
   3   y  3  y
  3  −2 −1   3  3  3   −1 
Therefore, the solution is x = 1, y = −1.
 11 2   5 2
3 BA =   and A = 
 44 8   2 0 
1  0 −2 
A−1 = − 
4  −2 5 
 11 2  −1 1  11 2   0 −2  1  −4 −12   1 3 
B=  A =−  =−  =
 44 8    
4  44 8   −2 5  4  −16 −48   4 12 
 3 1 a b  a b  4 8 
4 AX + X = B ⇒  + =
 −5 6   c d   c d   0 −3 
 3a + c 3b + d   a b   4 8   4a + c 4b + d   4 8 
⇒ + = ⇒ =
 −5a + 6c −5b + 6d   c d   0 −3   −5a + 7c −5b + 7d   0 −3 
By comparing the corresponding elements, we obtain two systems of linear equations:
4a + c = 4 4b + d = 8
−5a + 7c = 0 − 5b + 7d = −3
28 20 59 28 28, b = ___
59, c = ___
20, and d = ___
28.
a= ,c = b= ,d = So, a = ___
33 33 33 33 33 33 33 33

 5 −2 
5 A=
 7 1   1 2 
1  1 2 1  1 2  19 19 
a A−1 =  = = 

5 + 14  −7 5  19  −7 5   −7 5 
 19 19 

1
Chapter 6

b i X A + B = C ⇒ X A = C − B ⇒ X = ( C − B ) A−1
 1 2 
  −5 0   6 7    19 19  1  −11 −7   1 2  1  38 −57   2 −3 
ii X =   −     =     = =
 − 8 7   5 − 2   − 7 5  19  −13 9  −7 5  19  −76 19   −4 1 
 19 19 

 a b   1 2   a +1 b + 2   a b   1 2   a + bd 2a + bc 
6 a A+B =  + = b AB =  =
 c 1   d c   c + d c + 1   c 1   d c   c + d 3c 

7 a Using a GDC:

b The system of equations can be written as:


 x   1  x   1
A y = 2 ⇒ y
     = A−1  2 
       
 z   3   z   3
For A as above and we have:

6 3 8
The solution is: x = , y = ,z = .
5 5 5
 −2 4   5 2
8 Given C =  ,D =  , and 3Q = 2C − D , we have:
 1 7   −1 a 
 −3 2 
1     1  
(2C − D ) = 2  −2 4  −  5 2   =  −9 6  = 
1 
a Q= 14 − a
3 3   1 7   −1 a   3  3 14 − a   1 
 3 
 −2 4   5 2   −10 − 4 −4 + 4 a   −14 4 a − 4 
b CD =  = =
 1 7   −1 a   5 − 7 2 + 7a   −2 7a + 2 

 5 2 1  a −2 
c D −1 =  =
 −1 a  5a + 2  1 5 
9 a If A is the inverse of matrix B, then AB = BA = I must be satisfied. So, we have:
 a −4 −6   1 2 −2   a − 6 2a − 4b − 6 −2a + 14   1 0 0
AB =  −8 5 7   3 b 1  =  0 5b − 9 0  = 0 1 0
      
 −5 3 4   −1 1 −3   0 3b − 6 1   0 0 1
 1 2 −2   a −4 −6   a−6 0 0   1 0 0
BA =  3 b 1   −8 5 7  =  3a − 8b − 5 5b − 9 7b − 14  = 0 1 0
      
 −1 1 −3   −5 3 4   − a + 7 0 1   0 0 1
By comparing the corresponding elements, we can see that the solution a = 7, b = 2 satisfies all the equations.

2
b The system can be written as:
−1
 x  5  1 2 −2   x   5   x   1 2 −2   5 
B y  = 0 ⇒ 3 2 1   y  = 0  ⇒ y  = 3 2 1   0
            
 z   6   −1 1 −3   z   6   z   −1 1 −3   6 
Since AB = BA = I ⇒ B −1 = A , then:
−1
 x   1 2 −2   5   7 −4 −6   5   −1 
 y =    =   = 2 
   3 2 1   0   −8 5 7   0   
 z   −1 1 −3   6   −5 3 4   6   −1 

The solution is: x = 21, y = 2, z = 21.


10 a AB = C ⇒ B = A−1C .
b i For:

ii Matrix B = A−1C .

c The coordinates of the point of intersection of the planes are given as the solution of the system of equations that can
be written as:
 x  x
A  y  = C ⇒  y  = A−1C = B
   
 z   z 
The point has coordinates (1, 21, 2).

1 1 2
11 a det( A) = 1 2 1 = 110
( − 1) − 1( 5 − 2) + 2(1 − 4 ) = 0
2 1 5
b We transform the augmented matrix of the system:
 1 1 2 3
 1 2 1 4  R2 − R1
  R − 2R1
 2 1 5 λ  3
 1 1 2 3 
 0 1 −1 1  {R3 + R2
 
 0 −1 1 λ − 6 
 1 1 2 3 
 0 1 −1 1 
 
 0 0 0 λ−5

The general solution of the system exists if λ − 5 = 0 ⇒ λ = 5.

3
Chapter 6

For λ = 5 we have:
 1 1 2 3
 0 1 −1 1  {R1 − R2
 
 0 0 0 0
 1 0 3 2
 0 1 −1 1 
 
 0 0 0 0
For z = t ⇒ x = 2 − 3t , y = 1 + t .

12 If X 3 = 0 we have:
( I − X )( I + X + X 2 ) = II + I X + I X 2 − X I − XX − XX 2 = I + X + X 2 − X − X 2 − X 3 = I − X 3 = I − 0 = I
( I + X + X 2 )( I − X ) = II − I X + X I − XX + X 2I − X 2 X = I − X + X − X 2 + X 2 − X 3 = I − X 3 = I − 0 = I
We can conclude: ( I − X )−1 = ( I + X + X 2 ) and ( I + X + X 2 )−1 = ( I − X ).

4
Chapter 7
Practice questions

Solution Paper 1 type

1 a The length is a function of time, so we are looking for the value of the function when t = 2:

L( 2) = 110 + 25 cos (2π ⋅ 2) = 110 + 25 cos ( 4 π ) = 110 + 25 ⋅ 1 = 135 cm

b We are looking for the minimum value of a function of the form y = a cos [b( x + c )] + d . Since the smallest value of
cosine is −1, the minimum value of L will be:

Lmin = 110 + 25 ⋅ ( −1) = 85 cm

Note: We have found the minimum value of the function using the fact that the minimum value of cosine is 21. We
could have found the result differently: The minimum value of a function of the form y = a cos [b( x + c )] + d is equal
to the vertical shift d minus the amplitude | a |; so we have: min = d − | a | = 110 − 25 = 85.

c We have to find the least value of t such that L = 85. From part b we can see that the lowest value will be obtained
when the cosine equals 21; hence, 2πt = π ⇒ t = __ 1.
2
Note: We could find t by solving the equation: 110 + 25 cos (2πt ) = 85 ⇒ 25 cos (2πt ) = 85 − 110 ⇒ cos (2πt ) = −1.
1
2πt = π ⇒ t = sec
2
d We have to determine the period of a function of the form y = a cos [b( x + c )] + d . So, the period is
2π 2π
= = 1 sec.
b 2π

Solution Paper 2 type

1 We will observe the function L = 110 + 25 cos (2πt ).


Firstly, we have to select radian measure and then input the
function.

a We have to find L(2):

b–c For the remaining parts of the question, we will use the graph of the function. So, we have to set a suitable window.

The minimum value is the vertical shift minus the amplitude, and the maximum value is the vertical shift plus the
amplitude. Therefore, we can use y-values from (less than) 110 2 25 = 85 to (greater than) 110 + 25 = 135.

1
Chapter 7

Another way would be to use the ZoomFit feature. The window will not be suitable for all the calculations, but we can
read out the range and then just extend it a bit in both directions.

We can solve both parts b and c by finding the minimum value with the smallest x.

d We can find the period of a trigonometric function by finding two successive


minimum points, or two successive maximum points, and then subtracting their
x-coordinates. We already know the smallest positive minimum has x = 0.5, so we
have to find the next positive minimum.

Hence, we can conclude that the period is 1.5 2 0.5 = 1.

2 2 sin2 x − cos x + 1 = 0
2 (1 − cos2 x ) − cos x + 1 = 0 Use the Pythagorean identity sin 2 x = 1 − cos 2 x .
−2 cos2 x − cos x + 3 = 0
−2t 2 − t + 3 = 0 ⇒ t1 = −1.5, t2 = 1 Substitute cos x = t .
cos x = 1.5 ⇒ has no solution
cos x = 1 ⇒ x = 0, 2π
Solutions are: 0, 2π.

3 The perimeter of the shaded sector contains two radii and an arc. So, the length of the arc can
be calculated: 25 = 2 ⋅ 6 + s ⇒ s = 13. We can use the arc length formula to find the angle θ1 in
radians:
1 13
s = r θ1 ⇒ 13 = 6 ⋅ θ1 ⇒ θ1 =
6
For angles θ and θ1 :
13
θ + θ1 = 2π ⇒ θ = 2π − ≈ 4.12
6

2
4 a i The amplitude of function f is 1, so the minimum value of the function is 21.

ii Period of g = = 4 π.
1
2

b It is quickest to find the


number of solutions from
the graph.

We can see that there are four points of intersection; hence, there are four solutions to the equation.
 2π 
5 For d = p + q cos  t :
 m 
a To find p we have to determine the mid-line; therefore, we have to find the average of the function’s maximum and
minimum value:
64 + 6
p= = 35
2
b To determine q, we have to determine the amplitude. The amplitude is the difference between the function’s
maximum value and the mid-line: | q | = 64 − 35 = 29. So, q is 29 or −29 . From the given data, we can establish that
the graph starts from the maximum value, so q is positive; hence, q = 29.

c From the data, we can see that the distance between two successive maximum points (or minimum points) is
0.5 seconds. So, the period is 0.5.

Using the formula for period, we have: period = = m. Hence, m = 0.5.

m
Note: It is useful to highlight the basic data using a rough sketch.

60 60

40 40

20 20

1 1

-20 -20

Solutions are: 0, 1.06, 2.05.

Note: The question does not tell us to use a specific method to solve the equation, so we can choose any method. The
most suitable method is graphical, because we can see the number of solutions and find them all.

3
Chapter 7

7 a Use the substitution cos x = t : y


2
1
2t 2 + 5t + 2 = 0 ⇒ t1 = −2, t2 = − 3
2
cos x = −2 has no solutions 1
-
x
1 2π 4 π 2
cos x = − ⇒ x = , 4
2 3 3
3
2π 4 π
Solutions are: , .
3 3
b Use double angle identity for sine:

2 sin x cos x − cos x = 0 y

Factorize:  π 3π 5π ππ
 cos x = 0 ⇒ x = ,
 2 2 6
cos x (2 sin x − 1) = 0 ⇒  6
1
 2 sin x − 1 = 0 ⇒ sin x = ⇒ x = , π 5 π x
 2 6 6
π π 5π 3π
Solutions are: , , , .
6 2 6 2
π
8 Given < x < π , it follows that sin x > 0 and cos x < 0 .
2
8 1
a Using the Pythagorean identity for sine, we have: sin2 x = 1 − cos2 x = 1 − = .
9 9
1 1
Using the fact that sin x > 0, we have: sin x = = .
9 3
8 16 7
b Using the double angle identity for cosine, we have: cos 2x = 2 cos2 x − 1 = 2 ⋅ − 1 = −1= .
9 9 9
8 8 2 2
c Using cos x < 0, we have: cos x = − =− =− . Hence,
9 3 3
1  2 2 4 2
sin 2x = 2 sin x cos x = 2 ⋅ ⋅  − =− .
3  3  9
9 We have to interpret the data as points on a graph: (5, 5.8)
6

High (maximum tide) at 5:00 with depth of 5.8 m → ( 5, 5.8 )

Low (minimum) tide at 10:30 with depth 2.6 m → (10.5, 2.6 ) 4

a The function can be written in the form: 2 (10.5, 2.6)


d = A sin (B ( x + C )) + D (or similar with the cosine
function). Firstly, we will determine D, so we have to find
5 10
the average of the function’s maximum and minimum value:
5.8 + 2.6
D= = 4.2
2
(5, 5.8)
Next we will determine A. The amplitude of the 6

function is the difference between the function’s


4
maximum value and the mid-line:
| A | = 5.8 − 4.2 = 1.6. So, A is 1.6 or −1.6. (-0.5, 2.6)
2 (10.5, 2.6)
From the data, we can see that the earlier minimum
is reached at (−0.5, 2.6) , so the graph after the
5 10

mid-line first reaches the maximum value, then the


minimum value; therefore, A is positive, A = 1.6.

4
We can also see from the data that the distance between successive maximum and minimum points is 5.5 hours. So,
half of the period = 5.5.
2π 2π 2π
Using the formula for period, we have: period = ⇒ 11 = ⇒B= .
B B 11
To determine the horizontal translation (phase shift), we
6
(5, 5.8)
have to find the points of the graph on the mid-line.
1 (7.75, 4.2)
Abscissae of those points are on a distance of a of the 4
4 (2.25, 4.2)
period from the abscissa of the maximum point. Hence,
2 (10.5, 2.6)
1 9 31
5 ± ⋅ 11 , so and .
4 4 4
9 5 10
Thus, C = − and the function is:
4
 2π  9
d = 1.6 sin  x −   + 4.2.
 11  4
b We have to find the value of the function at 12 noon, that is, 12 hours after midnight; therefore,
 2π  9
d(12) = 1.6 sin  12 −   + 4.2 ≈ 3.15 metres.
 11  4

c We have to find the time interval in which the value of d is greater than 3.5.

We have to convert from decimal numbers to minutes:

Therefore, from about 12:27 pm to 7:33 pm the boat can dock safely.

We can also see that around midnight the boat can dock safely again.

5
Chapter 7

10 Use the substitution tan x = t :

t 2 + 2t − 3 = 0 ⇒ t1 = 1, t2 = −3
π
tan x = 1 ⇒ x =
4
π
tan x = −3 cannot be solved exactly Solutions are: , 1.89.
4
3
11 a We will use the arc length formula: s = r θ = 10 ⋅ = 15 cm
2
3
b The angle in the shaded region is 2π − ; therefore, using the area of a sector
2
formula:
1 1  3
A = r 2 θ = ⋅ 102  2π −  ≈ 239 cm2
2 2  2

5 π 5 5
12 The function f ( x ) = cos  2x −  reaches a maximum value of and a minimum value of − . That means that
2  2 2 2
5 5 5 5 5 5
−  f ( x )  . The function will not reach values greater than or less than − . Hence, for k > and k < − ,
2 2 2 2 2 2
the equation will have no solutions.

13 From the graph, we can see that the point (0, 1) is on the mid-line, so k = 1.
To determine a, we need to find the amplitude. The amplitude is the difference between the function’s maximum value
and the mid-line: | a | = 3 − 1 = 2; so, a is 2 or −2. We can see that the graph, starting from initial position at x = 0, first
reaches the minimum value, and then the maximum value, so a is negative; hence, a = –2.
sin2 α 1 − cos2 α 1
14 We can write tan2 a using secant only: tan2 α = = = − 1 = sec 2 α − 1
cos2 α cos2 α cos2 α
So, the equation is: 2 sec 2 α − 2 − 5 sec α − 10 = 0 ⇒ 2 sec 2 α − 5 sec α − 12 = 0
3
Using the substitution sec α = t , we have: 2t 2 − 5t − 12 = 0 ⇒ t1 = − , t2 = 4 . Since α is in the second quadrant,
2
3
secant should be negative and the solution is sec α = − .
2
15 a Using the compound angle formula for sine, we have: sin( α + β ) = sin α cos β + cos α sin β
15 8
From the upper triangle, we can see that: sin α = , cos α =
17 17
6 8
From the lower triangle, we can see that: sin β = , cos β =
10 10
15 8 8 6 168 84
So, sin( α + β ) = ⋅ + ⋅ = = .
17 10 17 10 170 85
8 8 15 6 −26 13
b cos( α + β ) = cos α cos β − sin α sin β = − = =−
17 10 17 10 1700 85
84
sin (α + β ) 84
c tan( α + β ) = = 85 = −
cos (α + β) − 13 13
85
Note: We can find tan α and tan β from the drawing, and then use the compound formula for tangent.
1 1 2 2
16 From the diagram, we can see that: sin p = = and cos p = = .
1 +2
2 2
5 1 +2
2 2
5
1 2 4
Hence: sin 2 p = 2 sin p sin p = 2 ⋅⋅ = .
5 5 5
For sin 3p, we can use the compound angle formula: sin 3p = sin (2 p + p ) = sin 2 p cos p + cos 2 p sin p

6
4 1 3
So, we have to find cos 2 p = cos2 p − sin2 p = − = . Finally, we have:
5 5 5
4 2 3 1 11 11 5
sin 3p = + = = .
5 5 5 5 5 5 25
17 If B is obtuse, then the sine is positive and cosine negative.
5 5
a sin B = =
52 + 122 13
12 12
b cos B = − =−
5 + 12
2 2 13
5  12  120
c sin 2B = 2 sin B cos B = 2 −  = −
13  13  169
2 2
 12   5 119
d cos 2B = cos2 B − sin2 B =  −  −   =
 13   13 169
Note: In the solution, we have used the property of angles and sides in a right triangle.
We can find the solution by using the Pythagorean identity for sine and cosine:
sin x 5 5
=− ⇒ sin x = − cos x
cos x 12 12
2
 5  169 144 12
sin2 x + cos2 x = 1 ⇒  − cos x  + cos2 x = 1 ⇒ cos2 x = 1 ⇒ cos x = − =−
 12  144 169 13
5  12  5
Finally, sin x = −  −  = . Now we can continue using the double angle formulae.
12 13 13
3 2 tan θ 3
18 Using the double angle formula for tangent, we have: tan 2θ = ⇒ = ⇒ 8 tan θ = 3 − 3 tan2 θ
4 1 − tan2 θ 4
Using the substitution tan θ = t, we will have a quadratic equation:
1 1
3t 2 + 8t − 3 = 0 ⇒ t1 = −3, t2 = . Hence, the possible values of tan θ are: −3, .
3 3
19 We will use the compound angle formula for sine:
sin x cos α − cos x sin α = k (sin x cos α + cos x sin α )
Then we will divide both sides by cos x cos α :
sin x cos α cos x sin α  sin x cos α cos x sin α 
− =k + ⇒
cos x cos α cos x cos α  cos x cos α cos x cos α 
tan x − tan α = k ( tan x + tan α ) ⇒ tan x − k tan x = k tan α + tan α ⇒ tan x (1 − k ) = tan α (k + 1) ⇒
tan α( k + 1)
tan x =
1− k
−( k + 1)
Note: If we rearrange the equation differently, we obtain the result tan x = tan α , which is equivalent to the
k −1
above result.

20 We can see that either tan 2θ = 1 or tan 2θ = −1.


π π π π π π 3π
So, tan 2θ = 1 ⇒ 2θ = + k π ⇒ θ = + k ⇒ θ = , + = −
4 8 2 8 8 2 8
π π π π π π 3π
tan 2θ = −1 ⇒ 2θ = − + k π ⇒ θ = − + k ⇒ θ = − , − + =
4 8 2 8 8 2 8
π 3π
Hence, the solutions are: ± , ± .
8 8
21 a We have to set the
window on the GDC to
be the same as on the grid.

7
Chapter 7

The domain of the functions is −1  x  1 , so we just have to draw this part of the graph.

b No solutions

c Cosine function has a range from −1. But, here, its domain is restricted. Since cosine is an even function, it is enough to
observe its behaviour on the interval from 0 to 1. Hence, the function is decreasing on this interval; its largest value is
cos 0 = 1 , and the smallest value cos 2. Hence, the range is the set [cos 2, 1] .
ˆ = θ and AC = x.
22 Let CAD
2 2 5 5
From the drawing, we can see that: tan θ = ⇒ x = and tan 2θ = ⇒ x = .
x tan θ x tan 2θ
2 5
Hence: =
tan θ tan 2θ
2 5 5(1 − tan2 θ ) 1
Using the double angle formula for tangent, we have: = ⇒2= ⇒ 4 = 5 − 5 tan2 θ ⇒ tan2 θ =
tan θ 2 tan θ 2 5
1 − tan2 θ
Since θ is an angle in a right triangle, its tangent has to be positive.
1
So, tan θ = ⇒ θ ≈ 24.1.
5
2 5
Note: We can solve the equation = graphically.
tan θ tan 2θ
We are looking for the angle when those two sides are the same.

 πx 
23 We have to solve the equation 16 sec   − 32 = −16 + 10 . Hence,
 36 
 π x  26 13 1 13  πx  8
sec   = = ⇒ = ⇒ cos   =
 36  16 π x  
cos  
8 8 36 13
 36 
We need the first positive solution, so:
πx  8 36  8
⇒ = arccos   ⇒ x = arccos  
36  13  π  13
36  8
Hence, the solution of the equation is: arccos   , and the width of the water surface in the channel is:
π  13
36  8  72  8
2⋅ arccos   = arccos   cm.
π  13 π  13

8
Chapter 8
Practice questions

1 The shortest distance from AB to O is the perpendicular from O to AB, so OC = 3 units, and B
A C
CB = 52 − 32 = 4 units. Therefore, AB = 2 ⋅ 4 = 8 units.
3
5 5
We are asked to find the exact value of the sine of the angle, which means that we have to
O 5
avoid all calculator use!
Method I:

In triangle AOB all sides are given, so we can find angle AOB using the law of cosines:
 52 + 52 − 82 7
cos AOB = =−
2⋅5⋅5 25
Using the Pythagorean identity for sine and cosine, and taking into account the fact that the sine of an angle in a triangle
is always positive, we have:
 7
2
252 − 72 18 ⋅ 32 9 ⋅ 2 ⋅ 2 ⋅ 16 3 ⋅ 2 ⋅ 4 24
sin AOB = 1 −  −  = = = = =
 25 252 25 25 25 25
Method II:
  3
From the right triangle OBC, we can find acute angle OBC : sin OBC =
  5  3
sin AOB sin OBC  8 ⋅ sin OBC 8 5 24
Using the law of sines in triangle AOB, we have: = ⇒ sin AOB = = =
8 5 5 5 25
Method III:

From the right triangle OCB, we can find the sine and cosine of the acute angle OBC :
 4  3
sin OBC = and cos OBC =
5 5
 
The angle AOB = 2 ⋅ OBC , so, using the double angle identity for sine, we have:
   3 4 24
sin AOB = 2 sin OBC ⋅ cos OBC = 2 ⋅ ⋅ =
5 5 25
3
2 In the given right triangle, tan θ = . Therefore, the triangle has legs of 3 and 7 and hypotenuse of 32 + 72 = 58 .
7
3 7
Hence, sin θ = and cos θ = .
58 58
Using double angle identities, we have: 58
3 7 42 21 3
sin 2θ = 2 sin θ cos θ = 2 ⋅ ⋅ = =
58 58 58 29
49 9 20
cos 2θ = cos2 θ − sin2 θ = − = 7
58 58 29
3 In the triangle SSS is given, so we can find any angle using the law of cosines. The largest angle is opposite the longest
side; therefore, we are looking for the angle opposite the side of size 7.
4 2 + 52 − 72 1 1
cos θ = = − ⇒ θ = cos−1  −  ≈ 101.5
2⋅4 ⋅5 5  5
4 If A is obtuse, then cos A is negative. Therefore, using the Pythagorean identity for sine and cosine, we have:
25 12
cos A = − 1 − sin2 A = − 1 − = − . Hence, using the double angle identity for sine, we have:
169 13
5  12  120
sin 2 A = 2 sin A cos A = 2 −  = − .
13  13  169

1
Chapter 8

PQ
5 a From the right triangle BQP, we have: tan 36 = ⇒ PQ = 40 tan 36 ≈ 29.1 m
40

b In triangle ABQ, the angle AQB = 180 − 70 − 30 = 80. Using the law of sines, we can find side AB :
AB 40 40 sin 80

= 
⇒ AB = ≈ 41.9 m
sin 80 sin 70 sin 70

6 In triangle ABC, SSS is given; hence, we can find angle CAB using the law of cosines:
 482 + 322 − 562 
cos CAB = ⇒ CAB ≈ 86.4 
2 ⋅ 48 ⋅ 32

7 a In the triangle SSS is given, so we can find any angle using the law of cosines. The smallest angle is opposite the
shortest side; therefore, we are looking for the angle opposite the side of size 5.
72 + 82 − 52 11
cos θ = ⇒ θ = cos−1   ≈ 38.2
2⋅7 ⋅8  14 
1
b Using the area formula with sides 7 and 8 and the included angle from a: A = ⋅ 7 ⋅ 8 sin θ ≈ 17.3 cm2
2

8 a In Triangle 2, ABC, SSA is given.



We can use the law of sines to find angle ACB :
 A
sin ACB sin 50  20 ⋅ sin 50
= ⇒ sin ACB = ≈ 0.901 2287...
20 17 17
  20 cm
20cm 17cm
17 cm
In Triangle 2, angle ACB is obtuse, so ACB = 180 − sin−1 0.901 2288 ≈ 116.
50°
B C

   
b In Triangle 1, angle ACB is acute, so ACB = sin−1 0.901 2288 ≈ 64.3 ; therefore, angle BAC = 180 − 50 − BCA ≈ 65.68 .
Hence, the area is:
1 
A = 20 ⋅ 17 ⋅ sin BAC ≈ 155 cm2.
2

2
9 In 2.5 hours, boat A covers a distance of 20 ⋅ 2.5 = 50 km, and boat B covers a distance of 32 ⋅ 2.5 = 80 km.
A
In triangle PAB, SAS is given, so we can find side AB using the law of cosines:
50 km
AB = 502 + 802 − 2 ⋅ 50 ⋅ 80 cos 70 ≈ 78.5 km
70°
Therefore, after 2.5 hours, the boats are approximately 78.5 km apart. B
P 80 km

10 In triangle JKL, SSA is given. The angle K JL is the angle opposite the longer side, so there will be a unique triangle with the

given dimensions. We can determine angle JKL using the law of sines:

sin JKL sin 51  25 sin 51 
= ⇒ sin JKL = ≈ 0.51128 ⇒ JKL ≈ 31
25 38 38


11 a In triangle ABC, ASA is given, so we can find AB using the law of sines. Angle A = 180 − 60 − 40 = 80.
AB 5 5 sin 40

= 
⇒ AB = ≈ 3.26 cm
sin 40 sin 80 sin 80
1
b Area = 5 ⋅ AB sin 60 ≈ 7.07 cm2
2

12 B
B
a 2
A
a A
3
O 2
a
3
2
O

In triangle OAB, sides OA and OB are each half the length of the space diagonals, and side AB is the side diagonal of the
cube.
AB = a 2
1
( ) a
2
OA = OB = a 2 + a2 = 3
2 2
In triangle OAB, SSS is given. We can use the law of cosines to find angle θ :
2 2


2
 
2
 (
 a 3 +  a 3 − a 2 2 3a2 3a2
4
)
+
4
− 2a 2 1
cos θ = = = − ⇒ θ ≈ 109.47
2  a  a 
3 3
3a2 3
 2   2  2
4
The acute angle between the diagonals is 180 − θ ≈ 70.5 .

3
Chapter 8

13 a BC = 104 2 + 652 − 2 ⋅ 104 ⋅ 65 cos 60 = 10 816 + 4225 − 6760 = 8281 = 91 m


1 1 3
b Area = 65 ⋅ 104 sin 60 = 65 ⋅ 104 ⋅ = 1690 3
2 2 2
1 1 1 65x
c i Area of A1 = 65 ⋅ x sin 30 = 65 ⋅ x ⋅ =
2 2 2 4
1 1 1
ii Area of A2 = 104 ⋅ x sin 30 = 104 ⋅ x ⋅ = 26 x
2 2 2
iii From b we know that the area is A = 1690 3 . Since A = A1 + A2, we have:
65x 169 x
1690 3 = + 26 x ⇒ 1690 3 = ⇒ 40 3 = x.
4 4
   
d i The angles ADC + ADB = 180 ; hence, sin ADC = sin ADB.

ii Method I:
BD 65 65 sin 30
We will use the sine rule in triangle ADB: 
=  ⇒ BD = 
sin 30 sin ADB sin ADB
DC 104 104 sin 30 104 sin 30
Next, we will use the sine rule in triangle ADC: 
=  ⇒ DC =  = 
sin 30 sin ADC sin ADC sin ADB
Therefore:
65 sin 30

BD 65 5
= sin ADB  = =
DC 104 sin 30 104 8

sin ADB

Method II:
65x  
We can find areas A1 = and A2 = 26 x using the sides BD and DC and angles ADC and ADB :
4
65x 1 
A x BD sin ADB BD
4 = 1 = 2 =
26 x A2 1 
x DC sin ADC DC
2
65x
5 BD 5
Since 4 = , we have established that = .
26 x 8 DC 8
14 a Using the law of sines, we have:
x
=
x−2
( )
⇒ 2x = 2x − 4 ⇒ x 2 − 2 = 4 ⇒ x =
4
=
(
4 2+ 2 ) = 4+2 2
sin 45 
sin 30 
2− 2 4−2
1
b A = x ( x − 2) sin 105. So, we have to determine the exact value of sin105 . Using the compound formula for sine,
2
3 2 1 2 6+ 2
we have: sin 105 = sin (60 + 45 ) = sin 60 cos 45 + cos 60 sin 45 = + =
2 2 2 2 4

(
1
)
Now we have: A = 4 + 2 2 ( 2 + 2 2 )
6+ 2
=
( )(
2 + 2 1+ 2 )( 6+ 2 )
2 4 2
(
A=
)(
4+3 2 6+ 2 ) =
4 6+4 2+6 3+6
=2 6 +2 2+3 3+3
2 2
1
15 T1 = CD ⋅ BD sin CDB
2
1
T2 = CD ⋅ AD sin CDA
2
   
Since CDA = 180 − CDB , their sines are the same: sin CDA = sin CDB .
1
CD ⋅ BD sin CDB
T1 2 BD
Hence: = = .
T2 1 AD
CD ⋅ AD sin CDA
2

4
16 a 60 + θ < 180 ⇒ θ < 120 ; hence, 0 < θ < 120.
1
b A = 30 ⋅ KJ sin θ ; we have to determine KJ. Using the law of sines:
2
KJ 30 30
= 
⇒ KJ = sin L
sin L sin 60 3
2
Since L = 180 − ( 60 + θ ) ⇒ sin L = sin( 60 + θ ), we have:
30 ⋅ 2
KJ = sin (60 + θ) = 20 3 sin (60 + θ). Finally, the area is:
3
1 1
A= 30 ⋅ KJ sin θ = 30 ⋅ 20 3 sin (60 + θ) sin θ = 300 3 sin θ sin (60 + θ).
2 2
c

Hence, 60 gives the maximum area for the enclosure.

1
17 a A= 30 ⋅ 172 − 152 = 120 cm2
2
 2 ⋅ 172 − 302 
b cos ABC = = −0.557 09... ⇒ ABC ≈ 2.16
2 ⋅ 172
c (
R = AA1 − AA2 − AT )
1
AA1 = 152 π ≈ 353.43 cm2
2
1  1
AA2 = r 2 ⋅ ABC ≈ 172 ⋅ 2.16 = 312.12 cm2
2 2
R = AA1 − AA2 + AT ≈ 161 cm2

18 a Using the law of cosines, we have:

L2 = 12 + 12 − 2 ⋅ 1⋅ 1⋅ cos α = 2 − 2 cos α ⇒ L = 2 − 2 cos α


α α α α 1 − cos α
b cos 2   = 1 − 2 sin2   ⇒ 2 sin2   = 1 − cos α ⇒ sin   =
 2  2  2  2 2
 1 − cos α  1 − cos α  α
c L = 2 − 2 cos α = 2 (1 − cos α ) = 4  = 2 = 2 sin  
 2 2  2
19 Using the law of sines, we have:
a b
=
sin θ sin 2θ
Applying the double angle formula for sine, we have:
a b b b
= ⇒a= ⇒ cos θ =
sin θ 2 sin θ cos θ 2 cos θ 2a

5
Chapter 9
Practice questions

  


1 a DC = DM + MC = −u + v = v − u
 1  1  1
b AM = AB = DC = ( v − u)
2 2 2
    1 1
c BC = BM + MC = − AM + v = − ( v − u) + v = (u + v )
   2 2
1 3 1
d AC = AB + BC = ( v − u) + (u + v ) = v − u
2 2 2
2 a w = 2u + v = 2(i − 2 j) + 4i + 3 j = 6i − j = ( 6, −1)
6 6 6 36 6 6
b z= w= ( 6i − j) = ( 6i − j) = i− j= (66, −1)
w 6 + ( −1)
2 2
37 37 37 37

3 a OR = 102 + ( 5 5 )2 = 15

Point R lies on the circle because the distance from O to R is 15 (the radius of the circle).
    10   15   −5 
b AR = OR − OA =   −  = 
 5 5  0   5 5
 
AO ⋅ AR −15 × ( −5) + 0 × 5 5 75 1
c cos ∠OAR =   = = =
AO ⋅ AR 155 ( −5) + ( 5 5 )
2 2 15 150 6
2
 1  5
sin ∠OAR = 1 − (cos ∠OAR ) = 1 − 
2
d =
 6  6
1   1 5
Area MAR = AM ⋅ AR sin ∠OAR = ⋅ 30 ⋅ 150 ⋅ = 75 5
2 2 6

4
y
C(3,8)
8

T
6

v
S
4

R(11,4)
3

2 u
A(6,2)
1
P
O

-2 2 4 6 8 10 12
M(0 ,0) x

-1

-2

1
Chapter 9

  11   0   11    3   6   −3 
a MR =   −   =   , AC =   −   = 
 4   0  4   8   2   6 
 
  MR ⋅ AC 11 × ( −3) + 4 × 6 9  
b cos  (MR , AC ) =   = = ⇒  (MR , AC ) = 83.4 
MR ⋅ AC 11 + 4 ⋅ ( −3) + 6
2 2 2 2
137 ⋅ 45

 17   3
c The midpoints are P ( 3, 1), Q  , 3 , S  , 4 , and T (7, 6 ), so the vectors joining the midpoints are:
 2  2 
 17   11   3   11 
  3   7 
u = PQ =  2  −   =  2  , v = ST =   −  2  =  2 
   1    6    
 3   2   4  2 
1 
We can see that u = v = MR ⇒ u = v and u  v , so the points PQST form the parallelogram.
2
5 m(u + v ) − 5i + 7j = n(u − v )
m [( 5i + 3 j) + (i − 4 j)] − 5i + 7j = n [( 5i + 3 j) − (i − 4 j)]
m [6i − j] − 5i + 7j = n [4i + 7 j]
( 6m − 5)i + ( −m + 7 ) j = 4 ni + 7nj
6m − 5 = 4 n 63 37
 ⇒m= ,n=
−m + 7 = 7n 46 46
6
12 B(9,12)
y Marc o at 08:00

10
M(9,10.2)
Marc o at 12:30

8
A(4.5,6)
Marc o at 07:30
6

O
5 10 15 x 20
Base
Start
-2 at 07:00 T(9,-2)
Tony at 12:30

-4
D(9,-4)
Tony at 07:30
-6

-8

-10

a The speed of ‘Marco’ is: m = 92 + 122 = 15 km/h.

The speed of ‘Tony’ is: t = 182 + ( −8 )2 ≈ 19.7 km/h.

b At 07:30 each crew will have been travelling for 0.5 hour, so their position vectors will be equal to one-half of their
velocity vectors.
 1  9   4.5 
Position of ‘Marco’ is: OA =  = .
2  12   6 
 1  18   9 
Position of ‘Tony’ is: OD =  = .
2  −8   −4 

2
c The distance between the vehicles at 07:30 is:
  4.5 
AD =  = 4.52 + ( −10 )2 ≈ 10.97 km.
 −10 
d ‘Marco’ is directly north of ‘Tony’ when the x-coordinate of its position point is 9, which is one hour after they left the
base port; i.e. ‘Marco’ starts work at 08:00 (and is at B(9,12)).

e The ‘Marco’ crew work for 4.5 hours and lay 4.5 × 0.4 = 1.8 km of pipe in a southerly direction.

The ‘Tony’ crew work for 5 hours and lay 5 × 0.4 = 2 km of pipe towards the north.

At 12:30 ‘Marco’ is at point M(9,12 2 1.8) = M(9, 10.2) and ‘Tony’ is at point T(9, 24 + 2) = T(9, 22).

The distance between them at this time is: 10.2 + 2 = 12.2 km.

f The distance from M to base port for ‘Marco’ is: OM = 92 + 10.22 ≈ 13.6 km.

As they travel at 15 km/h, it would take them 13.6 / 15 = 0.9069 hours = 54 minutes to return to base.
     3   5  8
7 a OR = OT + TR =  + =
 −1   6   5 
 
  3
As TI = kj, points T and I have the same x-coordinate, i.e. OI =   .
 a

   5  8−3 55
TR ⋅ IR = 0 ⇒   ⋅  = 0 ⇒ 5 × 5 + 6( 5 − a ) = 0 ⇒ a =
 6   5 − a  6

I(3, 55/6)
y

R(8,5)

5 x 10
O

T(3,-1)
-2

-4

 3   5 
     8
b IR = OR − OI =   −  55  =  25 
 5    − 
 6   6 

3
Chapter 9

 x   25   360   745 
8 a For t = 2, the position of the AUA plane is:  y  =   + 2  480  =  1000  .
   40

 360 
b The speed of the plane is:  = 3602 + 4802 = 600 km/h.
 480 
 x   −155   480 
c The position of the LH plane is given by:  y  =   +t .
   1300   −360 
The planes will collide if the following system of equations has a unique solution:
 x   25   360 
  =   + t  480 
y
   40
 ⇒
 x   −155   480 
 y  =  1300  + t  −360 
 
25 + 360t = −155 + 480t ⇒ t = 1.5

40 + 480t = 1300 − 360t ⇒ t = 1.5
Therefore, the planes will collide after 1.5 hours, i.e. at 01:30.
 x   −155   480   325 
d The position vector of the LH plane at t = 1 is:  y  =   + 1 = .
   1300   −360   940 
 325   450   775 
e After 2 hours the LH plane is at:   + 1 = .
 940   −390   550 
 775   745   30 
The distance between the planes after 2 hours is:  − =  = 302 + ( −450 )2 = 451 km.
 550   1000   −450 

9  3n   2n − 1   3n   2n − 1 
 2n + 3  ⊥  4 − 2n  ⇒  2n + 3  ⋅
 4 − 2n 
= 0 ⇒ 3n( 2n − 1) + ( 2n + 3)( 4 − 2n) = 0 ⇒ 2n2 − n + 12 = 0

For this quadratic equation, the discriminant ∆ = ( −1)2 − 4 × 2 × 12 = −95 < 0, so there is no solution for n.

10 For vectors a = cos θi + sin θ j and b = sin θi + cos θ j , we have:


cos θ ⋅ sin θ + sin θ cos θ 2 sin θ cos θ
cos ( a, b ) = =
cos θ + sin θ sin θ + cos θ
2 2 2 2 1⋅ 1
 π  π
= sin( 2θ ) = cos  − 2θ ⇒ α = − 2θ
2  2

 xa   xb   x a + xb   x a − xb 
11 For vectors a =   , b =   ⇒ a + b =  ,a−b =  .
 ya   yb  
 y a + yb   y a − yb 
Then:
a+b = a−b ⇒
( x a + x b )2 + ( y a + yb )2 = ( x a − x b )2 + ( y a − yb )2
x a2 + 2x a x b + x b2 + y a2 + 2 y a yb + yb2 = x a2 − 2x a x b + x b2 + y a2 − 2 y a yb + yb2 ⇒
2 x a x b + 2 y a y b = −2 x a x b − 2 y a y b ⇒ x a x b = − y a y b
x a x b + y a yb x a xb − x a xb
a⋅b = = =0
x +y
2
a
2
a x +y
2
b
2
b x + y a2 x b2 + yb2
2
a

4
Chapter 10
Practice questions

1 Method I:
1 − 3i (1 − 3i ) (1 + i ) 1 + 3 + i (1 − 3)
From (1 − i ) z = 1 − 3i , we have: z = = = = 2 − i. Hence, x = 2, y = −1 .
1− i 1+ 1 2
Method II:
We can solve the task using the notation z = x + y i . In this case, we have to solve a system of equations:
(1 − i ) ( x + y i ) = 1 − 3i ⇒
x + y + i ( y − x ) = 1 − 3i
x + y = 1 x=2
Hence:  ⇒
− x + y = −3 y = −1

2 a Since w is the non-real solution of the equation z 3 = 1, then w ≠ 1. Hence, we have:


=1
1+ w + w = 2 (1 + w + w 2 ) (1 − w ) 1 − w 3 0
= = =0
1− w 1− w 1
Note: We can calculate the value for each possible w:
 2π   2π   4π   2π   2π   4π   4π 
If w = cis   , then 1 + w + w 2 = 1 + cis   + cis   = 1 + cos   + i sin   + cos   + i sin  
 3  3  3  3  3  3  3
1 3 1 3
= 1− +i − −i =0
2 2 2 2
 4π   4π   8π   4π   2π 
If w = cis   , then 1 + w + w 2 = 1 + cis   + cis   = 1 + cis   + cis   . This is the same as in the
 3  3  3  3  3
previous case; hence, the value is 0.
=1 
=w =1
b (w x + w y ) (w x + w y ) = w x + w xy + w xy + w 3 y 2
2 2 3 2 4 2

= x 2 + y 2 + (w + w 2 ) xy
= x 2 + y 2 + (−1) xy 2
Since w + w = −1 (using the result from a).
= x 2 + y 2 − xy
3 a (1 + i )2 = 1 + 2i + i 2 = 1 + 2i − 1 = 2i
b Let P(n) be the statement: (1 + i ) = ( −4 ) .
4n n

Basic step:
The basis step must be P(1).

(1 + i )4 = ((1 + i )2 ) = (2i )2 = 22 i 2 = −4 = (−4)1 ; hence, P(1) is true.


Inductive step:
Assume that for some k ∈  + , P (k ) is true.

P (k ) ... (1 + i ) = ( −4 )
4k k

Now, (1 + i ) = (1 + i ) = (1 + i ) (1 + i ) = (−4) (−4) = (−4)


4 ( k +1) 4k + 4 k 4 k k +1
.

Hence, P (k + 1) is true, and, by mathematical induction, P(n) is true for all k ∈  + .

1
Chapter 10

c (1 + i )32 = (1 + i )4⋅8 = (−4)8 = 65 536


2
6 2 2 1 π
4 a For z1 : z1 = + = 2 , tan θ = − =− , the fourth quadrant, θ = − ; hence,
4 4 6 3 6
2
 π π 
z1 = 2  cos  −  + i sin  −  
  6  6 
1 π
For z 2 : z 2 = 1 + 1 = 2 , tan θ = − = −1, the fourth quadrant, θ = − ; hence,
1 4
  π  π
z 2 = 2  cos  −  + i sin  −  
  4  4 

 π π
2  cos  − 
 + i sin  − 
z1   6 6   π π π π  π π
b = = 1 cos  − +  + i sin  − +   = cos   + i sin  
z2  π π   6 4  6 4  12   12 
2  cos  − 
 + i sin  − 
  4 4 

6 −i 2 6 − 2i 2 + 6i − i 2
c
z1
= 2 ⋅
1+ i
= 2 =
6 + 2+i ( 6− 2 )
z2 1− i 1+ i 1+ 1 4

π 6+ 2  π 6− 2
Hence, a = cos   = , b = sin   = .
 12  4  12  4
3 3 3
 z1  a   π π  π π a   π  π 
5  z  =  b  cos  4 − 3  + i sin  4 − 3    =  b  cos  − 12  + i sin  − 12   
3

a3  π π 3
 a  π π  a3  2 2
= 3 
cos  − ⋅ 3 + i sin  − ⋅ 3  = 3  cos  −  + i sin  −   = 3  −
b   12   12   b   4  4  b  2 2 
3
z  a3 2 a3 2
Hence,  1  = −i .
 z3  2b 3
2b 3
Note: We can obtain the same result by firstly raising to the cube power and then dividing. However, be sure that you
a3
express the number in the form x + y i, which means that you multiply by 3 . You can write the result in the form:
b
a3 2 a3 2
x= ,y=− (or any equivalent form).
2b 3 2b 3
6 Let z = x + y i. Then:

( x + 16)2 + y 2 = 4 ( x + 1)2 + y 2 ⇒ ( x + 16)2 + y 2 = 16 (( x + 1)2 + y 2 )


x 2 + 32x + 256 + y 2 = 16 x 2 + 32x + 16 + 16 y 2

15x 2 + 15 y 2 = 240 ⇒ x 2 + y 2 = 16

Hence, z = x 2 + y 2 = 16 = 4.

7 Method I:
5 − i 5 − i 2 + i 10 + 1 + i (5 − 2) 11 + 3i
a + bi = = ⋅ = =
2−i 2−i 2+i 4 +1 5
11 3
Therefore, a = ,b = .
5 5
Note: If this is a Paper 2 task, you can perform the division using a GDC.

2
Method II:
We can find the solution using multiplication: (2 − i ) (a + bi ) = 5 − i ⇒ 2a + b + i (2b − a) = 5 − i
Therefore:
2a + b = 5 11 3
 ⇒ a = ,b =
− a + 2b = − 1 5 5
8 Method I:
1 1 1
If arg (b + i ) = 60 ⇒ arg (b + i ) = 30 ⇒ tan 30 = ⇒
2
= ⇒b= 3
b 3 b
Method II:
2b 2b
arg (b + i ) = 60 ⇒ arg (b2 − 1 + 2bi ) = 30 ⇒ tan 30 = 2
2
⇒ 3= 2
b −1 b −1
1
3b2 − 2b − 3 = 0 ⇒ b1 = 3 , b2 = −
3
Since b is positive, the solution is b = 3.

9 i (z + 2) = 1 − 2z ⇒ z i + 2i = 1 − 2z ⇒ z i + 2z = 1 − 2i ⇒ z (2 + i ) = 1 − 2i

1 − 2i 1 − 2i 2 − i 2 − 2 + i (−1 − 4) −5i
z= = ⋅ = = = −i
2+i 2+i 2−i 4 +1 5
10 a z 5 − 1 = (z − 1) (z 4 + z 3 + z 2 + z + 1)

b z 5 = 1 = cis 0
Hence, the zeros are the fifth roots of unity:
 2k π  k 0 1 2 3 4
cis  ; = , , , ,
 5 
2π 4π 6π 8π
1, cis   , cis   , cis   , cis   .
 5  5  5  5
 
4π 2π
cis  −  cis  − 
 5   5

 2π   4π 
The solutions are: 1, cis  ±  , cis  ± .
 5  5 

  2π     2π     2π   2π     2π   2π  
c  z − cis     z − cis  −   =  z − cos   − i sin     z − cos  −  − i sin  −  
5 5 5 5 5 5
 2π 2π    2π  2π 
=  z − cos   − i sin     z − cos   + i sin   
  5  5    5  5 
2
  2π    2π 
=  z − cos    + sin2  
  5   5
2π   2π   2π 2π 
= z 2 − 2 cos   z + cos2   + sin2   = z 2 − 2 cos   z + 1
 5  5  5  5
In the same way:

  2π     2π     4π   4π     4π   4π  
 z − cis     z − cis  −   =  z − cos   − i sin     z − cos  −  − i sin  − 
5 5 5 5 5 5 
2
 2π 2π   4π 4π   4π   4π
=  z − cos   − i sin     z − cos   + i sin    =  z − cos    + sin2  
  5  5    5  5    5   5
2
2π 2π   4π 4π   4π  4π 4π
− cos   − i sin     z − cos   + i sin    =  z − cos    + sin2   = z 2 − 2 cos   z + 1
 5  5    5  5    5   5  5
  2π    4π 
Hence: z 4 + z 3 + z 2 + z + 1 =  z 2 − 2 cos   z + 1  z 2 − 2 cos   z + 1
  5   5 

3
Chapter 10

π  π π
11 a 8i = 8, tan θ is not defined, positive y-axis, θ = , 8i = 8  cos + i sin 
2  2 2
 π  π 
  2k π   2k π   3   π 2k π  i sin  π 2k π   ; k = 0, 1, 2
b i 3
8i = 3 8  cos  2 + + i sin  2 + = 8  cos  + +  + 
3 3  3 3   6 3  6 3 
     
π π π
For k = 0, the number is in the first quadrant: 3 8i = 2  cos + i sin  = 2 cis
 6 6 6

 π π  3 1 
ii 3
8i = 2  cos + i sin  = 2  + i = 3 + i
 6 6   2 2 
12 a i All the numbers are of modulus 1; hence, their product and quotient is of modulus 1 and z = 1.
2 3
  π  π   π  π
 cos  −  + i sin  −    cos   + i sin   
4 4 3 3
ii z= 4
  π π
 
 cos   + i sin   
24 24

  π  π   π  π
 cos  −2 ⋅  + i sin  −2 ⋅    cos  3 ⋅  + i sin  3 ⋅  
4 4 3 3
=
  π  π 
 cos  4 ⋅  + i sin  4 ⋅  
24 24

  π  π
 cos  −  + i sin  −   (cos (π ) + i sin (π ))
2 2
=
  π  π
 cos   + i sin   
6 6
π π 2π
Hence, arg z = − +π+ = .
2 6 3
2π 2π
b Since z = cos   + i sin   , then
 3  3
3
  2π  2π  2π 2π
z 3 =  cos   + i sin    = cos  3 ⋅  + i sin  3 ⋅  = cos (2π ) + i sin (2π ) = 1
  3  3   3   3      
=1 =0

Hence, z is a cube root of one.


2
2π  2π   2π  2π  
c (1 + 2z ) (2 + z 2 ) = 2 + 4 z + z 2 + 2 z3 = 4 + 4 z + z 2 = 4 + 4  cos  + i sin    +  cos   + i sin   
=1
  3   3    3  3 
 1 3  4π 4π  1 3 3 3 3
= 4 + 4 − + i +  cos   + i sin    = 4 − 2 + 2 3i − − i = + i
 2 2    3  3  2 2 2 2
Note: We can write 4 + 4 z + z 2 as 1 + z + z 2 + 3 + 3z . Using the property of a root of one, 1 + z + z 2 = 0 , we have:

 2π 2π   1 3 3 3 3
4 + 4 z + z 2 = 3 + 3z = 3 + 3  cos   + i sin    = 3 + 3  − + i = + i
  3  3   2 2  2 2

2 2 1+ i 2 + 2i
13 z = + 1 − 4i = ⋅ + 1 − 4i = + 1 − 4i = 1 + i + 1 − 4i = 2 − 3i
1− i 1− i 1+ i 1+ 1
z = (2 − 3i ) = 4 − 12i − 9 = −5 − 12i
2

4
14 a Let P(n) be the statement: (cos θ + i sin θ) = cos nθ + i sin nθ .
n

Basis step:
The basis step is P(1) and it is true, because both sides are cos θ + i sin θ .

Inductive step:
Assume that for some k ∈  + P (k ) is true.

P (k ) ... (cos θ + i sin θ) = cos k θ + i sin k θ


k

Now, (cos θ + i sin θ) = (cos θ + i sin θ) (cos θ + i sin θ) = (cos k θ + i sin k θ) (cos θ + i sin θ)
k +1 k

= cos k θ cos θ − sin k θ sin θ + i (cos k θ sin θ + sin k θ cos θ)

= cos( k θ + θ ) + i (sin( k θ + θ )) = cos (( k + 1)θ) + i sin (( k + 1)θ)

Therefore, P (k + 1) is true and, by mathematical induction, P(n) is true for all k ∈  + .

b i Method I:
1 1 cos θ − i sin θ cos θ − i sin θ
= ⋅ = = cos (− θ) + i sin (− θ)
z cos θ + i sin θ cos θ − i sin θ cos2 θ + sin2 θ
Method II:
1
Using de Moivre’s theorem: = z −1 = (cos (θ) + i sin (θ)) = cos (− θ) + i sin (− θ)
−1

z
Note: In this part we can’t use the result from a, since here n = −1 and in a n ∈  + .

z − n = (z −1) = (cos (− θ) + i sin (− θ)) = cos (− nθ) + i sin (− nθ)


n n
ii

z n = (cos (θ) + i sin (θ)) = cos (nθ) + i sin (nθ)


n

z n + z − n = cos (nθ) + i sin (nθ) + cos (− nθ) + i sin (− nθ) = cos (nθ) + i sin (nθ) + cos (nθ) − i sin (nθ) = 2 cos (nθ)

c i (z + z )
−1 5
= z 5 + 5z 4 z −1 + 10z 3z −2 + 10z 2z −3 + 5zz −4 + z −5 = z 5 + 5z 3 + 10z + 10z −1 + 5z −3 + z −5

ii Using the result from i, we have: (z + z −1) = (2 cos θ) = 32 cos5 θ


5 5

and (z 5 + z −5 ) + 5 (z 3 + z −3 ) + 10 (z + z −1) = 2 cos (5θ) + 5 ⋅ 2 cos (3θ) + 10 ⋅ 2 cos (θ)

Thus, 32 cos5 θ = 2 (cos (5θ) + 5 cos (3θ) + 10 cos (θ)) .


1
Therefore, cos5 θ = (cos (5θ) + 5 cos (3θ) + 10 cos (θ)) , and a = 1, b = 5, c = 10 .
16
15 2 p + 2iq = q − ip − 2 (1 − i ) ⇒ 2 p + 2iq = −2 + q + i (2 − p)
2 p = −2 + q 2 p − q = −2
Hence:  ⇒ ⇒ p = −0.4 , q = 1.2
2q = 2 − p p + 2q = 2
5
   2π 2π     2π  2π 
16 i z15 =  2  cos   + i sin     = 25  cos  5 ⋅  + i sin  5 ⋅   = 32 (cos (2π ) + i sin (2π )) = 32
   5  5    5   5 
2
   2π 2π    4π 4π  4π
ii z12 =  2  cos   + i sin     = 4  cos   + i sin    = 4 cis
   5  5    5  5  5
3
   2π 2π    6π 6π  6π
z13 =  2  cos   + i sin     = 8  cos   + i sin    = 8 cis
   5  5    5  5  5
4
   2π  2π     8π 8π  π
z14 =  2  cos   + i sin     = 16  cos   + i sin    = 16 cis
   5  5    5  5  5
5
   2π 2π  
z15 =  2  cos   + i sin     = 32 (cos (2π ) + i sin (2π )) = 32
   5  5 

5
Chapter 10

iii

iv The transformation is a combination (in any order) of an enlargement of scale factor 2, with the origin as the

centre, and an anti-clockwise rotation of , again with the origin as the centre.
5
3
17 a Let z = a + bi . Then: a2 + b2 = a2 + (b − 3) ⇒ a2 + b2 = a 2 + b 2 − 6b + 9 ⇒ 6b − 9 = 0 ⇒ b =
2

2
b i

1.5 1 π
ii Since arg z1 = θ and cos θ = = ⇒θ= .
3 2 6

π 5π
iii arg z 2 = π − arg z1 = π − =
6 6
 zkz  k π 5π π k π π
c arg  1 2  = arg (z1k ) + arg (z 2 ) − arg (i ) = + − = +
 2i  6 6 2 6 3
kπ π
Hence: + =π⇒k =4
6 3
 2a + b = 7
18 2a + b + i (2 − ab) = 7 − i ⇒ 
 2 − ab = −1
Substituting b = 7 − 2a (from the first equation) into the second equation:
1
2 − a (7 − 2a) = −1 ⇒ 2a 2 − 7a + 3 = 0 ⇒ a1 = , a2 = 3
2
Since a , b ∈ , the solution is a = 3, b = 1.

6
z n = (cos (θ) + i sin (θ)) = cos (nθ) + i sin (nθ)
n
19 a
1
= z − n = cos (− nθ) + i sin (− nθ) = cos (nθ) − i sin (nθ)
zn
1
Hence, z n + = cos (nθ) + i sin (nθ) + cos (nθ) − i sin (nθ) = 2 cos (nθ) .
zn
4
 z + 1 3 1 1 1 1 1 1  1 1
+ 6z 2 2 + 4 z 3 + 4 = z 4 + 4 z 2 + 6 + 4 2 + 4 =  z 4 + 4  + 4  z 2 + 2  + 6

b  = z + 4 z
4
   
z z z z z z z z z 
4
1 1 1
Since  z +  = (2 cos (θ)) , z 4 + 4 = 2 cos ( 4 θ) , z 2 + 2 = 2 cos (2θ) , we have:
4
 z  z z
24 cos 4 (θ) = 2 cos ( 4 θ) + 4 ⋅ 2 cos (2θ) + 6 ⇒ 8 cos 4 (θ) = cos ( 4 θ) + 4 cos (2θ) + 3
1
Hence, cos 4 (θ) = (cos ( 4 θ) + 4 cos (2θ) + 3) .
8
1 2i θ
e 1 1
20 a z = 2 iθ = eiθ b z = , so it is less than 1.
e 2 2
u1 eiθ
c Using the formula: S∞ = =
1− r 1
1 −  e i θ 
2 
cos θ + i sin θ
d i S∞ =
1
1 − (cos θ + i sin θ)
2
1 2 i θ 1 3i θ  1 1   1 1 
ii S∞ = e i θ + e + e + ... =  cos θ + cos 2θ + cos 3θ + .... + i  sin θ + sin 2θ + sin 3θ + ...
2 4  2 4   2 4 
 1 1   1 1  cos θ + i sin θ
 cos θ + cos 2θ + cos 3θ + ... + i  sin θ + sin 2θ + sin 3θ + ... = 1
2 4 2 4 1 − (cos θ + i sin θ)
2
Taking the real parts, we have:
 
1 1  cos θ + i sin θ 
cos θ + cos 2θ + cos 3θ + ... = Re  
2 4 1
 1 − (cos θ + i sin θ)
2
   1 1
1 − cos θ + i sin θ 
 cos θ + i sin θ   cos θ + i sin θ 2 2 
Re 
1  = Re  1

1 1 
 1 − (cos θ + i sin θ)  1 − (cos θ + i sin θ) 1 − cos θ + i sin θ 
2 2 2 2
1 1
cos θ  1 − cos θ + i 2 sin2 θ
1 1
 2  cos θ − cos2 θ − sin2 θ
2 2 2
= =
2
 1 − 1 cos θ + 1 sin2 θ 1 1
  1 − cos θ + cos2 θ + sin2 θ
2 4 4 4
1
cos θ −
2 4 cos θ − 2
=
1 − cos θ +
1 5 − 4 cos θ
4
21 Method I:
If −3 + 2i is a root, then −3 − 2i is another root; therefore:

( )
P ( z ) = (z + 2) (z + 3 − 2i ) (z + 3 + 2i ) = (z + 2) (z + 3) − (2i ) = (z + 2) (z 2 + 6z + 13) = z 3 + 8z 2 + 25z + 26
2 2

So, a = 8, b = 25, c = 26.

7
Chapter 10

Method II:
P ( −2) = 0 ⇒ −8 + 4 a − 2b + c = 0

P ( −3 + 2i ) = 0 ⇒ 9 + 46i + a (5 − 12i ) + b (−3 + 2i ) + c = 0 ⇒ (9 + 5a − 3b + c ) + ( 46 − 12a + 2b) i = 0

Hence, we have to solve the system of equations:


4 a − 2b + c = 8

5a − 3b + c = −9 ⇒ a = 8, b = 25, c = 26
−12a + 2b = −46

22 Let z = a + bi :

a2 + b2 = 2 5 ⇒ a2 + b2 = 20
25 15 25 (a − bi ) − 15 (a + bi )
− = 1 − 8i ⇒ = 1 − 8i
a + bi a − bi (a + bi ) (a − bi )
10a − 40bi 10a − 40bi
= 1 − 8i ⇒ = 1 − 8i
a2 + b2 20
10a − 40bi = 20 − 160i ⇒ a = 2, b = 4

Hence, z = 2 + 4i .

 i z 1 + 2z 2 = 3
23 
 z1 + (1 − i ) z 2 = 4 / ⋅( −i )
 i z1 + 2z 2 = 3

−i z1 − (1 + i ) z 2 = −4i

3 − 4i 7 1
( 2 − 1 − i )z 2 = 3 − 4 i ⇒ z 2 = = − i
1− i 2 2
Substituting in the second equation: z1 + 3 − 4i = 4 ⇒ z1 = 1 + 4i

4 ± 16 − 32 4 ± 4i
24 a z1, 2 = = = 2 ± 2i ⇒ z1 = 2 + 2i , z 2 = 2 − 2i
2 2 π
π i
z1 = 4 + 4 = 2 2 , tan θ = 1, in the first quadrant, θ = ; hence, z1 = 2 2e 4
4
π
π −i
z2 = 4 + 4 = 2 2 , tan θ = −1, in the fourth quadrant, θ = − ; hence, z1 = 2 2e 4

4
( )
4 i

z14 2 2 e 4
 3π 3π
n  = −8i
i
b = −2 π = 8 e
2
= 8  cos + i sin
z 22  2
( ) 2
2 i
2 2 e 4

= (2 2) e
4 i
c z14 4
= 64 e i π = −64
−4 π
= (2 2) e
4 i
z 24 4
= 64 e − i π = −64

Thus, they are the same.


π −π
i i π π π
−π π π
z1 z 2 2 2e 4 2 2e 4 i  +  i  − −  i i
 4 4
d + = −π + π = e + e  4 4 = e 2 + e 2 = i − i = 0
z 2 z1 i i
2 2e 4 2 2e 4
nπ 3n nπ

( )
n i i
e z1n = 2 2 e = 22 e 4 4


The number is real if = k π ⇒ n = 4k , k ∈  .
4

8
2π 2π 7 ⋅ 2π 7 ⋅ 2π
7

25 a z 7 =  cos + i sin  = cos + i sin = cos
2 π + i sin
 2π = 1 ; hence z 7 − 1 = 0.
 7 7  7 7 =1 =0

b (z − 1) (z 6 + z 5 + z 4 + z 3 + z 2 + z + 1) = z 7 + z 6 + z 5 + z 4 + z 3 + z 2 + z − z 6 − z 5 − z 4 − z 3 − z 2 − z − 1 = z 7 − 1
Using the result from a, we have: 0 = z 7 − 1 = (z − 1) (z 6 + z 5 + z 4 + z 3 + z 2 + z + 1) . Since z ≠ 1, then z − 1 ≠ 0 and
hence z 6 + z 5 + z 4 + z 3 + z 2 + z + 1 = 0 .

c Using the result from b, we have:


12π  10π  8π 6π 4π  2π
0 = z 6 + z 5 + z 4 + z 3 + z 2 + z + 1 = cis  + cis  + cis   + cis   + cis   + cis   + 1
 7   7   7  7   7   7
 12π   10π   8π   6π   4π   2π 
cis  + cis  + cis   + cis   + cis   + cis   = −1
 7   7   7  7   7   7
  12π   10π   8π   6π   4π   2π  
Hence, Re  cis  + cis  + cis   + cis   + cis   + cis    = −1
  7   7   7  7  7   7 
  12π   10π   8π   6π   4 π   2π  
Re  cis  + cis  + cis   + cis   + cis   +   
  7   7   7  7  7   7 
12π  10π  8π  6π   4π   2π
= cos  + cos  + cos   + cos   + cos   + cos  
 7   7   7  7  7   7
 12π   2π   10π   4π   8π   6π 
Since cos  = cos   , cos  = cos   , cos   = cos   , we have:
 7   7  7   7   7  7 
 12π   10π   8π   6π   4π   2π    6π   4π   2π  
cos  + cos  + cos   + cos   + cos   + cos   = 2  cos   + cos   + cos   
 7   7   7  7   7   7   7   7   7 
  6π   4π   2π    6π   4π   2π  1
Finally: 2  cos   + cos   + cos    = −11 ⇒ cos   + cos   + cos   = −
  7   7   7   7  7   7 2

9
Chapter 11
Practice questions

30
360
1 a ∑y i = 360 ⇒ µ =
30
= 12
i=1

30
925
∑ (y − µ) = 925 ⇒ sn =
2
b i ≈ 5.55
i =1 30

2 µ=
∑xf i i
⇒ 34 =
10 × 1 + 20 × 2 + 30 × 5 + 40 × n + 50 × 3
⇒ 34 =
3550 + 40n

∑f i 11 + n 11 + n
374 + 34 n = 350 + 40n ⇒ 24 = 6n ⇒ n = 4

3 a
Time (minutes) 1.6 2.1 2.6 3.1 3.6 4.1 4.6 5.1 5.6 6.1 6.6
Frequency 2 5 5 5 14 6 6 2 3 2 0

16 Frequency

14

12

10

2
Time
0
1.5 2 2.5 3 3.5 4 4.5 5 5.5 6 6.5 7

b There are 7 out of 50 measurements that are greater than or equal to 5.1; therefore, the fraction of the measurements
43
less than 5.1 is: = 0.86 = 86%.
50
c There are 50 pieces of data, so, to determine the median, we need to find the 25th and 26th observations. We notice
that these two observations are within the interval 3.6–4.1; therefore, the median is approximately 3.9.

d We can input the time as a sequence by using the sequence list feature on a GDC.

The mean value is 3.68, whilst the standard deviation is 1.11, correct to three significant figures.

1
Chapter 11

e Cumulative frequency

50

40

30

20

10

Time
0
2 3 4 5 6

f Estimates for the minimum and maximum values are 1.6 and 6.6 respectively. The first and third quartiles correspond
to the cumulative frequencies of 12.5 and 37.5 respectively; therefore, an estimate for the first quartile is 3.15 and the
third quartile is 4.65. An estimate for the median (which corresponds to a cumulative frequency of 25) is 3.9.

4 a The median and the IQR would best represent the data, since the data is skewed to the right and there are a few
outliers on the right.

b Firstly, we are going to read the frequencies from the histogram and input them into the frequency distribution table
on our GDC.

Finally, we apply the statistical calculation for one variable.

The mean value is 682.6 and the standard deviation is 536.2.

c Since we have grouped data, the endpoints of our intervals will be 150, 250, 350, ..., and so on. On a calculator, we can
use the adding a number to the list feature. Alternatively, we can calculate the cumulative frequencies from the List
menu.

2
d There are 460 cities, so, to estimate the median, we need to draw a horizontal line from 230. To find the lower and
upper quartiles, we need to draw horizontal lines from 115 and 345 respectively.

For a better estimation, we will use the zoom box feature and turn the grids on.

So, the median is about 500. The first quartile is about 330 and the third quartile is about 830. Therefore, the IQR is
about 830 – 330 = 500.

e There are a few outliers to the right. The outliers are those points which are over Q3 + 1.5IQR, i.e.
830 + 1.5 × 500 = 1580, which gives us 50 cities from the histogram.

f The data is skewed to the right with quite a few outliers to the right (1600 and above). The data is also bimodal, with
the modal values being 300 and 400.

5 a It appears that Spain produces both the most expensive (estimated €152 per case) and the cheapest (estimated €55
per case) red wine.

b Red wines are generally more expensive in France as we can see that the median price is the highest; the minimum
value in France is also the highest, but the upper 50% of wines are also within a very small range of approximately €10
per case.

c It appears that the wines are, on average, more expensive in France, where the prices are skewed towards the higher
end. In Spain, you can find a higher percentage of cheaper wine than in the other two countries, but you also find
the most expensive wines on the market; so Spain has the widest range of prices. Italy seems to have the most
symmetrical distribution of wine prices.

6 a The mean value of the data is 52.6 and the standard deviation is 7.60, both given correctly to three significant figures.

b The median value is 51.3. The upper and lower quartiles are 49.9 and 52.6 respectively; therefore, the IQR is 2.65, all
correct to three significant figures.

c Since there is one outlier (112.72), the mean value is more influenced by it than the median value and IQR.

7 a The distribution is not symmetric since the median is not the midpoint between the minimum and maximum value,
nor is it the midpoint between the first and the third quartile.

b The outliers lie 1.5IQR further to the left of the lower quartile and 1.5IQR further to the right of the upper quartile. If
there are any outliers, they will lie below 37 and above 99, which is outside of the given range; therefore, there are no
outliers.

3
Chapter 11

y
c 4

0 x
40 50 60 70 80 90

d Additionally to the description provided in part a, we can say that the data is skewed to the left.

8 a From the graph, we can estimate that the median cholesterol level (50% of the cumulative percentage) is 225.

b The first and third quartiles are estimated from 25% and 75% of the cumulative percentage; our estimations are 210
and 260 respectively. The 90th and 10th percentiles are estimated from 90% and 10% of the cumulative percentage;
our estimations are 300 and 190 respectively.

c Using our answers from part b, the estimated IQR will be 50. Since 2000 patients have been studied, the number of
patients in the middle 50%, ranging from 210 to 260, is 1000.
y
d 4

0 x
100 125 150 175 200 225 250 275 300 325 350 375 400

e We notice that the data is skewed to the right a bit, with more outliers on the right side, since the outliers lie outside
of the interval 135 to 335. From the cumulative frequency graph, we read that there are almost 100 patients who have
a cholesterol level greater than 335 mg/dl and only a few patients with a level less than 135 mg/dl.
9a Speed Frequency b 10 Frequency density

26–30 10
8

31–34 16
6
35–38 30
39–42 23 4

43–46 12
2
47–50 8
Speed
51–54 1 0
25 28 31 34 37 40 43 46 49 52 55 58

4
c d
xi fi xi × fi xi2 × fi xi fi ci
28 10 280 7840 30 10 10
32.5 16 520 16 900 34 16 26
36.5 30 1095 39 967.5 38 30 56
40.5 23 931.5 37 725.75 42 23 79
44.5 12 534 23 763 46 12 91
48.5 8 388 18 818 50 8 99
52.5 1 52.5 2756.25 54 1 100
Σ 100 3801 147 770.5
µ = 38.01 s2 = 32.9449
s = 5.739 765

e In order to estimate the median, we need to draw the cumulative frequency diagram. We will do this using a calculator.

An estimate of the median is 37. Q1 is 34 and Q3 is 42; therefore, the IQR is 8.

f Since Q3 is 42 and 1.5IQR is 12, there is a possible outlier, which is the largest observation of 54.

10 a
Classes Frequency
1101–1200 1
1201–1300 1 Using Autograph software with the grouped data, we obtain the
following results:
1301–1400 0
The mean value is 1850.5 and the standard deviation is 232.379.
1401–1500 1
The median value is 1900.5. The first and third quartiles are
1501–1600 3 1714.14 and 2019.25 respectively; therefore, the IQR is 305.11.

1601–1700 5
1701–1800 11
1801–1900 3
1901–2000 11
2001–2100 8
2101–2200 4
2201–2300 2
b By looking at the frequency distribution table, we can surmise that there may be some outliers to the left, so we need
to calculate Q1 – 1.5IQR = 1256. From the table, we can see that we have one outlier.

5
Chapter 11

c 5 f

Gasoline (l)
0
1000 1100 1200 1300 1400 1500 1600 1700 1800 1900 2000 2100 2200 2300 2400

d µ ± sn = 1850.5 ± 232.379 ⇒ [1618, 2083]


e Germany will definitely be an outlier since Q3 + 1.5IQR = 2477 < 2758; therefore, it will influence the mean and
standard deviation. Conversely, the median, the first and third quartiles, and the IQR will not change much.
90
4460
11 a ∑x i = 4460 ⇒ µ =
90
≈ 49.6 minutes
i=1
4460 + 35 + 39 + 28 + 32 4594
b µNew = = ≈ 48.9 minutes
90 + 4 94

12 a
Marks < 10 < 20 < 30 < 40 < 50 < 60 < 70 < 80 < 90 < 100
Number of candidates 30 130 330 670 1190 1630 1810 1900 1960 2000
b 2200 Cumulative frequency

2000

1800

1600

1400

1200

1000

800

600

400

200
Marks
10 20 30 40 50 60 70 80 90 100

c i By looking at the graph, we estimate that the median score (which corresponds to the cumulative frequency of
1000) is 46.

ii We draw a vertical line from 35 on the Marks axis and reach the cumulative frequency curve at the point at which
the cumulative frequency is about 500. Therefore, 500 candidates had to retake the exam.

iii The highest scoring 15% corresponds to the highest 300 results; therefore, we draw a horizontal line from 1700 on
the Cumulative frequency axis and reach the curve at the point at which the number of marks is about 62. Hence,
a distinction will be awarded if 62 or more marks are scored on the test.

72 × 1.79 + 28 × 1.62 174.24


13 µ = = ≈ 1.74 cm, correct to the nearest centimetre.
72 + 28 100
25
300
14 a ∑ xi = 300 ⇒ µ = 25 = 12
i=1
25
625
∑ (x − m) = 625 ⇒ sn =
2
b i =5
i =1 25

6
15 Score 10 20 30 40 50
Number of competitors 1 2 5 k 3

10 × 1 + 20 × 2 + 30 × 5 + 40 × k + 50 × 3
x = 34 ⇒ = 34 ⇒ 350 + 40k = 374 + 34k ⇒ 6k = 24 ⇒ k = 4
11 + k
16 a To calculate an estimate for the mean, we will take the midpoints of the intervals (15, 45, 75, and so on) and the
corresponding frequencies.

So, an estimate for the mean of the waiting times is 97.2 seconds.

(Note: Even though we could have used only the mean feature from the List menu, we used the whole statistics
calculation since there is a chance that we will need further statistics in the following parts of the problem.)

b Waiting time (seconds) < 30 < 60 < 90 < 120 < 150 < 180 < 210 < 240
Cumulative frequency 5 20 53 74 85 92 97 100
c
110 Cumulative frequency

100

90

80

70

60

50

40

30

20

10
Waiting time (s)
10 20 30 40 50 60 70 80 90 100 110 120 130 140 150 160 170 180 190 200 210 220 230 240

d To find the three estimations asked for, we need to draw a horizontal line at 50 for the median and at 25 and 75 for
the quartiles. An estimation of the mean value is 87, whilst the lower and upper quartiles are 65 and 121 respectively.

17 a Taking our readings from the histogram:

i There are 10 plants that have a length between 50 and 60 cm.

ii To find the number of trees that have a length between 70 and 90 cm, we need to add two frequencies:
14 + 10 = 24.

b As in question 15, we again take the midpoints of the intervals (15, 25, 35, ..., 95) and the corresponding frequencies.

xi 15 25 35 45 55 65 75 85 95

fi 1 5 7 9 10 16 14 10 8

We input the two lists into a GDC and get the following results.

7
Chapter 11

So, an estimate for the mean is 63, and the standard deviation is 20.5.

c The data is skewed to the left; therefore, the median and the mean are different values.

d Since there are 80 plants, the median corresponds to the cumulative frequency of 40. Since the values of the
cumulative frequencies are symmetrical around 40, 32 for 60 and 48 for 70, we can estimate the cumulative frequency
of 40 as 65, i.e. the median is less than or equal to 65 cm.

18 a Again, we will use the midpoints of the intervals (82.5, 87.5, 92.5, ..., and so on) and the corresponding frequencies. We
put the two lists into our GDC and obtain an estimation of the standard deviation.

So, an estimate for the standard deviation of the weights is 7.405.

b
Weight (W) W < 85 W < 90 W < 95 W < 100 W < 105 W < 110 W < 115
Number of packets 5 15 30 56 69 76 80
c
Cumulative frequency
80

70

60

50

40

30

20

10
Weight (g)
0
80 85 90 95 100 105 110 115

i An estimate for the median (which corresponds to the cumulative frequency of 40) is 97.

ii An estimate for the upper quartile (which corresponds to the cumulative frequency of 60) is 101.

d (W − W ) + (W
1 2 − W ) + (W3 − W ) + ... + (W80 − W ) = (W1 + W2 + W3 + ... + W80 ) − 80 × W

W1 + W2 + W3 + ... + W80
= (W1 + W2 + W3 + ... + W80 ) − 80 × =0
80
e There are 71 packets that satisfy the condition 85 < W  110 . There are 20 packets that satisfy the condition
20
100 < W  110. Therefore, the probability is: P (E ) = ≈ 0.282 , correct to three significant figures.
71

8
19 a We will use the midpoints of the intervals (65, 75, 85, ..., and so on) and the corresponding frequencies. We put the
two lists into our GDC and calculate the mean.

So, the mean speed is 98.2 km h−1, correct to three significant figures.
b i To find the value of a we can either add 70 (the frequency of the speed interval 90–100) to the previous
cumulative frequency, 95; or subtract 71 (the frequency of the speed interval 100–110) from the next cumulative
frequency, 236. In both cases we get the same value: a = 165.

In a similar way, we find the value of b: b = 236 + 39 = 275.


ii 300 Cumulative frequency
280
260
240
220
200
180
160
140
120
100
80
60
40
20 Speed (km/h)
0
60 70 80 90 100 110 120 130 140

c i We draw a vertical line from v = 105 until we reach the cumulative frequency curve. This gives us an estimate for
the cumulative frequency of 200. So, there are 100 cars that will exceed the speed of 105 km h−1 and
100
P= = 0.333... ≈ 33.3%.
300
ii If 15% of the cars exceed this speed, then 85% do not exceed that speed. 85% of 300 is 255, so we draw a
horizontal line from y = 255 until we reach the cumulative frequency curve. This gives us an estimate of a speed
of 115 km h−1.

20 a i We take a horizontal line across from 100 on the vertical axis until it touches the graph. From that point, we take a
vertical line down to the horizontal axis and read off the value. An estimate for the median fare is $24.

ii We take a vertical line up from 35 on the horizontal axis until we reach the graph. From that point, we take a
horizontal line across to the vertical axis and read off the value. An estimate for the number of cabs in which the
fare taken is $35 or less is 158.

b 40% of the cabs is 0.4 × 200 = 80. So, we take a horizontal line from 80 on the vertical axis until we reach the graph
and then we estimate the x-coordinate, which is 22. Therefore, the fare is $22. To find the number of kilometres, we
need to divide the fare by 0.55 (which is the fare per kilometre for distance travelled). Therefore, the distance travelled
is 40 km.

c If the distance travelled is 90 km, the driver will earn 90 × 0.55 = 49.5 dollars. We will use the graph to estimate the
number of cabs that will earn less than 49.5 dollars: there are 184, and therefore there are 16 cabs that will earn more
16
than that. So, the percentage of the cabs that travel more than 90 km is: = 0.08 = 8%.
200

9
Chapter 11

21 Since the three numbers are given in order of magnitude, a < b < c , we know that the middle one (the median) is 11, so
b = 11. Given that the range is 10, we know that the difference between the minimum and maximum value is: c 2 a = 10.
a + 11 + c
Since the mean value is 9, we can establish another equation in terms of a and c: = 9 ⇒ a + c = 16. Solving
3
these two equations (using the elimination method), we get: 2a = 6 ⇒ a = 3 . So, finally, c = 13.

22 a
110 Cumulative frequency
105
100
95
90
85
80
75
70
65
60
55
50
45
40
35
30
25
20
15
10
5 Price (thousands of $)
50 100 150 200 250 300 350 400 450 500

b By using horizontal lines at 25 and 75, we estimate the values of Q1 and Q3 as $130 000 and $250 000 respectively.
Hence, the IQR is $120 000.

c To find the frequencies a and b we need to subtract two successive cumulative frequencies.

fi = ci − ci −1 : a = 94 − 87 = 7, b = 100 − 94 = 6

d We input the midpoints of the intervals in the first list and the corresponding frequencies in the second list:

So, an estimate of the mean selling price is $199 000.

e i An estimate of the cumulative frequency for $350 000 is 92; therefore, there are about eight houses that can be
described as De Luxe.

ii Out of eight De Luxe houses, six were sold for $400 000; therefore, the probability that both selected houses have a
selling price more than $400 000 is:
6 5 15
P (E ) = × = .
8 7 28
23 a i To mark the median, we draw the horizontal line y = 40 until it hits the graph, at which point we draw a vertical
line down to the Diameter axis. An estimate for the median is 20 mm.

ii To mark the upper quartile, we draw the horizontal line y = 60 until it hits the graph, at which point we draw a
vertical line down to the Diameter axis. An estimate for the upper quartile is 24 mm.

b The interquartile range is: IQR = 24 − 14 = 10 mm.

10
24 a In this question we can accept an error of ±2 students.

We need to read the cumulative frequencies at the endpoints of the intervals and then subtract the successive ones
to obtain the frequencies.

For 40 the cumulative frequency is 74, and therefore the corresponding frequency is 74 − 22 = 52.

For 60 the cumulative frequency is 142, and therefore the corresponding frequency is 142 − 74 = 68.

For 80 the cumulative frequency is 180, and therefore the corresponding frequency is 180 − 142 = 38.

Mark (x) 0 < x < 20 20 < x < 40 40 < x < 60 60 < x < 80 80 < x < 100
Number of students 22 52 68 38 20

b 40% of 200 students is 80. The cumulative frequency of 80 corresponds to about 42 marks. So, the pass mark is about
42%.

25 a To find the median height we draw the horizontal line y = 60 until it hits the graph. We then estimate the
x-coordinate of the point of intersection. We estimate 183 cm.
b For the lower and upper quartiles, we draw two horizontal lines: y = 30 and y = 90. Then we estimate the
x-coordinates of the points of intersection. Therefore: Q1 = 175, Q3 = 189 ⇒ IQR = 189 − 175 = 14 .
26 Since the modal value is 11, we know that c = d = 11. Given that the range is 8, we can find the value of a:
11 − a = 8 ⇒ a = 3 . Finally, given that the mean value is 8, we can find the remaining number b:
3 + b + 11 + 11
= 8 ⇒ 25 + b = 32 ⇒ b = 7 .
4
27 a We draw the vertical line x = 40 until it hits the graph. We then estimate the y-coordinate of the point of intersection
as 100. So, the number of students who scored 40 marks or less is 100.

b There are 800 students, so the middle 50% is between 200 and 600 students. For a cumulative frequency of 200, the
estimated mark is 55; whilst for 600, the estimated mark is 75. Hence, we say that the middle 50% of test results lie
between 55 and 75 marks: a = 55, b = 75.

Solution Paper 1 type

28 We are going to add all the known observations and use the mean formula to find the first equation relating the
x + y + 90
unknowns: 13 = ⇒ 104 = x + y + 90 ⇒ x + y = 14
8
We are going to use the sum of the squares of the known observations and the variance formula to find the second
equation relating the squares of the unknowns:
x 2 + y 2 + 1404 x 2 + y 2 + 1404
21 = − 132 ⇒ 190 = ⇒ 1520 = x 2 + y 2 + 1404 ⇒ x 2 + y 2 = 116
8 8
Now, we need to solve the simultaneous equations by using the substitution method.
 x + y = 14  y = 14 − x  y = 14 − x  y = 14 − x
 2 ⇒ 2 ⇒ 2 ⇒ 2 ⇒
 x + y = 116  x + (14 − x ) = 116
2 2
 2x − 28 x − 80 = 0  x − 14 x − 40 = 0

 y = 14 − x  y = 14 − x  y = 10 or y = 4

 ⇒ ⇒
 ( x − 4 ) ( x − 10 ) = 0  x = 4 or x = 10  x = 4 or x = 10
Since x < y , we can discard the second solution.

11
Chapter 11

Solution Paper 2 type

28 Since the calculator cannot solve simultaneous equations by using the list features, we need to reduce the equation to
only one unknown. Therefore, the mean value is going to give us the substitution.
x + y + 90
13 = ⇒ 104 = x + y + 90 ⇒ x + y = 14 ⇒ y = 14 − x
8
The problem with the calculator is that the feature that calculates the variance actually calculates the unbiased estimate
n−1 7
of the variance; therefore, we need to multiply by , which in our case is .
n 8

Since x < y , we can discard the second solution; therefore, x = 4 and y = 14 − 4 = 10 .

12
Chapter 12
Practice questions

1 a Since the events are independent: P ( A ∩ B ) = P ( A) × P (B ) ⇒ 0.18 = k × (k + 0.3) ⇒ k 2 + 0.3k − 0.18 = 0


(k + 0.6) (k − 0.3) = 0 ⇒ k = −0.6 or k = 0.3
Note: Algebraically we get two solutions, but only 0.3 can be a probability value since probability cannot be negative.

b Using the addition formula: P ( A ∪ B ) = P ( A) + P (B ) − P ( A ∩ B ) = 0.3 + 0.6 − 0.18 = 0.72

c Since the events are independent, the complementary events are independent too.
P ( A ' B ') = P ( A ') ⇒ P ( A ' B ') = 1 − 0.3 = 0.7

2 a Since the tests are taken independently, we multiply the probabilities: P ( A) = 0.02 × 0.02 = 0.0004

b Using the complementary event: P (B ) = 1 − P ( A) = 1 − 0.0004 = 0.9996

c P (C ) = 0.02 × 0.02 = 0.0004

Note: The probabilities in parts a and b are equal because the events are the same.

3 Since they work independently of each other, we need to multiply the probabilities and then use the complementary event.

P ( A ') = 1 − P ( A) = 1 − 0.002 × 0.01 = 0.999 98


120 60 10 170 17
4 a i Using the addition formula: P (S ∪ F ) = P (S ) + P (F ) − P (S ∩ F ) =
+ − = =
200 200 200 2000 20
ii Either but not both means that we need to exclude the intersection from the union, so:
170 10 160 4
P (S ∪ F ) − P (S ∩ F ) = − = =
200 200 200 5
iii Does not take any French or Spanish is the complementary event of the union, so:
17 3
P ((S ∪ F ) ') = 1 − P (S ∪ F ) = 1 − =
20 20 10
P (F ∩ S ) 200 1
b Using the conditional probability formula: P (F S ) = = =
P (S ) 12 0 12
200
5 It would be a good idea to find the total sums first. There are 126, 84 and 160 disks produced after one run on machines
I, II and III respectively. There are a total of 20 defective and 350 non-defective disks. That means there are 370 disks
produced in total.
126 63
a i P ( Ai ) = =
370 185
4 2
ii P ( Aii ) = =
370 185
126 + 350 − 120 356 178
iii We need to use the addition formula: P ( Aiii ) = = = .
370 370 185
iv Since this is a conditional probability, our sample space is defective disks and the favourable outcomes are those
6 3
that are produced by machine I, so P ( Aiv ) = = .
20 10

1
Chapter 12

b If the quality is independent of the machine, then we can use the multiplication law:
126 63 20 1 63 6
P (M1) × P (D ) = × = ≠ = P (M1 ∩ D ) . So, the events are dependent.
370 185 200 10 1850 370
126 63
6 a There are 126 envelopes which satisfy our wish, so: P ( A) = = .
200 100
68 34
b There are 68 red envelopes without a prize, so: P (B ) = = .
70 35
P ( A ∩ B) 0.18 0.18
7 a P ( A B) = ⇒ 0.3 = ⇒ P (B ) = = 0.6
P (B ) P (B ) 0.3
b The events are independent since P (B ) = 0.6 = P (B A) .

c Given that A and B are independent, then A and B are independent too, so:
P (B ∩ A ') = P (B ) × P ( A ') = 0.6 × (1 − 0.3) = 0.42.

8 a Since we know that there are 74 students who took the test, the number of boys who failed is
74 − (32 + 16 + 12) = 14. There are 6 girls who are too young to take the test and, since there are 10 students
altogether that are too young to take the test, the number of boys who are too young is 10 − 6 = 4 . Since the total
numbers of boys and girls are 70 and 50 respectively, we calculate all of those who were training but did not take the
test as: 70 − (32 + 14 + 4) = 20, and 50 − (16 + 12 + 6) = 16.

Boys Girls
Passed the ski test 32 16
Failed the ski test 14 12
Training, but did not take the test yet 20 16
Too young to take the test 4 6

74 37
b i P (Bi ) = =
120 60
16 + 12 28 14
ii P (Bii ) = = =
50 50 25
iii These two events are independent so we multiply the probabilities:
32 16 16 8 128
P (Biii ) = × =
70 35 50 25 875
1 3 3
9 a P ( A ∩ B ) = P ( A B ) × P (B ) = × =
4 8 32
9 3 3 12 3
b P ( A ∪ B ) − P ( A ∩ B ) = P ( A) + P (B ) − 2 × P ( A ∩ B ) = + − 2× = =
16 8 32 16 16 4
27 5
c P (( A ∪ B ) ') = 1 − P ( A ∪ B ) = 1 − =
32 32
10 a Probability that she will miss both serves is: P ( A) = 0.4 × 0.05 = 0.02 = 2%.

b To win a point she will make the first serve and win the point or she will miss the first serve, make the second serve
and win the point, so: P (B ) = 0.6 × 0.75 + 0.4 × 0.95 × 0.5 = 0.64 = 64%.

11 a P ( A ∩ B ) = P ( A) + P (B ) − P ( A ∪ B ) = 0.6 + 0.8 − 1 = 0.4

b P ( A '∪ B ') = P (( A ∩ B ) ') = 1 − P ( A ∩ B ) = 1 − 0.4 = 0.6

2
12
Boys Girls Total
Television 13 25 38
Sport 33 29 62
Total 46 54 100
38 19
a P ( A) = =
100 50
13
b P (B ) =
46
13 a Outcomes
1
1 6
36
6

1 6
5
6 5 not 6
36
6

1 5
6
5 6 36
6 not 6

5 25
not 6
6 36

25 11
b Using the complementary event of not getting a 6: P (B ) = 1 − P (B ') = 1 − =
36 36
14 a
U A B

b i Since n ( A ∪ B ) = n (U ) − n (( A ∪ B ) ') = 36 − 21 = 15, then n ( A ∩ B ) = n ( A) + n (B ) − n ( A ∪ B ) = 11 + 6 − 15 = 2.


2 1
ii P ( A ∩ B ) = =
36 18
c Events A and B are not mutually exclusive since there are two elements in the intersection of the two sets: A ∩ B ≠ ∅ .

15 a There are 90 females, so there are 110 males. If 60 were unemployed, then 140 were employed. If 20 males were
unemployed, then 40 females were unemployed, and so on.

Males Females Totals


Unemployed 20 40 60
Employed 90 50 140
Totals 110 90 200
40 1
b i P (Bi ) = =
200 5
90 9
ii P (Bii ) = =
140 14

3
Chapter 12

16 There are three possible combinations of different colours and each can occur twice. In total, there are 26 balls in the bag.
 red green
red

white white 
green

10 10 2 10 2 6 6 10 2 20 + 12 + 12 44
P ( A) = 2 × × + 2× × + 2× × = =
26 13 25 5 26 13 25 5 26 13 25 5 65 65

17 a b Firstly, we need to find the number of elements in the intersection:


U A B
n ( A ∩ B ) = n ( A) + n (B ) − n ( A ∪ B ) = 30 + 50 − 65 = 15

So, n (B ∩ A ') = n (B ) − n ( A ∩ B ) = 50 − 15 = 35.


35 7
c P (B ∩ A ') = =
100 20

18 a b The student can take chemistry and biology or not take chemistry
B but take biology. So, P (B ) = 0.4 × 0.6 + 0.6 × 0.5 = 0.54.
0.6
c This is a conditional probability where the favourable event is when
0.4
C a student takes both subjects; therefore,
0.4
B' 0.4 × 0.6 4
P (C B ) = = .
0.54 9
0.5 B
0.6 C'

0.5 B'

19 a We will reuse the probability distribution table for the sum from Exercise 10.4 question 6:

S 2 3 4 5 6 7 8 9 10 11 12
 1
P(S) ___
36
 2
___
36
 3
___
36
 4
___
36
 5
___
36
 6
___
36
 5
___
36
 4
___
36
 3
___
36
 2
___
36
 1
___
36

21 7
P ( S < 8) = P ( S  7) = =
36 12
b There are 11 possible pairs when at least one die shows a 3: six pairs with 3 showing on the first die and six pairs with
11
3 showing on the second die, but the pair (3, 3) should only be counted once. So, P (B ) = .
36
c Now, the event from a becomes a sample space and we need to find the favourable pairs (out of 21 pairs) found in b.
The pairs that satisfy the condition are (1, 3), (2, 3), (3, 1), (3, 2), (3, 3), (3, 4) and (4, 3).
7 1
P (B S  7) = =
21 3
3 4 6 1
20 a P ( A ∩ B ) = P ( A) + P (B ) − P ( A ∪ B ) = + − =
11 11 11 11
3 4 12
b P ( A ∩ B ) = P ( A) × P (B ) = × =
11 11 121
21 a Since the probability of A didn’t change when B occurred, P ( A B ) = P ( A) ; therefore, the events are independent (I).

b If P ( A ∩ B ) = 0 ⇒ A ∩ B = ∅; therefore, the events are mutually exclusive (M).

c Given that P ( A ∩ B ) = P ( A) ⇒ A ∩ B = A ⇒ A ⊆ B , neither (N).

4
22 a n (E ∪ H ) = 88 − n ((E ∪ H ) ') = 88 − 39 = 49
Since we know how many study each subject, we can find the intersection:
b = n (E ∩ H ) = n (E ) + n (H ) − n (E ∪ H ) = 32 + 28 − 49 = 11
a = n (E ) − n (E ∩ H ) = 32 − 11 = 21
c = n (H ) − n (E ∩ H ) = 28 − 11 = 17
b Simply by looking at the Venn diagram, since we now have the values of a, b and c:
11 1
i P (E ∩ H ) = =
88 8
32 − 11 21
ii P (E \ (E ∩ H )) = =
88 88
Note: This can be directly read off the diagram.
c i There are 56 students who don’t study economics and, since a group of three students is selected, we obtain:
56 55 54 315
P (E ') = × × = .
88 87 86 1247
ii To find the probability that at least one student studies economics, we are going to use the complementary
315 932
event (no student studies economics): 1 − P (E ') = 1 − = .
1247 1247

red 
red  
yellow yellow

7 6 5 4 42 + 20 62 31 31
23 P (SC ) = × + × = = =
12 11 12 11 12 × 11 12 6 × 11 66
Note: The tins are chosen without replacement so once a tin is chosen there are 11 tins remaining.

24 a 7 1 1 3 7 3 35 + 12 47
1 L b P (L) = × + × = + = =
4
8 4 8 5 32 40 160 160
7
W
7
P (W ∩ L) 32 35
8 3 L' c P (W L) = = =
4 P (L) 47 47
3 160 5
L
1 5
8 W'

2
5 L'

120 1
25 a P (B ) = =
360 3
90 + 120 210 7
b P (S ) = 
= 
=
360 360 12
90 3
c P ( A S) = 
=
210 7
26 a b i P (R ∩ G ) = 0.4 × 0.9 = 0.36
Grows 
red  yellow
grows
 grows

0.9 ii P (G ) = 0.4 × 0.9 + 0.6 × 0.8 = 0.36 + 0.48 = 0.84
P (R ∩ G ) 0.36 3
0.4 Red iii P (R G ) = = = ≈ 0.429
0.1 Does P (G ) 0.84 7
not grow

0.8 Grows
0.6
Yellow

0.2 Does
not grow

5
Chapter 12

27 a There are six favourable outcomes, that is, the same number appears on both dice. There are 36 possible outcomes in
6 1
total, so: P (E ) = = .
36 6
3 1
b There are three possible pairs for a sum of 10: (4, 6), (5, 5) and (6, 4). Therefore, P (F ) = = .
36 12
c To find the probability of the union, we need to apply the addition formula. Note that the events E and F have one
common pair, that is, (5, 5).
1 1 1 6 + 3−1 8 2
P (E ∪ F ) = P (E ) + P (F ) − P (E ∩ F ) = + − = = =
6 12 36 36 36 9
80 8
28 a i P ( A) = =
210 21
35 1
ii P ( A ∩ B ) = =
210 6
100 10
iii To confirm whether the events are independent we firstly find: P (B ) = = . Now we look at the product of
210 21
8 10 80 1
the probabilities: P ( A) × P (B ) = × = ≠ = P ( A ∩ B ) . Hence, they are dependent.
21 21 441 6
50 10
b P (Y1 H ) = =
85 17
c We can select a student from year 1 and a student from year 2, or vice versa; therefore,
110 100 200
P (C ) = 2 × × = .
210 209 19 399
29 Let G be the event that a green ball is chosen and B a blue ball is chosen. If a blue and a green ball have to be selected in
any order, we can say that the chosen balls can be blue and green or green and blue.
3 4 4 3 1 1
P (BG ) + P (GB ) = P (B ) × P (G ) + P (G ) × P (B ) = × + × = 2× =
93 82 93 82 6 3
30 Let S be the event that the student speaks Spanish as his/her first language, and A that the student is Argentinean.
15 12 4
P (S ) × P ( A S ) × 4
P (S A) = = 21 15 7
= =
P ( A) 15 12 6 3 5 5
× + ×
21 15 21 6 7
31 Let D be the event that a patient has the disease, and T that the patient tests positive.
P (D ) × P (T D ) 0.0001 × 0.99
P (D T ) = = ≈ 0.00198
P (T ) 0.0001 × 0.99 + 0.9999 × 0.05
32 Let H be the event that team A will play against the higher-ranked team, and W that team A win the game.
3 6 19
P (W ) = P (H ) × P (W H ) + P (H ') × P (W H ') = × 0.4 + × 0.75 = ≈ 0.633
9 9 300
33 P ( A ∩ B ) + P ( A ∩ B ') = P ( A) ⇒ P ( A) = 0.3 + 0.3 = 0.6. Since the events are independent, we can use the multiplication

law.
P ( A ∩ B ) 0.3
P ( A ∩ B ) = P ( A) × P (B ) ⇒ P (B ) = = = 0.5
P ( A) 0.6
P ( A ∪ B ) = P ( A) + P (B ) − P ( A ∩ B ) = 0.6 + 0.5 − 0.3 = 0.8

34 Let R be the event that it is raining, and L that the girl is late.
1 2 1
P ( R ) × P (L R ) × 10
4 3 6
P (R L) = = = =
P (L) 1 2
× + ×
3 1 19 19
4 3 4 5 60 10

6
2 1 1
35 a P (Y ∩ X ) = P ( X ) × P (Y X ) = × =
3 4 6
1 1 1
P (Y ∩ X ') = P ( X ') × P (Y X ') = × =
3 4 12
1 1 1 1 3
P (Y ) = P (Y ∩ X ) + P (Y ∩ X ') = + = ⇒ P (Y' ) = 1 − P (Y ) = 1 − =
6 12 4 4 4
1 5
b P ( X '∪ Y ') = P (( X ∩ Y ) ') = 1 − P ( X ∩ Y ) = 1 − =
6 6
36 Let F be the event that the umbrella is left in the first shop, and S that the umbrella is left in the second shop.
2 1 2
× 2
9
P (S F ∪ S ) = 3 3 = =
1 2 1 5 5
+ ×
3 3 3 9

5 1 5
37 a i P ( A1) = × =
6 6 36
5 5 1 25
ii P ( A2 ) = × × =
6 6 6 216
n−1 n−1 2 n− 2
5 5 1 1 25 1 5
iii P ( An ) =  ×  × =   =  
 6 6 6 6  36  6  6
b We have an infinite geometric series: p
 
1 25  1 25 1  25  1  25  1 
2 3 2 3
1 25 1  25  1  25  1 1 25
p= + × +  +  + ... = +  + × +  +  + .... ⇒ p = + ×p
6 36 6  36  6  36  6 6 36  6 36 6  36  6  36  6  6 36
c Firstly, we will calculate the probability that Ann wins the game:
1 25 25 1 11 1 1 36 6 6
p= + ×p⇒p− p= ⇒ p= ⇒p= × ⇒p=
6 36 36 6 36 6 6 11 11
6 5
The probability that Bridget wins the game is complementary to the above event, so: P (Bridget wins) = 1 − =
11 11
d If Ann wins more games than Bridget, that means that she has to win 4, 5 or 6 times. We also need to find the number
of sequences with that number of wins. For example, if Ann wins four out of six games played, she can do that in
‘6 choose 4’ different ways (we are using the binomial coefficients).
Ann wins Bridget wins Ann wins Bridget wins Ann wins
4 times twice 5 times once 6 times
    
 6   6 4  5 2  6   6 5  5  6
 6  ≈ 0.432
P (D ) =       +    
  11  11  +  
 4   11  11  5  11

38 a If the first two selected apples are green, then 22 red apples and 1 green apple remain in the box. Therefore, the
22
probability that the next apple will be red is ≈ 0.957.
23
b There are three different selections that will give the result of exactly two red apples: RRG, RGR or GRR. So, the
22 11 21 3 693
probability is calculated as follows: P (B ) = 3 × × × = ≈ 0.301
25 24 4 23 2300
39 Let R be the event that it rains during the day, and T that the daily maximum temperature exceeds 25 °C.

P (T ) = P (R) × P (T R) + P (R ') × P (T R ') = 0.2 × 0.3 + 0.8 × 0.6 = 0.54


P (R ) × P (T R ) 0.06 1
P (R T ) = = = ≈ 0.111
P (T ) 0.54 9

7
Chapter 12

n
40 a The number of integers up to the number n that are divisible by p is given by   , which denotes the greatest
p
n
integer of the number . The number of multiples of 4 from the first 1000 numbers is 250; therefore, the probability
p
250 1
that we select one such number is: P ( A) = = .
1000 4
b The number of integers that are divisible by both 4 and 6 is actually the number of integers that are divisible by
1000 
their least common multiple, that is, 12:  = 83.3333...  = 83. The probability that we select one such number
 12 
83
is: P (B ) = = 0.083 .
1000
41 a Given that the events A and B are independent, we can use the multiplication law; therefore, P ( A ∩ B ) = P ( A) × P (B ) .
In combination with the addition law, we can calculate:

P ( A ∪ B ) = P ( A) + P (B ) − P ( A ∩ B ) = P ( A) + P (B ) − P ( A) × P (B )
0.48
0.88 = 0.4 + P (B ) − 0.4 × P (B ) ⇒ 0.48 = 0.6P (B ) ⇒ P (B ) = = 0.8
0.6
b This part can be calculated in two different ways:

Method I: P ( A ∪ B ) − P ( A ∩ B ) = 0.88 − 0.4 × 0.8 = 0.88 − 0.32 = 0.56

Method II: P ( A ∩ B ') + P ( A '∩ B ) = P ( A) × P (B ') + P ( A ') × P (B ) = 0.4 × 0.2 − 0..6 × 0.8 = 0.56

42 Let M be the event that the chosen day is Monday, and T that Robert catches the 08:00 train.

a P (T ) = P (M ) × P (T M ) + P (M ') × P (T M ') = 0.2 × 0.66 + 0.8 × 0.75 = 0.732


P (M ) × P (T M ) 0.132 11
b P (M T ) = = = ≈ 0.180
P (T ) 0.732 61
4 2
43 a P ( A) = =
6 3
2 4 2
b P (B ) = × =
6 6 9
c We are going to have an infinite geometric series:
2  2
2 3 2 3
2 1 1 2 1 1 2 1 1 2 1 1 1 1 2 93 3
P (C ) = + × × +  ×  × +  ×  × + ... =  1 + +   +   + ... = × = × =
3 3 3 3  3 3 3  3 3 3 3  9  9  9  3 1− 1 3 8 4 4
9
2 1 1
44 a P ( A) = × =
5 4 2 10
4 3 12 12 2
b P (B ) = × = ⇒ = ⇒ (n + 4) (n + 3) = 90 ⇒
n + 4 n + 3 (n + 4) (n + 3) (n + 4) (n + 3) 15
n2 + 7n − 78 = 0 ⇒ (n + 13) (n − 6) = 0 ⇒ n = −13 or n = 6. We can discard the negative solution since the number
of balls cannot be negative.
 Two red Bag B Two
Bag A
red
1 1 2 2 3 + 8 11
c P (C ) = × + × = =
3 10 3 15 90 90
d Let C be the event that two red balls are drawn, and X that the balls are drawn from bag A.
1
P ( X ) × P (C X ) 30 3
P (X C ) = = =
P (C ) 11 11
90 3

8
45 The first fact stated in the problem gives us the following: ( A ∪ B ) ' = ∅ ⇒ P ( A ∪ B ) = 1 .
6 1
P ( A ∪ B ) = P ( A) + P (B ) − P ( A ∩ B ) ⇒ 1 = + P (B ) − P ( A ∩ B ) ⇒ P ( A ∩ B ) = P (B ) −
7 7
P ( A '∩ B ) 1
P ( A ' B) = ⇒ P ( A '∩ B ) = P (B )
P (B ) 3
1 1 1 1 3
P (B ) = P ( A ∩ B ) + P ( A '∩ B ) = P (B ) − + P (B ) ⇒ P (B ) = ⇒ P (B ) =
7 3 3 7 7

9
Chapter 13
Practice questions

1 For f ( x ) = x 2 we have:

a f ′( x ) = 2x ⇒ f ′(1.5) = 3 is the slope

b f (1.5) = 1.5 = 2.25 ⇒ P (1.5, 2.25)


2

y − 2.25 = 3( x − 1.5) ⇒ y = 3x − 2.25


c 5

y
4

2 P(1.5,2.25)

-4 -2 O 2 4
x
-1

-2

-3

d y = 0 ⇒ 0 = 3x − 2.25 ⇒ x = 0.75 ⇒ Q( 0.75, 0 )


x = 0 ⇒ y = −2.25 ⇒ R( 0, −2.25)
e For P(1.5, 2.25) and R( 0, −2.25), the midpoint has coordinates:
1.5 + 0 2.25 − 2.25
x midpt = = 0.75 = x Q , ymidpt = = 0 = yQ
2 2
Hence, Q is the midpoint of PR.

f f ′( a ) = 2a ⇒ y − a 2 = 2a( x − a ) ⇒ y = 2a x − a 2

g y = 0 ⇒ 0 = 2a x − a2 ⇒ x = ⇒ T  , 0
a a
2 2 
x = 0 ⇒ y = − a 2 ⇒ U( 0 , − a 2 )
h For S( a , a2 ) and U( 0, − a 2 ), the coordinates of the midpoint are:
a+0 a a2 − a2
x MSU = = = xT , y MSU = = 0 = yT
2 2 2
Hence, T is the midpoint of SU.

1
Chapter 13

C
2 For y = Ax + B + , x ∈ , x ≠ 0 :
x
f (1) = A + B + C = 4
f ( −1) = − A + B − C = 0
C
f ′( x ) = A − 2
x
f ′(1) = A − C = 0, f ′( −1) = A − C = 0, since (1, 4 ) and ( −1, 0 ) are points of extrema.

The system of equations is:


A + B + C = 4

− A + B − C = 0 ⇒ A = 1, B = 2, C = 1
A − C = 0

d d
3 a
dx
( x 2 ( 2 − 3x 3 )) =
dx
(2x 2 − 3x 5 ) = 4 x − 15x 4
d  1 d 1
b
dx
  =
x dx
( x −1) = − x −2 = − 2
x
8
4 For f ( x ) = + 2x , x > 0, we have:
x
8
a f ′( x ) = − 2 + 2 = 0 ⇒ −8 + 2x 2 = 0 ⇒ x = 2, x = −2
x
f ′(1) = −8 + 2 = −6 < 0 ⇒ f ( x )  on ( 0 , 2 )
8 10
f ′( 3) = − +2= > 0 ⇒ f ( x )  on ( 0 , ∞ )
9 9
8 25
f ( 2)) =+ 2⋅2 = 8 y
2 r(x)=8/x+2x
Hence, point (2, 8) is the turning point (absolute minimum).
20

b For x = 0 the function is not defined, so x = 0 is the


vertical asymptote.
15
8
For x → ∞ , → 0 ⇒ f ( x ) → 2x
x
Oblique asymptote is y = 2x. 10

(2,8)
5

s(x)=2x

O 5 10 x 15

1 dy 1 1
d y y = 4 x 2 +1 we have:3
5 For = 8 x −1 = 0 ⇒ 8 x 3 = 1⇒ x =
= 8 x − 2 x= 0 ⇒ 8 x =d1x⇒ x = x 2 2
dx x 2 2
1 1 1
1For x = : y = 4 ⋅   +
2
1  1  = 3.
For x = : y = 4 ⋅   + = 3. 2  2 1
2  2 1
2
2
1 
So, the stationary point is  , 3 .
2 

2
d y y = a x23 − 2x 2 − x + 7 we have: d y = 3a x 2 − 4 x − 1
6 For = 3a x − 4 x − 1 dx
dx
At x = 2 ⇒ 3a ⋅ 4 − 4 ⋅ 2 − 1 = 3 ⇒ a = x1 = 2 ⇒ 3a ⋅ 4 − 4 ⋅ 2 − 1 = 3 ⇒ a = 1
At

7 Given f ( 2) = 3 and f ′( 2) = 5 :

a The equation of the tangent at (2, 3) is: y − 3 = 5( x − 2) ⇒ y = 5x − 7 .


1 1 1 17
b The normal has a slope of − , so the equation of the normal at (2, 3) is: y − 3 = − ( x − 2) ⇒ y = − x + .
5 5 5 5
8 a g( x ) has a maximum at x = 1 because g′( x ) is positive before it, and negative after it.

b The value of g(x) is decreasing when g′( x ) < 0 , i.e. x ∈ ( −3, −2) ∪ (1, 3).
1  1
c g( x ) has a point of inflexion at x = − because g′′  −  = 0.
2  2
d 3

y
2

maximum at x=1
1
inflexion
at x =-0.5

-3 -2 O 2
x

-1

minimum at x =-2

-2

-3

-4

9 For f ( x ) = x 2 − 3b x + ( c + 2) we have:
f (1) = 1 − 3b + c + 2 = 0 ⇒ 3b − c = 3
f ′( x ) = 2x − 3b
f ′( 3) = 6 − 3b = 0
The system of equations is:
3b − c = 3
 ⇒ b = 2, c = 3
6 − 3b = 0

10 Function Derivative diagram Reason

f1 d f ( x )  ⇒ f ′ < 0, then f ( x )  ⇒ f ′ > 0

f2 e f ( x )  ⇒ f ′ < 0, f ( x ) has a point of


inflexion at x = 0 ⇒ f ′ extreme at x = 0

f3 b where f ( x ) has extremes f ′( x ) = 0

f4 a f (x)  ⇒ f ′ < 0

3
Chapter 13

11 For f ( x ) = 1 + sin x :
π  1 + sin π  − (1 + sin 0)
f   − f (0)  
 2 2 1 2
a The average rate of change is: = = =
π π π π
−0
2 2 2

π π 2
b The instantaneous rate of change: f ′( x ) = cos x ⇒ f ′   = cos   =
 4  4 2
2 π 2  2
c f ′( x ) = , 0 < x < ⇒ cos x = ⇒ x = arccos   ≈ 0.881
π 2 π  π

3x − 2 2
12 For y = = 3 − we have:
x x
a i The vertical asymptote is x = 0 because the function is not defined there.

ii The horizontal asymptote is y = 3 because for x → ±∞ : y → 3.


dy 2
b = −2( −1)x −2 = 2
dx x
dy
c > 0 for the whole domain, so y is increasing for x ∈ ( −∞ , 0 ) ∪ ( 0, ∞ ).
dx
dy
d ≠ 0 for the whole domain, so there are no stationary points.
dx

13 For h( x ) = 2x 2 − x 4 we have:
h′( x ) = 4 x − 4 x 3 = 4 x (1 − x 2 ) = 4 x (1 + x )(1 − x )
h′( x ) = 0 ⇒ x = −1, x = 0, x = 1
h( −1) = 1, h( 0 ) = 0, h(1) = 1

Since the function is continuous, three stationary points, ( −1, 1), ( 0, 0 ), (1, 1), mean that there are four intervals that need to
be tested:
h′ (−2) = 4 ⋅ (−2) − 4 ⋅ (−2) = 24 > 0 ⇒ h( x )  on ( −∞ , −1)
3

3
1 1 1 3
h′  −  = 4 ⋅  −  − 4 ⋅  −  = − < 0 ⇒ h( x )  on ( −1, 0 )
 2  2  2 2
3
 1  1  1 3
h′   = 4 ⋅   − 4 ⋅   = > 0 ⇒ h( x )  on ( 0, 1)
 2  2  2 2
h′ (2) = 4 ⋅ 2 − 4 ⋅ 23 = −24 < 0 ⇒ h( x )  on (1, ∞ )
Therefore, the function has maximum points at ( −1, 1) and (1, 1), and a minimum point at (0, 0).
1 1
14 For the curve y = x 2 + x 3 we have:
d y 1 − 21 1 − 23
= x + x
dx 2 3
1 1 5
At x = 1: + =
2 3 6
1 6
The slope of the normal is: − =− .
5 5
6
6 6 16
The equation of the normal at (1, 2) is: y − 2 = − ( x − 1) ⇒ y = − x + .
5 5 5

4
The normal intersects the axis at:
6 16 8
y =0⇒0=− x+ ⇒x=
5 5 3
16
x=0⇒ y=
5
8 16
So, a = , b = .
3 5
1 2
15 For the displacement function s(t ) = 10t − t ,t 0 :
2
a The velocity function is v (t ) = s ′(t ) = 10 − t .

When t = 0, v( 0 ) = 10 m/s.

b v (t ) = 0 ⇒ t = 10 seconds
1
c s(10 ) = 10 ⋅ 10 − ⋅ 102 = 50 metres
2
16 For the height function h = h(t ) = 14t − 4.9t 2 , t  0 :

a Velocity: v (t ) = h′(t ) = 14 − 9.8t


Acceleration: a(t ) = v ′(t ) = −9.8

b h′(t ) = 0 ⇒ 14 − 9.8t = 0 ⇒ t ≈ 1.43 seconds


h(1.43) = 14 ⋅ 1.43 − 4.9 ⋅ 1.432 ≈ 10 metres

c v (1.43) = 0 m/s
a(1.43) = −9.8 m/s2

17 For y = x 3 + 12x 2 − x − 12 we have:


dy d2 y
= 3x 2 + 24 x − 1, 2 = 6 x + 24
dx dx
d y
2
= 0 ⇒ 6 x + 24 = 0 ⇒ x = −4
dx2
For x = −4 , y = ( −4 )3 + 12( −4 )2 − ( −4 ) − 12 = 120

The inflexion point is ( −4 , 120 ).

18 For f ( x ) = 2 cos x − 3 we have:


π π 3
a f ′( x ) = −2 sin x , f ′   = −2 sin   = −2 ⋅ =− 3
 3  3 2
 π  π 1
f   = 2 cos   − 3 = 2 ⋅ − 3 = −2
 3  3 2
π   π π 3
Tangent at  , −2 : y + 2 = − 3  x −  ⇒ y = − 3x + −2
3   3  3
π  1  π 3 π 3
b Normal at  , −2 : y + 2 = x − ⇒y= x− −2
3  3 3 3 9

27 − r 2
19 a Surface area: S = 2r π( r + h) = 54 π ⇒ r 2 + rh = 27 ⇒ h =
r
27 − r 2
Volume: V = V ( h) = r 2 π h = r 2 π ⋅ = r π( 27 − r 2 ) = π( 27r − r 3 )
r

5
Chapter 13

b V ′( h) = π( 27 − 3r 2 ) = 0 ⇒ ( 3 − r )( 3 + r ) = 0 ⇒ r = 3, r = −3
The volume is a maximum for r = 3 cm.

20 For y = a x 2 + b x + c :

Passes through ( 2, 18 ) ⇒ 4 a + 2b + c = 18

Passes through ( 0, 10 ) ⇒ c = 10
dy dy
Maximum at x = 2 : = 0 and = 2a x + b ⇒ 4 a + b = 0
dx dx
The system of equations:
4 a + 2b + c = 18
 4 a + 2b = 8
4 a + b = 0 ⇒ ⇒ b = 8, a = −2
c = 10 4 a + b = 0

The function is y = −2x 2 + 8 x + 10.

1
21 For the function f ( x ) = x 2 − 5x + 3 :
2
1
a f ( −2) = ( −2)2 − 5 ⋅ ( −2) + 3 = 15, f ′( x ) = x − 5, f ′( −2) = −2 − 5 = −7
2
Tangent at ( −2, 15) : y − 15 = −7( x + 2) ⇒ y = −7 x + 1
1 1 107
b Normal at ( −2, 15) : y − 15 = ( x + 2) ⇒ y = x +
7 7 7
22 For f ( x ) = x 4 − x 3 we have:
3
a f ′( x ) = 4 x 3 − 3x 2 = 0 ⇒ x 2 ( 4 x − 3) = 0 ⇒ x = 0, x =
4
f ′( −1) = 4( −1)3 − 3( −1)2 = −7 < 0 ⇒ f ( x )  on (−∞ , 0)
3 2
 1 1 1 1 3
f ′   = 4   − 3   = − < 0 ⇒ f ( x )  on  0, 
 2  2  2 4  4
3 
f ′(1) = 4 − 3 = 1 > 0 ⇒ f ( x )  on  , ∞
4 
4 3
 3  3  3 27
f   =   −  = −
 4  4  4 256
3 27 
Absolute minimum at  , − .
4 256 
 27 
b Domain is  and the range is  − ,∞ .
 256 
1
c f ′′( x ) = 12x 2 − 6 x = 0 ⇒ 6 x ( 2x − 1) = 0 ⇒ x = 0, x =
2
4 3
1 1 1 1
f ( 0 ) = 0, f   =   −   = −
 2  2  2 16
1 1
The inflexion points are (0, 0) and  , −  .
 2 16 

6
d
2

f(x) = x4- x3
y

1.5

inflexion
0.5

-1 O 1 x 2
(0.5,-0.625)
-0.5

minimum
(0.75,-0.1055)
-1

2 − 3x + 5x 2 2 3
2 − 3x + 5x 2 − +5 0−0+5 5
23 a lim = lim x 2
= lim x 2
x = =−
x →∞ 8 − 3x 2 x →∞ 8 − 3x 2 x →∞ 8 0 − 3 3
−3
x2 x2

x+4 −2 x+4 −2 x+4 +2 x+4−4


b lim = lim ⋅ = lim
x →0 x x →0 x x + 4 + 2 x →0 x ( x + 4 + 2 )
x 1 1 1
= lim = lim = =
x →0 x ( x + 4 + 2 ) x →0 ( x + 4 + 2 ) 0+4 +2 4

x3 − 1 ( x − 1) ( x 2 + x + 1)
c lim = lim = lim( x 2 + x + 1) = 1 + 1 + 1 = 3
x →1 x − 1 x →1 x −1 x →1

( x + h) + 2 − x + 2 ( x + h) + 2 − x + 2 ( x + h) + 2 + x + 2
d lim = lim ⋅
h→0 h h → 0 h ( x + h) + 2 + x + 2

= lim
[( x + h) + 2] − [x + 2] = lim h
=
1
=
1
h→0 h ( ( x + h) + 2 + x + 2 )
h→0 h ( ( x + h) + 2 + x + 2 ) ( x + 0) + 2 + x + 2 2 x + 2

x2 − 4x 3 1

24 a f ( x ) = = x 2 − 4x 2
x
3 21 1 −1 3 x 2 3x − 4
f ′( x ) = x − 4 ⋅ x 2 = − =
2 2 2 x 2 x

b f ( x ) = x − 3 sin x
3

f ′( x ) = 3x 2 − 3 cos x
1 x x
c f (x) = + = x −1 +
x 2 2
−2 1 1 1
f ′( x ) = ( −1)x + = − 2 +
2 x 2
7 7
d f (x) = = x −13
3x 13 3
7 91
f ′( x ) = ⋅ ( −13)x −14 = − 14
3 3x

7
Chapter 13

dy
25 For the curve y = x 3 + x 2 − 9 x − 9, we have = 3x 2 + 2 x − 9 .
dx
The slope of the tangent at ( p , q ) is 3p2 + 2 p − 9 .

As (p, q) is on the curve, q = p 3 + p 2 − 9 p − 9 .

The equation of the tangent at (p, q) is y − ( p3 + p2 − 9 p − 9 ) = ( 3p2 + 2 p − 9 )( x − p ) .

The point ( 4 , −1) is on the tangent, so:


−1 − ( p 3 + p 2 − 9 p − 9 ) = ( 3p 2 + 2 p − 9 )( 4 − p ) ⇒ −1 − p 3 − p 2 + 9 p + 9 = 12 p2 + 8 p − 36 − 3p3 − 2 p2 + 9 p ⇒
2 p3 − 11p2 − 8 p + 44 = 0 ⇒ 2 p3 − 4 p2 − 7 p2 + 14 p − 22 p + 44 = 0 ⇒
2 p2 ( p − 2) − 7 p( p − 2) − 22( p − 2) = 0 ⇒
( p − 2)( 2 p 2 − 7 p − 22) = 0 ⇒
11
( p − 2)( p + 2)( 2 p − 11) = 0 ⇒ p1 = 2, p2 = −2, p3 =
2
q1 = 23 + 22 − 9 ⋅ 2 − 9 = −15
q2 = ( −2)3 + ( −2)2 − 9( −2) − 9 = 5
3 2
 11  11 11 1105
q3 =   +   − 9 ⋅ − 9 =
 2  2 2 8
 11 1105
There are three solutions for point (p, q): ( 2, −15), ( −2, 5),  , .
2 8 
 1 1 1
26 Let  a , a 3 +  be the point at which the normal y = − x + c , c  0 , intersects the curve y = x 3 + . The slope
 3  12 3
dy 1
function for this curve is = 3x 3, so the slope of the normal at x = a is − 2 . From the equation of the normal, we
dx 3a
1 1 1
can see that the slope is − , so − 2 = − ⇒ a2 = 4 ⇒ a1 = 2, a2 = −2.
12 3a 12
 25 25 1 17
For a1 = 2, the intersection point is  2,  . It is on the normal, so: = − ⋅ 2 + c ⇒ c1 = .
 3 3 12 2
 23 
For a2 = −2, the intersection point is  −2, −  . It is on the normal, so:
 3
23 1 47 17
− = − ⋅ ( −2 ) + c ⇒ c 2 = − . This solution is negative, so the only solution is c = .
3 12 6 2
1 3 dy
27 For the curve y = x − x , the slope function is = x 2 − 1. If the tangent is parallel to the line y = 3x , it should have
3 dx
the same slope, so: x 2 − 1 = 3 ⇒ x 2 = 4 ⇒ x1 = 2, x 2 = −2 .
1 2
For x1 = 2 : y1 = ⋅ 23 − 2 = .
3 3
1 2
For x 2 = −2 : y2 = ⋅ ( −2)3 − ( −2 ) = − .
3 3
2 2
There are two points with tangents parallel to the given line:  2,  and  −2, −  .
 3  3
dy
28 For the curve y = x − x 2 , the slope function is = 1 − 2x .
dx
1
The slope of the normal at (1, 0) is − = 1, so the equation of the normal is: y − 0 = 1⋅ ( x − 1) ⇒ y = x − 1 .
1− 2
To find the intersection points, we have to solve the following system of equations:
y = x − x2
 ⇒ x − x 2 = x − 1 ⇒ x 2 = 1 ⇒ x = ±1
 y = x − 1
We have been given the intersection point (1, 0), so for x = −1 : y = −1 − (−1) = −2.
2

Point ( −1, −2) is the other intersection point.

8
29 For f ( x ) = x + 2 we have:

f ( x + h) − f ( x ) ( x + h) + 2 − x + 2
f ′( x ) = lim = lim
h→0 h h→0 h

= lim
( x + h) + 2 − x + 2 ( x + h) + 2 + x + 2
⋅ = lim
[( x + h) + 2] − [x + 2]
h→0 h (
( x + h) + 2 + x + 2 h→0 h ( x + h) + 2 + x + 2 )
h 1 1
= lim = =
h→0 h ( )
( x + h) + 2 + x + 2 ( x + 0) + 2 + x + 2 2 x + 2

30 For the position function s(t ) = t 3 − 9t 2 + 24t :

a The velocity function:


ds
v (t ) = = 3t 2 − 18t + 24
dt
v (t ) = 0 ⇒ 3t 2 − 18t + 24 = 0 ⇒ t 2 − 6t + 8 = 0 ⇒ (t − 2)(t − 4 ) = 0 ⇒ t = 2, t = 4
s( 2) = 23 − 9 ⋅ 22 + 24 ⋅ 2 = 20
s( 4 ) = 4 3 − 9 ⋅ 4 2 + 24 ⋅ 4 = 16
b The acceleration function:
dv
a(t ) = = 6t − 18
dt
a(t ) = 0 ⇒ 6t − 18 = 0 ⇒ t = 3
s( 3) = 33 − 9 ⋅ 32 + 24 ⋅ 3 = 18

31 For the displacement function s(t ) = t + sin t , 0  t  2π :

a The velocity function is v (t ) = s ′(t ) = 1 + cos t .

Since −1  cos t  1 ⇒ v (t )  0 , which means that the particle does not change direction.

b For 0  t  π : sin t  0 ⇒ t + sin t  0

For π  t  2π : −1  sin t  0 ⇒ t + sin t  π − 1  0

So, s(t )  0 for 0  t  2π , which means that the particle is always on the same side of the origin.

c The acceleration function is a(t ) = v ′(t ) = − sin t .


a(t ) = 0 ⇒ sin t = 0 ⇒ t = 0, t = π , t = 2π

d 7
maximum
s(t)
(2 π,2π)
6

O 1 2 3 4 5 6 t
2 4 6 8

9
Chapter 13

dy d2 y
32 For the curve y = a x 3 + b x 2 + c x + d : = 3a x 2 + 2b x + c , 2 = 6a x + 2b
dx dx
d2 y
Inflexion when x = −1 : = 0 ⇒ 6a( −1) + 2b = 0 ⇒ −6a + 2b = 0
dx2
dy
Turning point when x = 2 : = 0 ⇒ 3a ⋅ 22 + 2b ⋅ 2 + c = 0 ⇒ 12a + 4b + c = 0
dx
Passes through ( 3, −7 ) : a ⋅ 33 + b ⋅ 32 + c ⋅ 3 + d = −7 ⇒ 27 a + 9b + 3c + d = −7

Passes through ( −1, 4 ) : a( −1)3 + b( −1)2 + c ⋅ ( −1) + d = 4 ⇒ − a + b − c + d = 4

We have the following system of linear equations:

27a + 9b + 3c + d = −7
− a + b − c + d = 4


12a + 4b + c = 0
−3a + b = 0

The system can be easily solved with a GDC:

1 3 3 2 5 1 3 5 19
So, y = x + x − 6 x − , and, at x = 2, y = ⋅ 23 + ⋅ 22 − 6 ⋅ 2 − = − .
4 4 2 4 4 2 2

9 18
33 For the function f ( x ) = 1 − + = 1 − 9 x −2 + 18 x −4 :
x2 x4
18( x 2 − 4 ) 18( x + 2)( x − 2)
f ′( x ) = 18 x −3 − 72x −5 = = = 0 ⇒ x = −2, x = 2
x5 x5
Since the curve is not defined for x = 0 (vertical asymptote), we have four intervals to check:
18 (( −3)2 − 4) 10
f ′( −3) = =− < 0 ⇒ f ( x )  on ( −∞ , −2)
( −3)5 27
18 (( −1)2 − 4)
f ′( −1) = = 54 > 0 ⇒ f ( x )  on ( −2, 0 )
( −1)5
18 (12 − 4)
f ′(1) = = −54 < 0 ⇒ f ( x )  on ( 0, 2)
15
18 (32 − 4) 10
f ′( 3) = = > 0 ⇒ f ( x )  on ( 2, ∞ )
35 27
And because
9 18
lim f ( x ) = lim 1 − + = 1− 0 + 0 = 1
x →±∞ x →±∞ x2 x4
9 18 1
f ( −2 ) = 1 − + =−
( −2 ) ( −2 )
2 4
8
9 18 1
f ( 2) = 1 − 2 + 4 = −
2 2 8
1 1
We conclude that both stationary points,  −2, −  and  2, −  , are absolute minima.
 8  8

10
1 dy 1
34 a For the curve y = = x −1, = − x −2 = − 2 , so the slope of the tangent at (1, 1) is −1, and the equation of the
x dx x
tangent is: y − 1 = −1⋅ ( x − 1) ⇒ y = − x + 2 .

dy π   π
b For the curve y = cos x , = − sin x , so the slope of the tangent at  , 0 is − sin   = −1, and the equation of
dx 2   2
π π
the tangent is: y − 0 = −1⋅  x −  ⇒ y = − x + .
 2 2
π π 1
c Since 2 > ⇒ − x + 2 > − x + , all the tangents on y = are above the tangents on y = cos x , which means
2 2 x
1 π
that > cos x , 0  x  .
x 2
dy
35 For the curve y = x 3 − x + 2, the slope function is = 3x 2 − 1 .
dx
If the point ( a , a3 − a + 2) is a point of tangency, then the equation of the tangent is: y − a3 + a − 2 = ( 3a2 − 1)( x − a ).

The tangent should pass through the origin, so: 0 − a3 + a − 2 = ( 3a 2 − 1)( 0 − a ) ⇒ − a 3 + a − 2 = −3a 3 + a ⇒ a 3 = 1 .

This gives us only one solution: a = 1.

The tangent is: y − 13 + 1 − 2 = ( 3 ⋅ 12 − 1)( x − 1) ⇒ y = 2x at (1, 2) .

36 For the displacement function s = s (t ) = 50t − 10t 2 + 1000 :


ds
a Velocity: v (t ) = = 50 − 20t
dt 2
5  5  5  5
b v (t ) = 0 ⇒ 50 − 20t = 0 ⇒ t = , so the maximum displacement is: s   = 50 ⋅   − 10 ⋅   + 1000 = 1062.5 m
2  2  2  2

37 25

y
20

maximum
15
f ′(x)

10

f(x)
5

-4 -2 O b
a minimum x

-5

inflexion
-10
points

11
Chapter 14
Practice questions

      


1 a CD = CO + OD = −OC + OD = OD − OC
 1  1   1  
b OA = CD = − (−OC + OD ) = (OD − OC )
 2  2   2 1    1  1  1  
c AD = − AO + OD = − OA + OD = − (OD − OC ) + OD = OD + OC = (OD + OC )
2 2 2 2
2 a u + 2v = (−i + 2 j) + 2 (3i + 5 j) = −i + 2 j + 6i + 10 j = 5i + 12 j
1 1
b The unit vector in the direction of u + 2v is: (5i + 12 j) = (5i + 12 j) . So,
5 + 12
2 2 13
1
w = 26 ⋅ (5i + 12 j) = 2 (5i + 12 j) = 10i + 24 j.
13

3 a OA = 62 + 02 = 6, so A lies on the circle.

OB = (−6) + 02 = 6 , so B lies on the circle.
2

 2
OC = 52 + 11 = 6, so C lies on the circle.
      5   6   −1 
b AC = AO + OC = OC − OA =   −  = 
 11   0   11 
c Method I: Using a scalar product
 −6   −1 
    
 AO ⋅ AC  0   11  6 1 1 3
cos OAC =   = = = = =
AO AC 6 (−1)2 + 112 6 12 12 2 3 6

Method II: Using a cosine rule in triangle OAC


 −1 
In triangle OAC, SSS is given: OA = OC = 6, and AC =   = 12; hence,
 11 
( )
2
 6 + 12 − 6
2 2
12 3
cos OAC = = =
2 ⋅ 12 2 12 6
d Method I: Using the result from c

Using the Pythagorean identity for sine, sin2 θ = 1 − cos2 θ , and the fact that sine is positive for angles from
2
  3 1 11
0 − 180 we have: sin OAC = 1 −   = 1 −

= . Hence,
 
6 12 12
1  1 11
A = AB AC sin A = 12 ⋅ 12 = 6 11.
2 2 12
Method II: Finding the area using side and height dimensions

In triangle ABC , side AB = 12; the height on this side is the second coordinate of point C, so:
1
A = 12 ⋅ 11 = 6 11 .
2
Method III: Using the triangle in half circle property
1 1
The triangle ABC is a right triangle, with right angle in C. So: A = AC AB = 11 ⋅ 12 = 6 11 .
2 2

1
Chapter 14

  10    2 − 5   −3 
4 a OB =  , AC =  =
 5   7 − 1   6 
b The diagonals of quadrilateral OABC are OB and AC . So, the angle between the diagonals is the same as the angle
 
between vectors OB and AC .
 
OB ⋅ AB = 10 ⋅ (−3) + 5 ⋅ (6) = 0; hence, the diagonals are perpendicular, 90°.

5 u + v = 4i + 3 j
 4a = 8 a=2
Then, a ( 4i + 3 j) = 8i + (b − 2) j ⇒  ⇒
 3a = b − 2 6+2=b⇒b=8

 3 − (−1)   4   −1   4 
6 The direction vector is   =  −5  ; hence, the equation of the line is: r =  4  + t  −5  .
 −1 − 4   
 −4   3 
Note: For the direction vector we can use  , and for the initial point  . So, the equation of the line can be any
 5   −1 
of these combinations.

 18 
7 a The speed of the Toyundai:  = 182 + 24 2 = 30 km/h
 24 
 36 
= 362 + (−16) = 1552 ≈ 39.4 km/h
2
The speed of the Chryssault: 
 −16 
1  18   9  1  36   18 
b i After half an hour, the vehicles have covered half the distance:  = ; =
2  24   12  2  −16   −8 
 9 − 18   −9 
ii The vector joining their positions at 06:30 is   =  ; hence, the distance between the
 12 − (−8)   20 
 −9 
vehicles is:  = 92 + 202 = 481 ≈ 21.9 km.
 20 
 18 
c The Toyundai must continue until its position vector is  , so until k = 24. At that point, its position is
 k 
 18 
 24  . To reach this position, it must travel for a total of one hour. Hence, the crew start work at 07:00.

d The southern (Chryssault) crew lay: 800 ⋅ 5 = 4000 m of cable.

The northern (Toyundai) crew lay: 800 ⋅ 4.5 = 3600 m of cable.

Their starting points were 24 − (−8) = 32 km apart; hence, they are now 32 − 3.6 − 4 = 24.4 km apart.

 18   18 
e The position vector of the Toyundai at 11:30 is  = .
 24 − 3.6   20.4 
 18 
The distance to base camp is:  = 182 + 20.4 2 = 740.16 ≈ 27.2 km.
 20.4 

27.2
The time needed to cover this distance is: ⋅ 60 = 54.4 ≈ 54 minutes.
30
     0  2
8 The line passes through point  0  and has a direction vector  2  . Hence, its equation is: r =   + t   , or
 2  0  3  0   3
r=t .
 3

2
9 a
y
C

0 x

b Point C has the same x-coordinate as A; hence, C (2, y ) . We find the y-coordinate by using the fact that
    2 − 5   −3 
AB and BC are perpendicular. From the graph, point B is (5, 1). So: BC =   =  and:
 y −1  y −1
 2 
   3   −3  13  13 
 .
0 = AB ⋅ BC =    y − 1  = −9 + 4 y − 4 = −13 + 4 y ⇒ y = 4 . Hence, C  2, 4  . So, OC =  13
 4   
 4 
 16   16 
10 a i Initially, Air One is at position r =  ; hence, its distance from the origin is:  = 162 + 122 = 20 km.
 12   12 
 12   12 
= 122 + (−5) = 13 km/min.
2
ii The velocity vector is v =  ; hence, its speed is: 
 −5   −5 

 16   12   x   16 + 12t   x = 16 + 12t
b r=  +t  ⇒  =  ⇒
 12   −5  y
   12 − 5t  y = 12 − 5t

x − 16
From the first equation, we have t = . Substituting into the second equation:
12
x − 16 144 − 5x + 80
y = 12 − 5 = ⇒ 12 y = 224 − 5x ⇒ 5x + 12 y = 224.
12 12
Note: If we multiply the vector equation of the line by the vector perpendicular to the direction vector, we can find
the result quite quickly.
 x   16   12   5   x   5   16   5   12 
 y  =  12  + t  −5  ⇒  12   y  =  12   12  + t  12   −5  . Now we have:
              
5x + 12 y = 5 ⋅ 16 + 12 ⋅ 12 + t ⋅ 0 ⇒ 5x + 12 y = 224.
c We have to determine the angle between the direction vectors:
 12   2.5 
 −5   6  = 12 ⋅ 2.5 − 5 ⋅ 6 = 0; hence, the angle between the paths of the two aircraft is 90 .

 x − 23
 23   2.5   x   23 + 2.5t   x = 23 + 2.5t ⇒ t =
 2.5
d i r= +t ⇒  = ⇒
 −5   6   y   −5 + 6t   y = −5 + 6t ⇒ t = y + 5
x − 23 y + 5  6
Hence, = .
2.5 6
x − 23 y+5
Multiplying by 30: 30 = 30 ⇒ 12 ( x − 23) = 5 ( y + 5) ⇒ 12x − 5 y = 301
2.5 6
Note: We could also have used the method from b.

5x + 12 y = 224 25x + 60 y = 1120 12 ⋅ 28 − 301


ii  ⇒ ⇒ 169 x = 4732 ⇒ 28, y = =7
12x − 5 y = 301 144 x − 60 y = 3612 5
Hence, the paths cross at the point (28, 7) .

3
Chapter 14

e We will determine the time at which each of the planes is at (28, 7) .

For Air One:


 28   16 + 12t   28 − 16 = 12t
 7  =  12 − 5t  ⇒  7 − 12 = −5t ⇒ t = 1

For Air Two:
 28   23 + 2.5t   28 − 23 = 2.5t
 7  =  −5 + 6t  ⇒  7 + 5 = 6t ⇒t =2

So, the planes are not at the point where the two paths cross at the same time, i.e. the planes do not collide.
 1 6 
 2   −8  6 − 16 −10 −1
11 cos θ = = = =
 1  6  1 + 4 36 + 64 5 ⋅ 10 5
 2   −8 
−1
θ = cos −1 = 116.565...° ≈ 117
5
12 Method I:
 x−4
If ( x , y ) is a point on the line, then the vector  y + 1  is the vector on the line, and it is perpendicular to the vector
 

 2  2 x−4
 3  . Hence, their dot product is zero:  3   y + 1  = 0

 2 x−4
So, 0 =     = 2( x − 4 ) + 3( y + 1) = 2x − 8 + 3 y + 3 = 2x + 3 y − 5 and the equation of the line is:
 3   y +1 
2x + 3 y = 5 .

Method II:
 2  3   −3 
If vector   is perpendicular to the line, then the vector  , or  , is a direction vector of the line.
 3  −2   2 
 4   3 
So, a vector equation of the line is r =   +t . Now, we have to transform the equation into Cartesian
 −1   −2 
 x   4 + 3t  x = 4 + 3t ⋅2 2x = 8 + 6t
form: ⇒   =   ⇒ ⇒ ⇒ 2x + 3 y = 5
y
   −1 − 2t   y = −1 − 2t ⋅3 3 y = −3 − 6t
 x  0   6   6 
13 a At 13:00, t = 1 :   =   + 1⋅  −8  =  20 
 y  28
 x  x  6   0   6 
b i The velocity vector is:   −   =   −  = 
y y
  t =1   t =0  20   28   −8 
 6 
= 62 + (−8) = 10 km/h
2
ii The speed is the magnitude of the velocity vector; therefore: 
 −8 
 x  0   6   6t  x = 6t ⋅4 4 x = 24t
c   =  + t ⋅  −8  =  28 − 8t  ⇒  y = 28 − 8t ⋅3 ⇒ 3 y = 84 − 24t ⇒ 4 x + 3 y = 84
y
   28  

d The two ships will collide if the point (18, 4) is on the line. So:
 18   0   6   18 = 6t
 4  =  28  + t ⋅  −8  ⇒  4 = 28 − 8t ⇒ t = 3

Therefore, the ships will collide at t = 12 + 3 = 15: 00 hours.
 x   18   5   18 + 5t − 5   13 + 5t   13   5 
e   =  + (t − 1) ⋅   =  =  =  +t
y
   4   12   4 + 12t − 12  − 8 + 12t   −8   12 

4
 18   x   13   5   28 
f At t = 3, Aristides is at  and Boadicea is at   =   + 3 ⋅  12  =  28  .
 4  y
   −8
 28   18   10 
Therefore, their distance vector is:  − = ; hence, the ships are 102 + 24 2 = 676 = 26 km apart.
 28   4   24 
ab (−4i − 2 j) (i − 7 j) −4 + 14 10 10 1
14 cos θ = = = = = =
a b −4i − 2 j i − 7 j 16 + 4 1 + 499 20 50 10 2 5 10
1 
θ = cos−1  = 71.565 05... = 72
 10 

 x  2  0.7   3.4 
15 a At t = 2 :   =   + 2 ⋅  =
y
   0  1   2 

The distance from the point (0, 0) is: 3.4 2 + 22 = 15.56 ≈ 3.94 m

 0.7 
b The speed of the car is:  = 0.72 + 1 = 1.49 ≈ 1.22 m/s
 1 
 x  2  0.7   2 + 0.7t 
c  y  =  0  + t ⋅ 1  =   ⇒ t = y ; hence:
⇒ x = 2 + 0.7 y ⇒ x − 0.7 y = 2
       t
d Solve the system of equations:
x − 0.7 y = 2 170
 ⇒ x − 0.42x − 1.4 = 2 ⇒ x = ≈ 5.86
 y = 0. 6 x + 2 29
160
y= = 5.52
29
 170 160 
So, the collision point is  , .
 29 29 
e Since y = t, the time of collision is 5.52 seconds.
 0   5.86 
The distance covered by the motorcycle is:   −  = 5.862 + 3.522 ≈ 6.84 m. Therefore, the speed is the
 2   5.52 
6.84
quotient of distance and time, and we have: ≈ 1.24 m/s.
5.52
 6 −1  5   x    
16 The direction vector is   =   ; hence, the equation of the line is   =  1  + t  5  .
 5−3   2   y   3  2
 x  6  −5 
Note: We can use the other point as the initial point and then we will have the equation   =   + t  .
 y  5  −2 
 2x   x − 1 
17 a  = 0 ⇒ 2x 2 + 2x + 5x − 25 = 0 ⇒ 2x 2 + 7 x − 25 = 0
 x − 5   5 
−7 − 249 −7 − 249
b 2x 2 + 7 x − 25 = 0 ⇒ x1 = ≈ −5.69, x 2 = ≈ 2.19
4 4
IB exam question:
 2x   x + 1 
a  = 0 ⇒ 2x 2 + 2x + 5x − 15 = 0 ⇒ 2x 2 + 7 x − 15 = 0
 x − 3   5 
3
b 2x 2 + 7 x − 15 = 0 ⇒ x1 = −5, x 2 =
2  24 
  240   1  240   25   0.96 
18 a i OA =   ⇒ OA = 2402 + 702 = 250 . So, the unit vector is:   =  =  .
 70  250  70   7   0.28
 0.96   288   25 
ii v = 300  =
 0.28   84 

5
Chapter 14

250 5
iii t=
= hour, or 50 minutes
300 6
  480 − 240   240 
b AB =  =
 250 − 70   180 
 240   240 
 70   180  57 600 + 12 600 70 200
cos θ = = = = 0.936
 240   240  240 + 70
2 2
240 + 180
2 2
250 ⋅ 300
 70   180 
So, θ = cos −1 0.936 = 20.609... ≈ 20.6.
  339 − 240   99 
c i AX =  =
 238 − 70   168 
 −3   240  
ii  4   180  = −3 ⋅ 240 + 4 ⋅ 180 = 0; hence, n ⊥ AB.
 1  99   −3  −297 + 672
iii The scalar projection of AX in the direction of n is  = = 75; hence, the distance
5  168   4  5
XY is 75 km.

d Using Pythagoras’ theorem, we can find the distance from A to Y using the distances AX and XY So,
AX = 992 + 1682 = 38 025 = 195; hence, AY = 1952 − 752 = 32 400 = 180 km.
 x −1
 x   1  −2  x = 1 − 2t ⇒ −2 = t x −1 y − 2
19   =   + t   ⇒ ⇒ =
y
    2  3   y = 2 + 3t ⇒ y − 2 −2 3
=t
 3
3 ( x − 1) = −2 ( y − 2) ⇒ 3x + 2 y = 4 + 3 ⇒ 3x + 2 y = 7
20 y
U
15

10

V T
5

-10 10
x
S
-5

  7 − ( −2)   9     9 


a ST =   =   , and, since STUV is a parallelogram, VU = ST =   .
 7 − ( −2 )   9   9
  9  5   9  5   9  −4 
VU = u − v =   ⇒   −v=  ⇒  −  = v⇒ = v, and the coordinates of V are: (−4 , 6) .
 9  15   9  15   9   6 
 9
b The line contains the point (5, 15) and the direction vector is parallel to   . So, for the direction vector, we can
 9
 1  5   1
use the vector   . So, r =  + λ  .
 1  15   1
 9
Note: We can also use the direction vector   and initial point (−4 , 6) . However, the question directs us to use
 9
point U. This also ensures we avoid any possible mistakes we made when finding the coordinates of V.

6
 1  5   1  1 = 5 + λ ⇒ λ = −4
c  11  =  15  + λ  1  ⇒  11 = 15 + λ ⇒ λ = −4

So, the point is on the line when λ = −4.
  a − 1   a − 1 
d i EW =  =
 17 − 11   6 

EW = (a − 1) + 36 = a2 − 2a + 37 = 2 13 ⇒ a2 − 3a + 37 = 52
2

So: a2 − 2a − 15 = 0 ⇒ a1 = −3, a2 = 5

  −4    7 − 1   6 
ii For a = −3 : EW =  , ET =  =
 6   1 − 11   −4 
 −4   6 
 6   −4  −24 − 244 −48 12  12 
So, cos θ = = = = − , and θ = cos−1  −  = 157.38... ≈ 157.
 −4   6  16 + 36 16 + 36 52 13  13 
  
6   −4  
 4
21 The angle between the lines is the angle between their direction vectors. The direction vector of the first line is   ,
 3
 1 
and the second is  .
 −1 
 4 1 
 3   −1  4−3 1
The angle is: cos θ = = = . So,
 4  1  16 + 9 1 + 1 5 2
 3   −1 
1 
θ = cos−1  = 81.869... ≈ 81.9 .
 5 2 
Note: If the cosine of the angle was negative, then the angle would be obtuse; so, to find the acute angle, we would
have to subtract the angle from 180 . Here, it was not the case.

22 a a = 122 + 52 = 13

b b = 62 + 82 = 10
1
The unit vector in the direction of b is:(6i + 8 j) = 0.6i + 0.8 j.
10
ab 12 ⋅ 6 + 5 ⋅ 8 112 56
c cos θ = = = =
a b 13 ⋅ 10 13 ⋅ 10 65
23 The coordinates of the point of intersection should satisfy both equations. So:
 5 + 3 λ   −2 + 4 t  3 λ − 4t = −7
 1 − 2 λ  =  2 + t  ⇒ −2 λ − t = 1 ⇒ t = −2 λ − 1 ⇒ 3 λ + 8 λ + 4 = −7 ⇒ λ = −1, t = 1

    
Therefore, the position vector of the point is: OP =  5 − 3  =  2  .
 1+ 2   3 
Note: We can transform the vector equations to Cartesian form (2x + 3 y = 13, and x − 4 y = −10) and then solve the
system.
     10   7   3 
24 a OR = PQ = OQ − OP =  − =
 1   3   −2 
 −7   3 
   −3   −2 
 PO PQ −21 + 6 −15
b cos OPQ =   = = =
PO PQ 49 + 9 9 + 4 58 13 754

7
Chapter 14

   
c i Since PQR + OPQ = 180 , cos PQR (
 = cos 180 − OPQ )
= − cos OPQ.

ii Using the Pythagorean identity for sine and the fact that the sine of angles in a triangle is always positive, we
have:
   152 529 23
sin PQR = 1 − cos2 PQR = 1 − cos2 OPQ = 1 − = =
754 754 754
   3  7 23
iii Area of the parallelogram: A = OR OP sin θ =  = 13 53 = 23 units2
 −2   3  13 ⋅ 53
  −1    8 
25 a OB =  , OC =  
 7   9
b To find D, we have to find the vector of the side of the parallelogram:
     8   −1   9 
AD = BC = OC − OB =   −  = . Now we can find the position vector of D:
 9   7   2 
    2   9   11 
OD = OA + AD =   +   =   . Hence, d = 11.
 2  2  4 
  11   −1   12 
c BD =   −  =
 4   7   −3 
 x   −1    −1   12 
d i  y  =  7  + t BD =  7  + t  −3 
       
 4   x   −1   4 
Note: For the direction vector, we can use   . Then the equation would be   =   +t .
 −1   y  7   −1 
 −1   −1   12 
ii At point B, t = 0. We can see that   =  + 0⋅ .
 7   7   −3 
 8
 7   −1   12   7 = −1 + 12t ⇒ t =
 12 2
e   = +t ⇒ ⇒t =
 5   7   −3   5 = 7 − 3t ⇒ t = − 2 3
 −3
    7   8   −1 
f CP = OP − OC =   −   = 
 5   9   −4 
   −1   12   
CP ⋅ BD =     = −12 + 12 = 0; hence, CP ⊥ BD.
 −4   −3 
    −3   2   −5 
26 a i AB = OB − OA =  − =
 −1   −2   1 

ii AB = 25 + 1 = 26

    d   2   d − 2 


b AD = OD − OA =  − =
 23   −2   25 
    
c i BAD = 90 ; hence, AB ⊥ AD ⇒ AB ⋅ AD = 0.
   −5   d − 2 
0 = AB ⋅ AD =  = −5d + 10 + 25 ⇒ d = 7
 1   25 
  d   7 
ii OD =  =
 23   23 

8
   7 − 2   5   5 
d BC = AD =  =
 25   25   25
    −3   5   2 
OC = OB + BC =  + =
 −1   25   24 
 
e The area is the product of the sides: A = AB ⋅ AD = 25 + 1 25 + 625 = 26 650 = 130 units2.
  
27 a i BC = OC − OB = (−5i − 5 j) − (i − 3 j) = −6i − 2 j
    
ii OD = OA + AD = OA + BC = ( 4i + 2 j) + (−6i − 2 j) = −2i
  
b BD = OD − OB = −2i − (i − 3 j) = −3i + 3 j
  
AC = OC − OA = (−5i − 5 j) − ( 4i + 2 j) = −9i − 7 j
 
BD ⋅ AC 27 − 21 6 6
cos θ =   = = = ⇒ θ = 82.8749... ≈ 82.9
BD AC 9 + 9 81 + 49 18 1330 2340
c r = i − 3 j + t (2i + 7 j)

d We have to solve the vector equation: i − 3 j + t (2i + 7 j) = 4i + 2 j + s (i + 4 j) .


1 + 2t = 4 + s
Hence,  ⇒ s = −3 + 2t ⇒ −3 + 7t = 2 − 12 + 8t ⇒ t = 7, s = 11 . So, the position vector of the
−3 + 7t = 2 + 4 s
intersection is: r = i − 3 j + 7 ⋅ (2i + 7 j) = 15i + 46 j.
   
28 a OG = OA + AB + BG = 5i + 5 j + 5k
  
b BD = BC + CD = −5i + 5k

    
c EB = EF + FG + GB = 5i + 5 j − 5k
 3  3  3
1  4  = 1  4  , so the velocity vector of the balloon is:
29 a The unit vector in the direction of  4  is:
  32 + 4 2 + 02  0  5  0 
 0    
 3
18  
4 . Therefore, the equation of the path of the balloon is the same as the equation of the line through (0, 0, 5)
5  
 0  x  0
 3  3
18   18
with direction vector 4 : b =  y  =  0  + t ⋅  4  .
5    z   5  5  
 0  0
b i t = 0 ⇒ ( 49, 32, 0)
 −48   −48 
ii The velocity vector is  −24  , so the speed is:  −24 = 482 + 24 2 + 62 = 2916 = 54 km/h.
   
 6   6 
 18 ⋅ 3
t = 49 − 48t
 0  3   49   −48  5
18    
c i  
At R: 0 + t ⋅ 4 = 32  + t  −24  ⇒  18 ⋅ 4
  5        t 5 = 32 − 24t
 5  0  0   6 

 5 = 6t
5
t= hour (50 minutes) satisfies all three equations.
6
 49   −48   49 − 40   9 
5  + 5  −24
ii Substituting t = into the expression for h (or b):  32  =  32 − 20  =  12  .
6   6     
 0   6   0+5   5 
Hence, R (9, 12, 5) .

9
Chapter 14

    200   −600   8000 


30 a i AB = OB − OA =  − =
 400   −200   600 
 1  800   0.8 
ii AB = 8002 + 6002 = 1000; hence, the unit vector is: =  .
1000  600   0.6
 0.8   200 
b i v = 250  =
 0.6   150 
 −600   200   −400 
ii At 13:00, t = 1, so:   + 1⋅  =
 −200   150   −50 
1000
iii The distance from A to B is 1000 km, and, since the velocity of the aircraft is 250 km/h, the time is = 4 hours;
250
hence, the aircraft is flying over town B at 16:00.

c Method I: Evaluating the time needed


 81
Time taken to travel from A to B to C is 9 hours  hours . The warning light will go on after 16 000 litres of fuel
9 
16 000 80 1
have been used. Time taken to use 16 000 litres = = . Hence, hour remains and the distance to town
1800 9 9
1
C is: 250 ≈ 27.8 km.
9
Method II: Evaluating the distances needed
The distance from A to B to C is 2250 km. The distance covered using 16 000 litres of fuel is:
16 000
250 ≈ 2222.22 km. So, the distance to town C is 2250 − 2222.22 ≈ 27.8 km.
1800

Method III: Evaluating fuel usage


Fuel used from A to B = 1800 × 4 = 7200 litres.

Fuel remaining until the light goes on = 16 000 2 7200 = 8800 litres.
8800 8 1
Number of hours before the warning light goes on: = 4 hours; therefore, the time remaining is hour,
1800 9 9
1
and the distance to town C is: 250 ≈ 27.8 km.
9
31 a The vectors are perpendicular if their scalar product is zero. So, firstly, we have to find the vectors:

  1 − 3   −2     1 − 4   −3 
QR =  0 − 3  =  −3  , PR =  0 − 1  =  −1 
       
 2c − 5   2c − 5   2c + 1   2c + 1 
   −2   −3 
QR ⋅ PR =  −3   −1  = 6 + 3 + ( 2c − 5)( 2c + 1) = 4 c 2 − 8c + 4
  
 2c − 5   2c + 1 
The vectors are perpendicular if: 4 c 2 − 8c + 4 = 0 ⇒ 4 (c − 1) = 0 ⇒ c = 1
2

 
  −3   −3  
   1 − 4   −3 
b PR =  −1  =  −1  , PS =  1 − 1  =  0 
 2 (1) + 1   3    
 2+1  3 


    − 3   −3   3 
PS × PR =  0  ×  −1  =  0 
     
 3   3   3
c A vector equation of the line is:
 3  −3   3 − 3t 
r =  3  + t  −1  =  3 − t  , t ∈ 
     
 5  3   5 + 3t 

10
d We need one more direction vector (which is not parallel to the direction vector of the line) to determine a
  3 − 1   2 
normal to the plane. We will take a point on the line and point S: SQ =  3 − 1  =  2  . Hence, the normal will be:
   
 2   −3  i j k  9   5−2   3 
 2  ×  −1  = 2 2 3 =  −15  .
     
 3  3  −3 −1 3  4 
Therefore, the equation will be: 9 ( x − 1) − 15 ( y − 1) + 4 (z − 2) = 0 ⇒ 9 x − 15 y + 4 z = 2 .

Note: We have a point and two vectors in the plane, so we can write parametric equations of the plane:
 1  −3   2
r = 1 + λ −1 + µ  2  , λ, µ ∈ 
   
     
 2  3   3
e Method I:

PQ ⋅ n
Shortest distance is:
n  −1   9 
 2   −15 

  3 − 4   −1   
PQ ⋅ n  6   4 

15
Since PQ =  3 − 1  =  2  , we have: = =
    n 81 + 225 + 16 3222
 5+1   6 
Method II:
We can use the distance formula for a point ( x 0 , y0 , z 0 ) and a plane a x + b y + c z + d = 0:
a x 0 + b y0 + c z 0 + d
d=
a2 + b2 + c 2
Hence, for the point P ( 4 , 1, −1) and the plane 9 x − 15 y + 4 z − 2 = 0 , the distance is:

9 ( 4) − 15 (1) + 4 (−1) − 2 15
d= = .
9 + 15 + 4
2 2 2
322

  0 − 1   −1    1 − 0   1 
32 a AB =  −1 − 2  =  −3  , BC =  0 + 1  =  1 
       
 2−1   1   2−2   0 

   −1   1  i j k  −1 
b AB × BC =  −3  ×  1  = −1 −3 1 =  1 
     
 1   0 1 1 0  2 
 −1 
1 1 1 6
c We can use the formula for the area of a triangle: A = a × b . Hence: A = 1  = 1+ 1+ 4 = .
2 2   2 2
 2 

    −1 
d A normal to the plane is n = AB × BC =  1  . Since point A is on the plane, the equation is:
 
 2 
−1( x − 1) + 1( y − 2) + 2 (z − 1) = 0 ⇒ − x + y + 2z = 3

e The normal n is parallel to the required line. Hence,


x = 2−t
y = −1 + t , where t ∈ .
z = −6 + 2t

11
Chapter 14

a x 0 + b y0 + c z 0 + d
f The distance formula for a point ( x 0 , y0 , z 0 ) and a plane a x + b y + c z + d = 0 is: d = .
a2 + b2 + c 2
Hence, for the point (2, −1 − 6) and the plane − x + y + 2z − 3 = 0, the distance is:
(−1) 2 + 1(−1) + 2 (−6) − 3 18
d= = =3 6
1+ 1+ 4 6
 −1 
1 
g Since n = 1 + 1 + 4 = 6 , a unit vector in the direction of n is: 1 .
6 2 
 
h Firstly, we will find the point of intersection of the plane and the line through D perpendicular to the plane. Hence, we
have to find the intersection of the plane P and the line from part e:
Since x = 2 − t , y = −1 + t , z = −6 + 2t , we have:

− (2 − t ) + (−1 + t ) + 2 (−6 + 2t ) = 3 ⇒ 6t = 18 ⇒ t = 3. So, the point of intersection is (−1, 2, 0) . This point is the
x + 2 yE − 1 z E − 6 
midpoint between points D and E. Hence: (−1, 2, 0) =  E , ,  ⇒ E (−4 , 5, 6).
 2 2 2 
 1  2  i j k  7 
33 a u × v =  2  ×  −1  = 1 2 3 = i 2 3 − j 1 3 + k 1 2 =  4 
    −1 2 2 2 2 −1  
 3  2  2 −1 2  −5 

b Method I:
 λ + 2µ 
 
w =  2λ − µ 
 3 λ + 2 µ 
The line of intersection of the planes is parallel to u × v . So,
 λ + 2µ   7 
 
w (u × v ) =  2 λ − µ   4  = 7 λ + 144 µ + 8 λ − 4 µ − 15 λ − 10 µ = 0 (for all λ, µ ).
 
 3 λ + 2 µ   −5 

Hence, w is perpendicular to the line of intersection.

Method II:
The line of intersection is perpendicular to the normals of both planes; hence, on vectors u and v. Therefore, it will be
perpendicular to the plane containing those two vectors, that is, to all vectors of the form λu + µ v = w .

Method III:
The line of intersection is perpendicular to the normals of both planes; hence, on vectors u and v. Therefore, for a
direction vector d of the line, it holds:
d ⋅ u = 0
 ⇒ d ( λu + µ v ) = λd ⋅ u + µd ⋅ v = 0 ; and d is perpendicular to w.
d ⋅ v = 0

    2 + 2   4 


34 a OP = OA + OB =  1 − 1  =  0  ⇒ P( 4 , 0, −3)
   
 −2 − 1   −3 
    2 + 1   3 
OQ = OA + OC =  1 + 2  =  3  ⇒ Q (3, 3, 0)
   
 −2 + 2   0 
    2 + 1   3 
OR = OB + OC =  −1 + 2  =  1  ⇒ R (3, 1, 1)
   
 −1 + 2   1 

12
     2 + 2 + 1   5 
OS = OA + OB + OC =  1 − 1 + 2  =  2  ⇒ S (5, 2, −1)
   
 −2 − 1 + 2   −1 

   2   2   −3   3 
b OA × OB =  1  ×  −1  =  −2  = − 2  ; hence, the equation of the plane is:
       
 −2   −1   −4   4 
3( x − 2) + 2( y − 1) + 4( z + 2) = 0 ⇒ 3x + 2 y + 4 z = 0

Note: Parametric equations of the plane are:


 1  4   3
r =  2  + λ 0  + µ  3  , λ, µ ∈ 
     
 1  3   0

    −3  1


c V = (OA × OB ) ⋅ OC =  −2   2  = −3 − 4 − 8 = 15
  
 −4  2

  −1 + 1   0    0 + 1   1 
35 a AB =  3 − 2  =  1  , AC =  −1 − 2  =  −3 
       
 5−3   2   1 − 3   −2 
 0 1 
 1   −3 
  
 2   −2  −7
cos θ = = ⇒ θ = 1446.789... ≈ 147
1+ 4 1+ 9 + 4 5 14

b Method I:
1   1
A = AB AC sin θ = 5 14 sin 146.789... = 2.29 units2
2 2

Method II:
1  
A = AB AC sin θ
2
2
 −7  7 3
Using the Pythagorean identity for sine and cosine, we have: sin θ = + 1 −  = 1− =
 5 14  10 10
1   1 3 21
A = AB AC sin θ = 5 14 =
2 2 10 2
Method III:
 0  1   4 
1   1 1 1 21
A = AB × AC =  1  ×  −3  =  2  = 16 + 4 + 1 =
2 2    2  2 2
 2   −2   −1 

 2   0 x=2
c i For l1 : r =  −1  + t  1  ⇒ y = −1 + t , t ∈ 
   
 0   2 z = 2t

13
Chapter 14

 −1   1  x = −1 + s
   
For l2 : r = 1 + s −3 ⇒ y = 1 − 3s , s ∈ 
   
 1   −2  z = 1 − 2s
ii The lines are not parallel, because the direction vectors are not parallel. Hence, we have to solve the system:
 2 = −1 + s ⇒ s = 3

 −1 + t = 1 − 3s
 2t = 1 − 2s

From the first equation, s = 3, and substituting into the second equation: t = 2 − 3 (3) = −7, and third equation:
2 (−7) = 1 − 2 (3) ⇒ −14 ≠ −5. Therefore, the system has no solution and the lines do not intersect.
( e − d)(l1 × l2 )
d The shortest distance is given by , where d and e are position vectors of the points on the lines,
(l1 × l2 )
and l1 and l2 are direction vectors of the lines.
   4 
l1 × l2 = AB × AC =  2 
 
 −1 
  −1   2    4   −3   4 
  1  −  −1    2   2  2 
        
( e − d)(l1 × l2 )   1   0    −1   1   −1  9
Hence, = = = .
(l1 × l2 ) 16 + 4 + 1 21 21

36 a Method I:
We will use matrices and their properties:
−1
 1 3 −2   x   −6   x   1 3 −2   −6   1 
 2 1 3  y  = 7  ⇒ y  = 2 1 3   7  =  −1 
       
 z   3 −1 1   6   
 3 −1 1   z   6    2 

Method II:
We will use PolySmlt:

 1   2 i j k  11 
    3 −2 1 −2 1 3  −7 
b v = 3 × 1 = 1 3 −2 = i − j + k =
    1 3 2 3 2 1  
 −2   3  2 1 3  −5 
c Method I:
 1   2   m + 2n 
u = m  3  + n  1  =  3m + n 
     
 −2   3   −2m + 3n 

14
 11   m + 2n 
vu =  −7   3m + n  = 11m + 22n − 21m − 7n + 10m − 15n = 0
  
 −5   −2m + 3n 

Method II:
From part b, v is perpendicular to both a and b, so va = 0 and vb = 0. Hence, v (ma + nb) = mva + nvb = 0, for all
values of m and n.
 3 
d The line is perpendicular to vector v and to the vector  −1  . So, a direction vector of the line is:
 
 1 
 11   3  i j k  −12   6 
 −7  ×  −1  = 11 −7 −5 = i −7 −5 − j 11 −5 + k 11 −7 =  −26  = −2  13  ,
    −1 1 3 1 3 −1    
 −5   1  3 −1 1  10   −5 
 1   6 
and a vector equation of the line is: r =  −1  + λ  13 
   
 2   −5 

  1 − 1   0    2 − 1   1 
37 a i AB =  2 − 3  =  −1  , AC =  3 − 3  =  0 
       
 4 −1  3   6 −1  5 

   0   1  i j k  −5 
−1 3 0 3 0 −1 
   
AB × AC = −1 × 0 = 0 −1 3 = i −j +k = 3 
    0 5 1 5 1 0  
 3   5 1 0 5  1 

 −5 
1   1  1 35
ii A= AB × AC = 3  = 25 + 9 + 1 =
2 2  2 2
 1 
   −5 
b i The plane contains the point A and its normal is AB × AC =  3  ; hence, for the Cartesian equation, it holds:
 
 1 
−5( x − 1) + 3 ( y − 3) + 1(z − 1) = 0 ⇒ −5x + 3 y + z = −5 + 9 + 1

The equation is: −5x + 3 y + z = 5.


   −5 
iiThe line contains the point D and its direction vector is AB × AC =  3  ; hence, the Cartesian equation of the
 
line is:  1 
x − 5 y + 2 z −1
= =
−5 3 1
c We will firstly write the equation of the line in parametric form, and then solve the system:
x = 5 − 5t
y = −2 + 3t
z = 1+ t
−5 (5 − 5t ) + 3 (−2 + 3t ) + (1 + t ) = 5 ⇒ −25 + 25t − 6 + 9t + 1 + t = 5 ⇒ 35t = 35 ⇒ t = 1
x = 5 − 5 (1) = 0 

The point is: y = −2 + 3 (1) = 1 ⇒ (0, 1, 2)
z = 1 + (1) = 2 
d The distance is the same as the distance between points D and P:

d = (5 − 0) + (−2 − 1) + (1 − 2) = 25 + 9 + 1 = 35
2 2 2

15
Chapter 14

 1
38 a The line contains the point A and its direction vector is  1  ; hence, the Cartesian equation of the line is:
 
x − 2 y − 5 z +1  1
= =
1 1 1
b We will firstly write the equation of the line in parametric form, and then solve the system:
x = 2+t
y = 5+t
z = −1 + t
5
1(2 + t ) + 1(5 + t ) + 1(−1 + t ) − 1 = 0 ⇒ 2 + t + 5 + t − 1 + t − 1 = 0 ⇒ 3t = −5 ⇒ t = −
3
 5 
x = 2 + −  
 3

5  1 10 8
The point is: y = 5 +  −   ⇒  , , − 
 3 3 3 3

5 
z = −1 +  −  
 3 
c Method I:
 1 10 8
Denote the image point A′ . Then, the point of intersection,  , , −  , of the line and the plane is the
3 3 3
 1 10 8   2 + x ′ 5 + y ′ −1 + z ′ 
midpoint of AA′. Hence,  , , −  =  , , ⇒
3 3 3  2 2 2 
1 2 + x′ 2 4
= ⇒ x′ = − 2 = −
3 2 3 3
10 5 + y ′ 20 5
= ⇒ y′ = −5=
3 2 3 3
8 −1 + z ′ 16 13
− = ⇒ z′ = − + 1= −
3 2 3 3
 4 5 13
Thus, A′  − , , −  .
 3 3 3
Method II:
Parameters of the points of the line are:
t = 0 for A
5 10
t = − for the intersection; hence, t = − for the reflected point.
3 3
 10  
x = 2 +  −  
 3

 10   4 5 13
Thus: y = 5 +  −   ⇒  − , , − 
 3  3 3 3

10 
z = −1 +  −  
 3 

 1
  2 − 2   0 
d We have: AB =  0 − 5  =  −5  , and a direction vector of the line d =  1.
     
 6 +1  7   1

16
Hence:
 0   1  −12 
 −5  ×  1   7 
      
AB × d  7   1  5  218  654 
d= = = = =
d 1+ 1+ 1 3 3  3 

 3 
39 a The plane contains the point P and its normal is  −4  ; hence, the Cartesian equation of the plane is:
 
 1 
3 ( x − 1) − 4 ( y − 2) + 1(z − 11) = 0 ⇒ 3x − 4 y + z = 6

b i (1) + 3 (2) − (11) = 1 + 6 − 11 = 4; hence, P lies in π2 .


ii The intersection of the planes contains the point P and its direction vector is the vector product of the normals:
 3   1   1 
 −4  ×  3  =  4  . Hence, a vector equation of the line is:
     
 1   −1   13 
 1  1 
r =  2  + t  4  , t ∈
   
 11   13 
c The angle between the normals is:
 3  1 
 −4   3 
  
 1   −1  −10
cos θ1 = =
9 + 16 + 1 1 + 9 + 1 26 ⋅ 11
10
Hence, the angle between the planes is: cos θ = ⇒ θ = 53.7498... ≈ 53.7 .
26 ⋅ 11
x+2 y z−9
40 a = = = µ ⇒ x = −2 + 3 µ , y = µ , z = 9 − 2 µ ; hence: M (−2 + 3 µ , µ , 9 − 2 µ)
3 1 −2
x−4 y z+3
b i = =
3 1 −2
 −2 + 3 µ − 4   3µ − 6 
    
ii PM =  µ−0  = µ 
 9 − 2 µ + 3   −2 µ + 12 

 3µ − 6   3 
  3   
  µ 
c i PM ⋅ 1 = 0 ⇒ 
    1  = 0 ⇒ 9 µ − 18 + µ + 4 µ − 24 = 0 ⇒ 14 µ = 42 ⇒ µ = 3
 −2   −2 µ + 12   −2 

ii The distance between the lines of magnitude of the vector PM , where µ = 3:
 3µ − 6   3 
   
PM =  µ  = 3
 ; hence, the distance is: d = 9 + 9 + 36 = 54 = 3 6

( )
 −2 µ + 12   6 

 3   3   −12   2 
   
d A normal to the plane equals: 3 × 1 = 24 = −6  −4
   ; hence, the Cartesian equation of the plane is:
       
 6   −2   −6   1 

2 ( x − 4) − 4 ( y − 0) + 1(z + 3) = 0 ⇒ 2x − 4 y + z = 5

17
Chapter 14

Solution Paper 1 type

e The line is on π1 (from part d).

Testing the line on π2 : (−2 + 3 µ) − 5 ( µ) − (9 − 2 µ) = −2 + 3 µ − 5 µ − 9 + 2 µ = −11. Therefore, the line is in both


planes; hence, l1 is the line of intersection.

Solution Paper 2 type

e Solve the system:


 2x − 4 y + z = 5

x − 5 y − z = −11

So, the intersection is the line:
 11.5   −1.5 
r =  4.5  + t  −0.5  , t ∈ 
   
 0   1 
 −1.5   3 
A direction vector of the line is: −2 ⋅  −0.5  =  1  , and, for t = 9, the position vector of the point is:
   
 1   −2 
 11.5   −1.5   −2 
 4.5  + 9  −0.5  =  0  , which is line l .
      1

 0   1   9 

41 a L1 : x = 2 + t , y = 2 + 3t , z = 3 + t

L2 : x = 2 + s , y = 3 + 4 s , z = 4 + 2s
x = 2 + t = 2 + s

Hence, at the point of intersection:  y = 2 + 3t = 3 + 4 s
z = 3 + t = 4 + 2s

Method I:
From the first equation, we have t = s; from the second, 2 + 3t = 3 + 4t ⇒ t = −1 . Substituting into the third equation:
3 + (−1) = 2
 ; hence, the lines intersect at the point (1, −1, 2) .
4 + 2 (−1) = 2

Method II:
x = 2 + t = 2 + s t − s = 2 − 2
 
 y = 2 + 3t = 3 + 4 s ⇒ 3t − 4 s = 3 − 2
z = 3 + t = 4 + 2s t − 2s = 4 − 3
 

Hence, t = s = −1 and the point of intersection is (1, −1, 2) .

b The normal to the plane is perpendicular to both direction vectors, hence:


 1  1  2 
 3  ×  4  =  −1  . Since the plane contains the intersection point (1, −1, 2) , the Cartesian equation of the plane is:
     
 1  2   1 
2 ( x − 1) − 1( y + 1) + 1(z − 2) = 0 ⇒ 2x − y + z = 5.

18
 1 + 3 −1 + 4 2 + 3  3 5 
c The midpoint M of [PQ ] is: M =  , ,  =  2, ,  .
 2 2 2   2 2
   2   2t 
The vector MS is parallel to the normal to the plane π , so MS = t  −1  =  − t  ; hence,
   
3 5 
  1   t 
S  2t + 2, −t + , t +  . From PS = 3 ⇒

 2 2 
2 2 2 2
3 5 5 1
(2t + 2 − 1)2 +  −t + + 1 +  t + − 2 = (2t + 1)2 +  −t +  +  t + 
 2   2   2   2
15 1 1
6t 2 + = 3 ⇒ t2 = ⇒ t = ±
2 4 2
So, the possible solutions for S are:
1 3 1 5 1 3 1 5
S1  1 + 2, − + , +  = (3, 1, 3) and S2  −1 + 2, + , − +  = (1, 2, 2).
 
 2 2 2 2  2 2 2 2

 
Note: We used the fact that PS = 3. The line L is the symmetry line of the segment [PQ ] ; hence, QS should be 3.

That means that we will have the same equations if we use QS = 3.

x = 2 − 2λ + µ x = 2+s+t
42 a i L1 : y = 1 + λ − 3 µ L2 : y = 0 + 2s + t
z = 1+ 8λ − 9µ z = 1+ s + t

2 − 2 λ + µ = 2 + s + t

Hence, at the points of intersection: 1 + λ − 3 µ = 2s + t
1 + 8 λ − 9 µ = 1 + s + t

Subtracting the third equation from the first, we have: 1 − 10 λ + 10 µ = 1 ⇒ λ = µ.

Solution Paper 1 type

x = 2 − 2λ + λ = 2 − λ
ii If λ = µ for points on the plane L1, then those points are on the line: y = 1 + λ − 3 λ = 1 − 2 λ , whose vector
z = 1+ 8λ − 9λ = 1− λ
 2  −1 
equation is: r =  1  + λ  −2  , λ ∈ .
   
 1  −1 

19
Chapter 14

Solution Paper 2 type

2 − 2 λ + µ = 2 + s + t −2 λ + µ − s − t = 0
 
ii We have to solve the system: 1 + λ − 3 µ = 2 s + t ⇒  λ − 3 µ − 2 s − t = −1
1 + 8 λ − 9 µ = 1 + s + t 8 λ − 9 µ − s − t = 0
 

x = 2 + k − 1 = 1+ k
If we substitute into L2 : s = k , t = −1, we have: y = 0 + 2k − 1 = −1 + 2k
z = 1+ k − 1 = k
 1   1
Hence, the intersection line is: r =  −1  + k  2  , k ∈ .
   
 0   1
x = 2 − 2 (1 − l ) + (1 − l ) = 1 + l
Note: If we substitute into L1 : λ = 1 − l , µ = 1 − l , we have: y = 1 + (1 − l ) − 3 (1 − l ) = −1 + 2l ; hence,
z = 1 + 8 (1 − l ) − 9 (1 − l ) = l
 1   1
r =  −1  + l  2  , l ∈ .
   
 0   1

Note: The lines found above are the same, since the direction vectors are parallel and, if we substitute I = 1 into the
 2
second equation, we will have the point  1  from the first line. Hence, we can use any solutions in the remainder of the
 
task.  1

b The point (2, 0, −1) from the line is on the plane. Hence, the Cartesian equation is:
3 ( x − 2) − 2 ( y ) + (z + 1) = 0 ⇒ 3x − 2 y + z = 5
 2  −1 
c The planes intersect at the line r =  1  + λ  −2  , λ ∈ , so we have to find the intersection of this line and the
   
plane π3 .  1  −1 

We will firstly write the equation of the line in parametric form, and then solve the system:
x = 2− λ
y = 1− 2λ
z = 1− λ
3 (2 − λ) − 2 (1 − 2 λ) + (1 − λ) = 5 ⇒ 5 = 5

The equation is satisfied by any real value of λ ; hence, the plane π3 contains the line, and the intersection of the three
 2  −1 
planes is the line r =  1  + λ  −2  , λ ∈ .
   
 1  −1 

20
Chapter 15
Practice questions

1 The important points of the first derivative are zeros, where the original function reaches a minimum or maximum,
x = −4 and x = 1, and the maximum point, at the midpoint. The intervals of the positive and negative values
of the first derivative are to be established by the increasing/decreasing intervals of the original function. So:
f ' ( x ) > 0, −4 < x < 1, and f ' ( x ) < 0, x < −4 or x > 1 .
y

2 a Given the product form of the function, the values a and b are zeros that can be easily read from the graph.
Therefore, i a = −4 and ii b = 2.
b i We can use the product rule but we have to be careful since there are three factors.
f ' ( x ) = − ( x + 4) ( x − 2) − x ( x − 2) − x ( x + 4) = − ( x 2 + 2x − 8 + x 2 − 2x + x 2 + 4 x )
= − (3x 2 + 4 x − 8) = 8 − 4 x − 3x 2
ii f ' ( x ) = 0 ⇒ 8 − 4 x − 3x 2 = 0 ⇒ 3x 2 + 4 x − 8 = 0 ⇒
−4 ± 16 + 96 −4 ± 4 7 −2 ± 2 7
x= = = ⇒
6 6 3
−2 − 2 7 −2 + 2 7
x= or x =
3 3
iii f (1) = −1(1 + 4) (1 − 2) = 5
c i m = f ' (0) = 8 ⇒ Equation of tangent: y = 8 x
ii − x ( x + 4) ( x − 2) = 8 x ⇒ x ( 8 + x 2 + 2x −8 ) = 0 ⇒ x 2 ( x + 2) = 0 . Since the point differs from the origin,
we can conclude that the x-coordinate of the second point is x = −2.
3 a i When t = 0 : v (0) = 66 − 66e −0.15×0 = 66 − 66 × 1 = 0
ii When t = 10 : v (10) = 66 − 66e −0.15×10 = 66 (1 − e −1.5 ) ≈ 51.3 m/s
b i a (t ) = v ' (t ) = −66e −0.15t × (−0.15) = 9.9e −0.15t
ii a (0) = 9.9e −0.15×0 = 9.9 m/s2
  →0

c i lim  66 − 66 e −0.15t  = 66
t →∞
 
  →0

ii lim  9.9 e −0.15t  = 0
t →∞
 
iii Since the velocity is constant (66 m/s), the acceleration must be zero.

1
Chapter 15

2
4 a y ' = 3x 2 + 14 x + 8 ⇒ y ' = 0 ⇒ 3x 2 + 14 x + 8 = 0 ⇒ (3x + 2) ( x + 4) = 0 ⇒ x = − or x = −4
3
2 2 2
x x < –4 –4 −4 < x < − − x>−
3 3 3

f ′(x) positive 0 negative 0 positive


f (x) increases maximum decreases minimum increases

To find the exact coordinates, we can use synthetic division, also known as Horner’s algorithm.

f (x) 1 7 8
x 23

−4 1 3 24 13

2 19 34 149
− 1 −
3 3 9 27

 2 149 
So, the maximum point is (−4 , 13) and the minimum point is  − , − .
 3 27 
14 7
b y " = 6 x + 14 ⇒ 6 x + 14 = 0 ⇒ x = − =−
6 3
3 2
7 7 7 7 343 343 56 −343 + 1029 − 504 − 81 101
f  −  =  −  + 7 ×  −  + 8 ×  −  − 3 = − + − −3= =
 3  3  3  3 27 9 3 27 27

 7 101
So, the point of inflexion is  − , .
 3 27 
Note: Horner’s algorithm could also be used here.

f (x) 1 7 8
x 23

7 14 26 101
− 1 −
3 3 9 27

5 a i g ( x ) = 2 + e −3 x ⇒ g ' ( x ) = e −3 x × −3 = −3e −3 x
ii Since −3e −3 x < 0 for all real values of x, we can conclude that the function always decreases.
1
1 −3×  − 
b i g ( x ) = 2 + e −3 x ⇒ g  −  = 2 + e  3 = 2 + e
 3
1
1 −3×  − 
ii g ' ( x ) = −3e −3 x ⇒ g '  −  = −3e  3 = −3e
 3
1
c y = −3e  x +  + 2 + e ⇒ y = −3e x − e + 2 + e = −3e x + 2
 3
6 a Firstly, we are going to write the function in product form:
f ( x ) = (2x 2 − 13x + 20) ( x − 1) ⇒ f ' ( x ) = ( 4 x − 13) ( x − 1) + (2x 2 − 13x + 20) × (−2) ( x − 1)
−2 −2 −3

4 x 2 − 17 x + 13 −4 x 2 + 26 x − 40
= ( x − 1)
−3
((4 x − 13) (x − 1) + (2x 2
− 13x + 20) × (−2)) =
( x − 1)3
9 x − 27
= ,x ≠1
( x − 1)3

2
b We know that a minimum point has the first derivative equal to zero, and therefore:
9 x − 27 27
f ' (x ) = 0 ⇒ = 0 ⇒ 9 x − 27 = 0 ⇒ x = = 3. Also, the second derivative must be positive, so:
( x − 1)3 9
72 − 18 x 72 − 18 × 3 72 − 54 18 9
f " (x ) = ⇒ f " (3) = = = = > 0.
( x − 1)4 (3 − 1)4 16 16 8
Therefore, f(3) is a minimum.
c For the point of inflexion, the second derivative must be equal to zero.
72 − 18 x 72
f " (x ) = 0 ⇒ 4 = 0 ⇒ 72 − 18 x = 0 ⇒ x = =4
( x − 1) 18
2 × 4 2 − 13 × 4 + 20
y = f ( 4) = = 0 ⇒ I ( 4 , 0)
( 4 − 1)2
7 a Rewriting as an expression having an integer exponent:
−4 3
y = (2x + 3) ⇒ y ' = −2 × (2x + 3) × 2 =
−2 −3
3 , x ≠ −
(2x + 3) 2
b y = e sin(5 x ) ⇒ y ' = e sin(5 x ) × cos (5x ) × 5 = 5 cos (5x ) e sin(5 x )
4 x sin ( x 2 )
c y = tan2 ( x 2 ) ⇒ y ' = 2 tan ( x 2 ) sec 2 ( x 2 ) × 2x = 4 x tan ( x 2 ) sec 2 ( x 2 ) or y ' =
cos3 ( x 2 )
C C C C C
8 y = Ax + B + ⇒ y ' = A − 2 ⇒ y ' = 0 ⇒ A = 2 ⇒ x2 = ⇒ x = ±
x x x A A
C
By observing the given stationary points, we can establish a relationship between A and C : x = ±1 ⇒ = 1 ⇒ C = A .
A
Now, we need to use the fact that points P and Q lie on the curve itself, and therefore their coordinates satisfy the equation
of the curve.
A
P (1, 4) ⇒ 4 = A × 1 + B + ⇒ 2 A + B = 4 
1 
 ⇒ 2B = 4 ⇒ B = 2 ⇒ A = C = 1
A
Q (−1, 0) ⇒ 0 = A × (−1) + B + ⇒ −2 A + B = 0 
−1 
d 3x2 x2 12
9 x3 + y3 = 2 ⇒ 3x 2 + 3 y 2 y ' = 0 ⇒ y ' = − = − 2 ⇒ y ' (1, 1) = − 2 = −1
dx 3y 2
y 1
2
d
⇒ 6 x = − 3 (2 yy '× y '+ y 2 y ") ⇒ 2x + 2 y ( y ') = − y 2 y "
2
3 x 2 = −3 y 2 y '
dx
2x + 2 y ( y ') 2 × 1 + 2 × 1 × (−1)
2 2

y"= − ⇒ y " (1, 1) = − = −4


y2 12
x 1 × (e x − 1) − x × e x (1 − x ) e x − 1
10 a y= x ⇒ y'= =
e −1 (e x − 1)2 (e x − 1)2
b y = e x sin 2x ⇒ y ' = e x sin 2x + e x cos 2x × 2 = e x (sin 2x + 2 cos 2x )
1 1
c y = ( x 2 − 1) ln 3x ⇒ y ' = 2x × ln 3x + ( x 2 − 1) × = 2x ln 3x + x −
x x
1 1 1 1 3
11 y = x − 4 x ⇒ y ' = 2x − 4 ⇒ mN = −
2
= − , equation of normaal: y = − ( x − 3) − 3 ⇒ y = − x −
y ' (3) 2 2 2 2
1 3 3 3 1 3
x=0⇒ y=− × 0 − = − , P  0, −  ; y = 0 ⇒ 0 = − x − ⇒ x = −3, Q (−3, 0)
2 2 2  2 2 2
12 a The x-coordinate of P can be read from both graphs. In the first derivative graph, we can see that the curve has a
minimum point at the point where x = 3, whilst, in the second graph, we can see that the line has a zero at the point
where x = 3.
b Since point M is a maximum point, we know that the first derivative must be equal to zero; therefore, by observing
the first graph, we have two possible values: x = 1 or x = 5. Now, by looking at the second graph, we can see that
x = 1 ⇒ y " (1) < 0 and we have a maximum point. For the second point, where x = 5 ⇒ y " (5) > 0, we have a local
minimum.

3
Chapter 15

c 6 y
M

x
P
−2 2 4 6

−2

−4

−6

d 2x + y
13 x 2 + xy + y 2 − 3 y = 10 ⇒ 2x + y + xy '+ 2 yy '− 3 y ' = 0 ⇒ 2 x + y = y ' (3 − x − 2 y ) ⇒ y ' =
dx 3 − x − 2y
1 3−2−2×3 5 5 5 11
mN = − =− = ⇒ Equation of normall: y = ( x − 2) + 3 ⇒ y = x +
y ' (2, 3) 2×2+3 7 7 7 7

Solution Paper 1 type

128
14 V = r 2 π h ⇒ 128 π = r 2 π h ⇒ h =
r2
128 256π
S = 2r 2 π + 2r π h ⇒ S ( r ) = 2r 2 π + 2 r π 2
= 2r 2 π +
r r
256π 64 64 64 128
S ' (r ) = 4 r π − = 4 π  r − 2  ⇒ S ' (r ) = 0 ⇒ r − 2 = 0 ⇒ r = 2 ⇒ r 3 = 64 ⇒ r = 3 64 = 4 ⇒ h = 2 = 8
r 2  r  r r 4
So, the radius is 4 cm and the height is 8 cm.

Solution Paper 2 type

14 On a GDC, we simply input the surface area function and calculate a minimum.

So, the minimum surface area occurs when the radius is 4 cm and the height is 8 cm.

15 Let’s focus on the vertex of the rectangle in the first quadrant. The coordinates are ( x , y ) = ( x , 12 − x 2 ). Now, the
dimensions of the rectangle and its area are: l = 2x , w = y ⇒ A = lw = 2xy ⇒ A ( x ) = 2x (12 − x 2 ) = 24 x − 2x 3.
To find the maximum possible area, we need to find the zero of the first derivative.
A ' ( x ) = 24 − 6 x 2 ⇒ A ' ( x ) = 0 ⇒ 24 − 6 x 2 = 0 ⇒ 24 = 6 x 2 ⇒ x 2 = 4 ⇒ x = 2, since the vertex is in the first
quadrant. Again, we can verify that we have a maximum point since the second derivative test gives us a negative value:
A " ( x ) = −12x ⇒ A " (2) = −12 × 2 = −24 < 0 .
So, the dimensions are l = 2 × 2 = 4 and w = 12 − 22 = 8, and that gives us the maximum possible area of 32.

4
16 a The first derivative is negative when the function is decreasing, whilst the second derivative is negative when the
function is concave down. By looking for these features, we identify point E.
b The first derivative is negative when the function is decreasing, whilst the second derivative is positive when the
function is concave up. By looking for these features, we identify point A.
c The first derivative is positive when the function is increasing, whilst the second derivative is negative when the
function is concave down. By looking for these features, we identify point C.
2x − 1 2 ( x + 2) − (2x − 1) × 1 2x + 4 −2x + 1 5
17 y = ⇒ y'= = =
x+2 ( x + 2)2 ( x + 2)2 ( x + 2)2
5 1
y ' (−3) = = 5 ⇒ mn = −
(−3 + 2)2 5
1 1 3 1 32
Equation of normal: y = − ( x + 3) + 7 ⇒ y = − x − + 7 ⇒ y = − x +
5 5 5 5 5
4 4
18 y = ln ( 4 x − 3) ⇒ y ' = ⇒ y ' (1) = =4
4x − 3 4 × 1− 3
a mT = y ' (1) = 4 ⇒ Equation of tangent: y = 4 ( x − 1) + 0 ⇒ y = 4 x − 4
1 1 1 1 1
b mN = − = − ⇒ Equation of normal: y = − ( x − 1) + 0 ⇒ y = − x +
y ' (1) 4 4 4 4
1 2
19 y = x 2 ln x ⇒ y ' = 2x ln x + x 2 × = x (2 ln x + 1) ⇒ y " = 2 ln x + 1 + x × = 2 ln x + 3
x 1
x
1 − 1 1
a y ' = 0 ⇒ x (2 ln x + 1) = 0 ⇒ 2 ln x + 1 = 0 ⇒ ln x = − ⇒ x = e 2 = 1 orr ⇒
2 e
1 2
 −  1
1 1 1 e 2

; so, P  , − 

y =  e 2  ln e 2 = −
  2e  e 2e 
Notice that the domain of the function is x > 0; therefore, we discard the solution x = 0.
 − 1 −
1
y "  e 2  = 2 ln e 2 + 3 = −1 + 3 = 2 > 0 ; therefore, the point P is a minimum.
 
lim y = lim− x 2 ln x = 0 and lim y = lim x 2 ln x = ∞ ; therefore, the point P is the absolute minimum.
x →0 − x →0 x →∞ x →∞
2

3
1 1  −3 −
3
3  1 − 3 
b y " = 0 ⇒ 2 ln x + 3 = 0 ⇒ x = e 2
= = , y =  e 2  ln e 2 = − 3 ; so, I  , 
3
e e   2e e e 2e 3 
e2
a a 2a
20 a f ( x ) = x 2 + ⇒ f ' ( x ) = 2x − 2 ⇒ f " ( x ) = 2 + 3
x x x
a
i f ' (2) = 0 ⇒ 2 × 2 − 2 = 0 ⇒ a = 16
2
a
ii f ' (−3) = 0 ⇒ 2 × (−3) − = 0 ⇒ a = −54
(−3)2
a a  a 2a 2a
b f ' ( x ) = 2x − 2 = 0 ⇒ 2x 3 = a ⇒ x = 3 , f "  3  = 2 + 3 = 2+ =6>0
x 2  2   3 a a
 2  2
Since the second derivative is always positive, the stationary point cannot be a maximum.
21 A line y = mx + l that passes through (3, 2) satisfies the following equation:
2 = m × 3 + l ⇒ l = 2 − 3m; y = mx + 2 − 3m
3m − 2
x = 0 ⇒ y = 2 − 3m; y = 0 ⇒ 0 = mx + 2 − 3m ⇒ x =
m
Therefore, the area of the triangle is given by the expression:
1 3m − 2 (3m − 2)2 2 (3m − 2) × 3 × 2m − (3m − 2) × 2
2

A (m) = (2 − 3m) =− ⇒ A ' (m) = −


2 m 2m 4 m2
2 (3m − 2) (6m − 3m + 2) 2 (3m − 2) (3m + 2) 2 2
=− =− ⇒ A ' (m) = 0 ⇒ (3m − 2) (3m + 2) = 0 ⇒ m = or m = −
4 m2 4 m2 3 3

5
Chapter 15

2
We can discard the first solution since, for m = , the line passes through the origin and so the triangle doesn’t exist.
3
2  2 2
m = − ⇒ l = 2 − 3 × −  ; y = − x + 4
3  3 3
π π π π π π
22 y = x tan x : x = ⇒ y = tan = , P  , 
4 4 4 4  4 4
π π π π π π
y ' ( x ) = tan x + x sec 2 x ⇒ y '   = tan + sec 2   = 1 + × 2 = 1+
 4 4 4  4 42 2
π π π π π π π2
mT = y '   = 1 + ⇒ Equation of tangent: y =  1 +   x −  + ⇒ y = 1 +  x −
 4 2  2  4 4  2 8
1 1 2 2  π π
mN = − =− =− ⇒ Equation of normal:: y =  − x −  + ⇒
 π π 2 + π  2 + π   4  4
y'  1+
 4 2
2  2π π 2  4π + π 2
y =  −  x+ + ⇒ y =  −  x+
 2 + π 4 (2 + π ) 4  2 + π 4 (2 + π )
x2
x2 − − 2x x2
− e 2
× −
e 2
2 xe 2
23 a f (x ) = ⇒ f ' (x ) = =− ⇒
2π 2π 2π
x2
x2 − − 2x x2
e ×
e 2 ( x 2 − 1)
− 2 −
e 2
2
f " (x ) = − −x =
2π 2π 2π
e0 1 1 
f ' ( x ) = 0 ⇒ x = 0 , y = f (0 ) = = ⇒ M  0, 
2π 2π  2π 
( ±1)2

1  1   e 1  2
f " ( x ) = 0 ⇒ x 2 − 1 = 0 ⇒ x = ±1, y = − = ⇒ I1  −1,  , I2  1, 
2π 2e π  2e π   2e π 
b We notice that the function f is even, and therefore symmetrical with respect to the y-axis.
 0
x2

e 2
lim f ( x ) = lim f ( x ) = lim = 0; therefore, the x-axis is the horizontal asymptote.
x →−∞ x →∞ x →∞ 2π
c
y

(0, 0.3989)
(-1, 0.242) (1, 0.242)
x
–2 –1 1 2

–1

2x 4x  −x2 + 4x − 3
24 a f ( x ) = 2 ln ( x 2 + 3) − x ⇒ f ' ( x ) = 2 − 1= 2 − 1 = ⇒
x +3
2
x +3  x 2 + 3 
4x 4x
f ' (x ) = 0 ⇒ − 1= 0 ⇒ 2 = 1 ⇒ 4 x = x 2 + 3 ⇒ 0 = x 2 − 4 x + 3 ⇒ ( x − 3) ( x − 1) = 0
x2 + 3 x +3
x = 3, y = 2 ln 12 − 3; or x = 1, y = 2 ln 4 − 1 = 4 ln 2 − 1

6
We notice that the denominator is always positive; therefore, the sign of the first derivative depends on the numerator,
which is a quadratic expression that has a negative leading coefficient.

x ]− ∞ , 1[ 1 ]1, 3[ 3 ]3, ∞ [
f ’(x) negative 0 positive 0 negative
f (x) decreasing minimum increasing maximum decreasing

To conclude: (1, 4 ln 2 − 1) is a minimum point and (3, 2 ln 12 − 3) is a maximum point.


4 ( x 2 + 3) − 4 x × 2x 12 − 4 x 2
b f " (x ) = = ⇒ f " ( x ) = 0 ⇒ 12 − 4 x 2 = 0 ⇒ x = ± 3
(x 2
+ 3)
2
(x 2
+ 3)
2

Again, since the denominator is always positive, we can conclude that the sign depends on the numerator only, and
the numerator is a quadratic expression which changes its sign at the zeros. Therefore, we can conclude that the
x-coordinates we found are those of inflexion points.
2x 2 (18 + 0.015x 2 ) − 2x × 0.3x 2 (18 − 0.015x 2 )
25 f ( x ) = ⇒ f ' (x ) = =
18 + 0.015x 2
(18 + 0.015x 2 )2 (18 + 0.015x 2 )2
18
f ' ( x ) = 0 ⇒ 18 − 0.015x 2 = 0 ⇒ x 2 = = 1200 ⇒ x = 1200 = 20 3 ≈ 34.6 km h r
0.015
d 4 2 x 2x
26 2x 2 − 3 y 2 = 2 ⇒ 4 x − 6 yy ' = 0 ⇒ y ' = =
dx 6 3y 3y
x = 5 ⇒ 2 × 52 − 3 y 2 = 2 ⇒ 48 = 3 y 2 ⇒ y 2 = 16 ⇒ y = ± 16 = ±4
When x = 5 we have two values of y; therefore, there are two points and hence two gradients.
2 ×5 5 2 ×5 5
y ' (5, −4) = =− , y ' (5, 4) = =
3 × (− 4 2) 6 3× 42 6
−1 4x
27 y = arccos (1 − 2x 2 ) ⇒ y ' = × (−4 x ) =
1 − (1 − 2x )
2 2
1 − ( 1 − 4x2 + 4x 4 )
 −2
 , − 1< x < 0
4 2x  1− x2 gn ( x )
2 × sig
= =  =
2 x 1− x2  2 1 − x2
, 0  x < 1
 1− x2

Note: We had to restrict the domain of the derivative because of the expression in the denominator.
1
28 f ( x ) = x 2 ln x ⇒ f ' ( x ) = 2x ln x + x 2 × = x (2 ln x + 1) , x > 0
x
1 1
29 f ( x ) = sin 2x + cos x ⇒ f ' ( x ) = cos 2x × 2 − sin x = 1 − 2 sin2 x − sin x = (1 − 2 sin x ) (1 + sin x )
2 2
1
f ' ( x ) = 0 ⇒ (1 − 2 sin x ) (1 + sin x ) = 0 ⇒ sin x = or sin x = −1
2
d 6 3x 3x
30 3x 2 + 4 y 2 = 7 ⇒ 6 x + 8 yy ' = 0 ⇒ y ' = − =−
dx 8 4y 4y
x = 1 ⇒ 3 × 12 + 4 y 2 = 7 ⇒ 4 y 2 = 4 ⇒ y 2 = 1 ⇒ y = 1 because y > 0 .
3×1 3
y ' (1, 1) = − =−
4 ×1 4
2
31 a f ( x ) = ln (2x − 1) ⇒ f ' ( x ) =
2x − 1

7
Chapter 15

Solution Paper 1 type

2 1 ± 1 + 16 1 ± 17
b f ' (x ) = x ⇒ = x ⇒ 2 = 2x 2 − x ⇒ 0 = 2x 2 − x − 2 ⇒ x1, 2 = =
2x − 1 4 4
We need to check whether both solutions are in the domain of the function f. We notice that the domain is:

figures).
{
D (f ) = x x ∈  x >
1
2 }
; therefore, there is only one solution: x =
1 + 17
4
≈ 1.28 (correct to three significant

Solution Paper 2 type

b We can draw the derivative function on a calculator without finding the formula for it. To obtain a clearer diagram, we
deselect the original function so that we can find the point of intersection easier with the identity function.

Solution Paper 1 type

32 f ( x ) = x 2 e x ⇒ f ' ( x ) = 2x e x + x 2e x ⇒ f " ( x ) = 2e x + 2x e x + 2x e x + x 2e x = e x (2 + 4 x + x 2 )

Since e x > 0 : f " ( x ) = 0 ⇒ x 2 + 4 x + 2 = 0 ⇒ x1, 2 = −2 ± 4 − 2 = −2 ± 2 . The condition of the question is that the
x-coordinate of the point of inflexion is between 22 and 0; therefore, there is only one solution: x = −2 + 2 ≈ −0.586.

Solution Paper 2 type

32 As in the previous question, we can draw the graph of the second derivative and then simply find the zero that is
between –2 and 0.

1
33 Since the normal has a slope of 22, we need to find a point with a gradient of .
2
1 1
y = arctan ( x − 1) ⇒ y ' = 2 =
1 + ( x − 1) 2 − 2x + x 2
1
y ' (x ) =
⇒ 2 − 2x + x 2 = 2 ⇒ x 2 − 2 x = 0 ⇒ x ( x − 2) = 0 ⇒ x = 2 given the condition x > 0 .
2
π π π
x = 2 ⇒ y = arctan (1) = ⇒ = −2 × 2 + c ⇒ c = + 4 ≈ 4.79 (correct to three significant figures)
4 4 4
34 a f ( x ) = e1+ sin π x ⇒ f ' ( x ) = e1+ sin π x × cos (π x ) × π = π cos (π x ) e1+sin π x

8
b Since x  0 , we need to find the x-coordinates of the points where f ' ( x ) = 0:
π 1
cos (π x ) = 0 ⇒ π x = + nπ ⇒ x = + n, n ∈ 
2 2
1 3 5 2n + 1
x 0 = , x1 = , x 2 = , ..., x n = , n ∈
2 2 2 2
f ( x ) = x 3 ( x 2 − 1) , −1.4  x  1.4
2
35 a
In this instance, we need to set up the window on the calculator properly and simply copy it onto graphic paper.

Maximum 2 y
Zeros

x
–1.5 –1 –0.5 0.5 1 1.5

Minimum –1

–2

2 2 1
3x 2 − 3 + 4 x 2 7x 2 − 3
b i f (x ) = x 3
(x 2
− 1) ⇒ f ' ( x ) =
2
3
(x 2
− 1) + x ×
2

3
( x − 1) 3 × 2x =

=
33 x2 − 1 33 x2 − 1
D (f −1) = {x ∈  −1.4  x  1.4}
7x 2 − 3 3 ± 21
ii f ' (x ) = 0 ⇒ = 0 ⇒ 7 x 2 − 3 = 0 ⇒ x1, 2 = ± =
3 x −1
3 2 7 7
3 3
Therefore, the minimum occurs for x = − , whilst the maximum occurs for x = .
7 7
c For ease of calculation of the second derivative, we will rewrite the first derivative as a product.
1 1
1 1
1 4

(7 x 2 − 3) ( x 2 − 1) 3 ⇒ f " ( x ) = 14 x ( x 2 − 1) 3 + (7 x 2 − 3) ×  −  ( x 2 − 1) 3 × 2x 
− − −
f ' (x ) =
3 3  2 
2 2 4
2 4 (7 x 3 − 9 x )
= ( x − 1) 3 21x ( x 2 − 1) − (7 x 2 − 3) × x  =

4 21x 3 − 21x − 7 x 3 + 3x  = 4
9 ( x 2 − 1) 3 9 ( x 2 − 1) 3
9
3 7
f " (x ) = 0 ⇒ 7x 3 − 9x = 0 ⇒ x = ≈ 1.1339, since x > 0.
7
36 Given that the line y = 16 x − 9 is tangent at (1, 7), we can conclude that y ' (1) = 16. The given point on the curve gives us
the second equation.
y = 2 x 3 + a x 2 + b x − 9 ⇒ y ' = 6 x 2 + 2a x + b y ' (1) = 6 + 2a + b = 16 ⇒ 2a + b = 10
x 2 + 2a x + b y ' (1) = 6 + 2a + b = 16 ⇒ 2a + b = 10
y (1) = 2 + a + b − 9 = 7 ⇒ a + b = 14 , so we have a pair of simultaneous equations to solve.
 2a + b = 10  b = 10 − 2a  b = 10 − 2a  b = 18
 ⇒ ⇒ ⇒
 a + b = 14  a + 10 − 2 a = 14  − 4 = a  a = −4
dy
37 a y = tan x − 8 sin x ⇒ = sec 2 x − 8 cos x
dx
1 − 8 cos3 x 1 1
b sec 2 x − 8 cos x = 0 ⇒ = 0 ⇒ 1 − 8 cos3 x = 0 ⇒ cos3 x = ⇒ cos x =
cos2 x 8 2

9
Chapter 15

38 a y = x 3 + 4 x 2 + x − 6 ⇒ y ' = 3x 2 + 8 x + 1 ⇒ y ' (−1) = 3 − 8 + 1 = −4


x = −1 ⇒ y = −1 + 4 − 1 − 6 = −4 ; equation of tangent: y = −4 ( x + 1) − 4 ⇒ y = −4 x − 8
b To find where the tangent meets the curve again, we need to solve a pair of simultaneous equations. We will use the
substitution method.
 y = x 3 + 4 x 2 + x − 6  −4 x − 8 = x 3 + 4 x 2 + x − 6  x 3 + 4 x 2 + 5x + 2 = 0
 ⇒ ⇒ ⇒
 y = −4 x − 8  y = −4 x − 8  y = −4 x − 8

 ( x 3 + 4 x 2 + 4 x ) + ( x + 2) = 0  x ( x + 2)2 + ( x + 2) = 0  ( x + 2) ( x 2 + 2x + 1) = 0
 ⇒ ⇒ ⇒
 y = −4 x − 8  y = −4 x − 8  y = −4 x − 8

 ( x + 2) ( x + 1)2 = 0  x = −2 or x = −1
 ⇒
y = − 4 x − 8 y = −4 × (−2) − 8 = 0 or y = −4
 
So, the second point is (–2, 0).

39 y = sin (k x ) − k x cos (k x ) ⇒ y ' = cos (k x ) × k −k cos (k x ) + k x sin (k x ) × k = k 2 x sin (k x )


d
40 xy 3 + 2x 2 y = 3 ⇒ y 3 + x × 3 y 2 y '+ 4 xy + 2x 2 × y ' = 0 ⇒ y ' (3xy 2 + 2x 2 ) = − ( y 3 + 4 xy )
dx
y 3 + 4 xy 1+ 4
⇒ y'= − , y ' (1, 1) = − = −1 ⇒ Equation of tangent: y = −1( x − 1) + 1 ⇒ y = − x + 2
3xy + 2x
2 2
3+2

x2 − x + 1 (2x − 1) ( x 2 + x + 1) − ( x 2 − x + 1) (2x + 1)
41 a i f (x ) = ⇒ f ' (x ) =
x + x +1
2
(x 2 + x + 1)2
2 x 3 + x 2 + x − 1 −2 x 3 + x 2 − x − 1 2x 2 − 2 2 ( x 2 − 1)
= = =
(x+ x + 1)
2 2
(x 2 + x + 1) (x 2 + x + 1)
2 2

ii If the tangents are parallel to the x-axis, then the gradient is zero.
1+ 1+ 1 1− 1+ 1 1
f ' ( x ) = 0 ⇒ x 2 − 1 = 0 ⇒ x1, 2 = ±1, x = −1 ⇒ y = = 3, x = 1 ⇒ y = =
1− 1+ 1 1+ 1+ 1 3
1
So, the points are: A (−1, 3) and B  1,  .

 3
b 2 y

x
–4 –2 2 4

–1

–2

–3

–4

On the graph of the first derivative, the stationary points are points where the second derivative is zero, that is, the
points of inflexion. A calculator gives us the following:
x = −1.53 or x = −0.347 or x = 1.88
c i We notice that the denominator is never equal to zero, and therefore the domain of the function is the whole set
of real numbers. Also, the function has a horizontal asymptote.
1 1
x2 − x + 1 ÷ x2 1− + 2
lim f ( x ) = lim 2 = lim x x =1
x →±∞ x →±∞ x + x + 1 ÷ x 2 x →±∞ 1 1
1+ + 2
x x

10
Since the minimum point is below the asymptote and the maximum point is above the asymptote of this continuous
1
function, we can say that the range is  , 3 .
3 
2
 x 2 − x + 1 x2 − x + 1
  − 2 +1
 x − x + 1  x + x + 1
2 2
x + x +1
ii (f  f )( x ) = f  2 =
 x + x + 1  x 2 − x + 1 2 x 2 − x + 1
 x 2 + x + 1 + x 2 + x + 1 + 1

(x 2
− x + 1) − ( x 2 − x + 1) ( x 2 + x + 1) + ( x 2 + x + 1)
2 2

(x 2
+ x + 1)
2

=
(x 2
− x + 1) + ( x 2 − x + 1) ( x 2 + x + 1) + ( x 2 + x + 1)
2 2

(x 2
+ x + 1)
2

= ( x 2 + 1) −2x ( x 2 + 1) + x 2 − ( x 2 + 1) + x 2 + ( x 2 + 1) +2x ( x 2 + 1) + x 2 (x + 1) + 3x
2 2 2 2
2 2

=
(x 2
+ 1) −2x ( x 2 + 1) + x 2 + ( x 2 + 1) − x 2 + ( x 2 + 1) +2x ( x 2 + 1) + x 2
2 2 2
3 ( x + 1) + x
2 2 2

We again notice that the domain is the whole set of real numbers and, since the polynomials in the numerator
and denominator are both quartic and the leading coefficients are 1 and 3 respectively, the horizontal asymptote
1
has the equation y = .
3
We can input the composite function into a GDC and find the minimum and maximum point. Notice that,
using the GDC, we did not have to find the formula for the composite function; we merely had to define the
composition in the graphical mode.

1 7
So, the range of the composite function is  ,  .
 3 13 
dV 4 dV dr dr dV 1 dV dr 1 1
42 = 8 cm3 s , V = π r 3 ⇒ = 4 r 2π , = × = × ⇒ (r = 2) = ×8= cm s
dt 3 dr dt dV dt dV dt dt 4 × 22 π 2π
dr

d 3x 2 y 2 3x 2 y
43 x 3 y 2 = 8 ⇒ 3x 2 y 2 + x 3 × 2 yy ' = 0 ⇒ y ' = − 3 = − ⇒
dx 2x y 2x 3
3 × 22 × 1 3 1 4 4 4 5
y ' (2, 1) = − = − , mN = − n of normal: y = ( x − 2) + 1 ⇒ y = x −
= ⇒ Equation
2×2 3
4 y ' (2, 1) 3 3 3 3
44 a We need to rewrite the expression in product form for ease of calculation of the first and second derivative.
x2 2x − x 2 ln 2
i f ( x ) = x = x 2 × 2− x ⇒ f ' ( x ) = 2x × 2− x + x 2 × 2− x ln 2 × (−1) =
2 2x

11
Chapter 15

2 − 4 x ln 2 + x 2 ln2 2
ii f ' ( x ) = (2x − x 2 ln 2) × 2− x ⇒ f " ( x ) = (2 − 2x ln 2) × 2− x + (2x − x 2 ln 2) × 2− x ln 2 × (−1) =
2x
2x − x 2 ln 2 2
b i f ' (x ) = 0 ⇒ = 0 ⇒ x (2 − x ln 2) = 0 ⇒ x =
2x ln 2
We discard the second solution because of the domain of the function, x > 0.
2
2  2  2
2 − 4  ln 2 +  ln 2
 2   ln 2   ln 2  2−8+4 −2
ii f " = = = < 0 ; therefore, we have a maximum value of the
 ln 2   2 
 ln 2 
 2 
 ln 2 
 2 
 
2 2 2 ln 2
function f. We could have tested the nature of the stationary point by using the sign of the first derivative. We
notice that the denominator is always positive and that the numerator is a quadratic function with a negative
2
quadratic coefficient; therefore, at x = , it changes its sign from positive to negative, which yields the
ln 2
maximum value.
2 − 4 x ln 2 + x 2 ln2 2
c f " (x ) = 0 ⇒ = 0 ⇒ 2 − 4 x ln 2 + x 2 ln2 2 = 0 ⇒
2x

x1, 2 =
4 ln 2 ± 16 ln2 2 − 4 × ln2 2 × 2 4 ln 2 ± 8 ln2 2 4 ln 2 ± 2 ln 2 2 2 ln 2 2 ± 2
= = =
( )
2 ln2 2 2 ln2 2 2 ln2 2 2 ln2 2

2± 2 2− 2 2+ 2
= ⇒x= ≈ 0.845 or x = ≈ 4.93
ln 2 ln 2 ln 2
45 a f (t ) = 3 sec 2 t + 5t ⇒ f ' (t ) = 3 × 2 sec t × sec t × tan t + 5 = 6 sec 2 t tan t + 5

b i f (π ) = 3 sec 2 π + 5π = 3 + 5π ii f ' (π ) = 6 sec 3 π sin π + 5 = 5


46 a 2xy 2 = x 2 y + 3; x = 1 ⇒ 2 y 2 = y + 3 ⇒ 2 y 2 − y − 3 = 0 ⇒ (2 y − 3) ( y + 1) = 0 ⇒
3
y= or y = −1 because y < 0.
2
d
b 2xy 2 = x 2 y + 3 ⇒ 2 y 2 + 2x × 2 yy ' = 2xy + x 2 y ' ⇒ y ' ( 4 xy − x 2 ) = 2xy − 2 y 2 ⇒
dx
2 y (x − y ) 2 × (−1) (1 + 1) −4 4
y'= ⇒ y ' (1, −1) = = =
x (4 y − x ) 1 × ( 4 × (−1) − 1) −5 5
dy
47 a y = e 3 x sin (π x ) ⇒ = e 3 x × 3 × sin (π x ) + e 3 x cos (π x ) × π = e 3 x (3 sin (π x ) + π cos (π x ))
dx
dy 
>0
π
b = 0 ⇒ e 3 x (3 sin (π x ) + π cos (π x )) = 0 ⇒ 3 sin (π x ) + π cos (π x ) = 0 ⇒ tan (π x ) = −
dx 3
 π 1  π
π x = arctan  −  + k π , k ∈  ÷ π ⇒ x = arctan  −  + k , k ∈ 
 3 π  3
1 π
Given the condition, for the smallest positive value of x, we take k = 1: x = arctan  −  + 1 ≈ 0.743
π  3
dθ 1 3000 dx
48 = rad s , tan θ = ⇒ x = 3000 cot θ , = −3000 csc 2 θ
dt 60 x dθ
 
−1000
d x d x dθ dx  π 2  π 1 4 1 1000 200
= × ⇒  θ =  = −3000 csc   × = − 3000 × × =− =−
dt d θ dt dt 3 3 60 3 60 15 15 3
200
So, the speed is m s = 240 km hr .
3

12
f ( x ) = a (b + e − c x ) ⇒ f ' ( x ) = a × (−1) (b + e − c x ) × e − c x × (− c ) = ace − c x (b + e − c x ) ⇒
−1 −2 −2
49 a

f " ( x ) = ac e − c x × (− c ) (b + e − c x ) + e − c x × (−2) (b + e − c x ) × e − cx × (− c )


−2 −3

− ac 2 e − c x − ac 2 e − c x (b − e − c x ) ac 2 e − c x (e − c x − b)
= (b + e − cx
− 2e − c x ) = =
(b + e ) − cx 3
(b + e ) − cx 3
(b + e ) − cx 3

≠0

ac 2 e − c x (e − c x − b) ln b
b f " (x ) = 0 ⇒ = 0 ⇒ e − c x − b = 0 ⇒ e − cx = b ⇒ − c x = ln b ⇒ x = −
(b + e ) − cx 3
c
ln b a a  ln b a  .
For x = − ⇒y= = ; so, the point is  − ,
c − c ×  −
ln b 
 c  2b  c 2b 
b+e
c This is a point of inflexion because the second derivative changes its sign at that point. We notice that to the left of
zero the expression is positive, whilst to the right the expression is negative.

50 a 2x 2 y + 3 y 2 = 16 : x = 1 ⇒ 2 y + 3 y 2 = 16 ⇒ 3 y 2 + 2 y − 16 = 0 ⇒ (3 y + 8) ( y − 2) = 0
8
⇒ y = − or y = 2 ⇒ p = 2, since the condition is p > 0.
3
d
b 2x 2 y + 3 y 2 = 16 ⇒ 4 xy + 2x 2 y '+ 6 yy ' = 0 ⇒ y ' (2x 2 + 6 y ) = −4 xy ⇒
dx
4 2 xy 2xy 2 × 1× 2 4
y'= − =− 2 ⇒ y ' (1, 2) = − 2 =−
2 (x + 3 y )
2
x + 3y 1 +3×2 7
51 f ( x ) = 3x ⇒ f ' ( x ) = 3x ln 3 ⇒ f " ( x ) = 3x ln 3 × ln 3 = 3x ln2 3
2 2
f " ( x ) = 2 ⇒ 3x ln2 3 = 2 ⇒ 3x = 2 ln ⇒ x ln 3 = ln 2 ⇒
ln 3 ln 3
ln 2 − 2 ln (ln 3) ln 2 − 2 ln (ln 3)
x= = ≈ 0.460
ln 3 ln 3
dα 5
⇒ α = arctan   ⇒
h h
52 Let CAB = α ⇒ tan α = =
5  5 dh 25 + h2

dh
dt
= 2 cm s, and when the triangle is equilateral: h = 5 3 ⇒
dh
5 3 =
5
=
1
25 + 75 20
( )
d α d α dh dα d α dh
dt
= ×
dh dt

dt
h=5 3 = ( ×
dh dt 20
) =
1
×2=
1
10
rad
ds

dy 2 d2 y −4
53 y = ln (2x − 1) ⇒ = ⇒ 2 =
d x 2x − 1 dx (2x − 1)2
d 3y − x2
54 x 3 + y 3 − 9 xy = 0 ⇒ 3x 2 + 3 y 2 y '− 9 y − 9 xy ' = 0 ⇒ 3 y ' ( y 2 − 3x ) = 3 (3 y − x 2 ) ⇒ y ' = 2
dx y − 3x
1 42 − 3 × 2 10 5 5 5 13
mN = − =− =− = − ⇒ Equation of normal: y = − ( x − 2) + 4 ⇒ y = − x +
y ' (2, 4) 3 × 4 − 22 8 4 4 4 2
π π π
55 a f ' ( x ) = 2 sin  5x −  ⇒ f " ( x ) = 2 cos  5x −  × 5 = 10 cos  5x − 
 2  2  2
b This part cannot be solved using the topics that we have covered so far. We need to use a method of integration:
π 1 π 2 π
f ( x ) = ∫ 2 sin  5x −  d x = 2 ×  −  cos  5x −  + c = − cos  5x −  + c
 2  5  2 5  2
 π 2  5π − π  2 7
f   = 1 ⇒ − cos   + c = 1⇒ − + c = 1⇒ c =
 2 5  2 2 5 5
2  π 7
f ( x ) = − cos  5x −  +
5  2 5

13
Chapter 15

d 2 y 2 + 6 xy
56 3x 2 y + 2xy 2 = 2 ⇒ 6 xy + 3x 2 y '+ 2 y 2 + 2x × 2 yy ' = 0 ⇒ y ' (3x 2 + 4 xy ) = − (6 xy + 2 y 2 ) ⇒ y ' = − 2
dx 3x + 4 xy
1 3 × 12 + 4 × 1 × (−2) −5 5
mN = − = = =
y ' (1, −2) 2 × (−2)2 + 6 × 1 × (−2) −4 4

x5 + 2 5x 4 × x − ( x 5 + 2) × 1 4 x 5 − 2
57 f ( x ) = , x ≠ 0 ⇒ f ' (x ) = = = ( 4 x 5 − 2) x −2
x x2 x2
20 x 5 − 8 x 5 + 4 4 (3x + 1)
5

f " ( x ) = 20 x 4 × x −2 + ( 4 x 5 − 2) × (−2) x −3 = =
x 3
x3 1
4 (3x 5 + 1) 1 − +2 55 3
Point of inflexion: f " ( x ) = 0 ⇒ = 0 ⇒ 3x + 1 = 0 ⇒ x = − 5 , y = 3
5
=−
x 3
3 1 3
−5
 1 5 35 3
Thus, the coordinates are:  − 5 , − ≈ (−0.803, −2.08) .
 3 3 
58 a n (t ) = 650e kt
Since the number of bacteria double every 20 minutes, there are 1300 bacteria after 20 minutes.
ln 2
n (20) = 650e k ×20 = 1300 ⇒ e 20 k = 2 ⇒ 20k = ln 2 ⇒ k =
20
ln 2 ln 2 ln 2
t dn − t ln 2 65 ln 2 t
b n (t ) = 650e 20 ⇒ = 650e 20 × = e 20
dt 20 2
dn 65 ln 2
× 90 65 ln 2 × 16 2
(t = 90) = × (ln 2) e 20 = = 520 2 ln 2 ≈ 510 bacteria min
dt 2 2

59 f ( x ) = a x 3 + b x 2 + c x + d ⇒ f ' ( x ) = 3a x 2 + 2b x + c ⇒ f " ( x ) = 6a x + 2b
Now, we are going to use the conditions:
f (0) = 2 ⇒ d = 2, f ' (0) = −3 ⇒ c = −3, f (1) = f ' (1) ⇒ a + b − 3 + 2 = 3a + 2b − 3, f " (−1) = 6 ⇒ −6a + 2b = 6
So, we have to solve the simultaneous equations in a and b.
 2a + b = 2 
 −
 −3a + b = 3  ⇒ a = − 1 , b = 2 + 2 = 12 ; so, the polynomial is: f ( x ) = − 1 x 3 + 12 x 2 − 3x + 2 .
5a = −1 5 5 5 5 5
60 f ( x ) = cos3 ( 4 x + 1) , 0  x  1

a f ' ( x ) = 3 cos2 ( 4 x + 1) × (− sin ( 4 x + 1)) × 4 = −12 cos2 ( 4 x + 1) sin ( 4 x + 1)

b f ' ( x ) = 0 ⇒ −12 cos2 ( 4 x + 1) sin ( 4 x + 1) = 0 ⇒ cos ( 4 x + 1) = 0 or sin ( 4 x + 1) = 0


π π − 2 kπ
cos ( 4 x + 1) = 0 ⇒ 4 x + 1 = + k π ⇒ x = +
2 8 4
π−2 3π − 2
We notice that two values of x satisfy the domain of the function: k = 0 ⇒ x = and k = 1 ⇒ x =
8 8
−1 + k π
sin ( 4 x + 1) = 0 ⇒ 4 x + 1 = k π ⇒ x =
4
π −1
Only one value of x satisfies the domain of the function: k = 1 ⇒ x =
4
d
61 3x + y = x 3 + 3 y ⇒ 3x + y ln 3 × (1 + y ') = 3x 2 + 3 y ' ⇒ y ' (3x + y ln 3 − 3) = 3x 2 − 3x + y ln 3 ⇒
dx
3x 2 − 3x + y ln 3 x 2 − 3x + y −1 ln 3
y ' = (3 ln 3 − 3) = x + y
x+ y
= x + y −1
3 ln 3 − 3 3 ln 3 − 1
1
62 f ( x ) = ln (3x + 1) , x > −
3
3
a f ' (x ) =
3x + 1
1 7 7 7 14
b x = 2 ⇒ y = ln 7; mN = − = − ⇒ Equation of normal: y = − ( x − 2) + ln 7 ⇒ y = − x + + ln 7
f ' (2) 3 3 3 3

14
dy x 1
63 y = x arcsin x , x ∈ ] −1, 1[ ⇒
dx
= arcsin x + = arcsin x + x (1 − x )
2 − 2

1− x2
d y2
1 1  1 3 2 (1 − x 2 ) + x 2 2 − x2
dx 2
= + + x  −
 2  (1 − x )
2 − 2
× ( − 2 x ) = 3 = 3
1− x2 1− x2 (1 − x 2 ) 2 2 (1 − x 2 ) 2
d 1 y
64 e xy − y 2 ln x = e ⇒ e xy ( y + xy ') − 2 yy ' ln x − y 2 × = 0 ⇒ y e xy − = y ' (2 y ln x − x e xy ) ⇒
dx x x
xy e xy − y 2 e − 1 1− e
y'= ⇒ y ' (1, 1) = =
x (2 y ln x − x e xy ) −e e
2x
65 f ( x ) = , x  b, b ∈ 
x +6
2

2 ( x 2 + 6) − 2x × 2x 2 x 2 + 12 − 4 x 2 12 − 2x 2
a f ' (x ) = = = 2
(x 2 + 6)2 (x 2 + 6)2 (x + 6)2
b This function needs to be restricted to the interval where every value occurs only once, that is, from the maximum
point until the horizontal asymptote, which is the x-axis.
12 − 2x 2
f ' (x ) = 0 ⇒ 2 = 0 ⇒ 12 − 2 x 2 = 0 ⇒ x = 6
(x + 6)2
We can justify that for x = 6 the function has a maximum since the sign of the first derivative changes from positive
to negative.
d 2x + y
66 x 2 + xy + y 2 = 3 ⇒ 2x + y + xy '+ 2 yy ' = 0 ⇒ y ' ( x + 2 y ) = − (2x + y ) ⇒ y ' = −
dx x + 2y
2 × (−1) + k 2 − k
a y ' (−1, k ) = − =
−1 + 2k 2k − 1
b If the tangent is parallel to the x-axis, then the slope is equal to zero; therefore:
2−k
y ' (−1, k ) = 0 ⇒ = 0⇒2−k = 0⇒k = 2
2k − 1
d
67 x 3 y 2 = cos (π y ) ⇒ 3x 2 y 2 + x 3 2 yy ' = − sin (π y ) × π × y ' ⇒ y ' (2x 3 y + π sin (π y )) = −3x 2 y 2 ⇒
dx
3x 2 y 2 −3 3
y'= − 3 ⇒ y ' (−1, 1) = − =
2x y + π sin (π y ) 2 + π sin (π ) 2

0
68

a AQ 2 = 4 + x 2 ⇒ T = TS + TR = 5 5 × 4 + x 2 + 5 × (2 − x ) = 5 5 4 + x 2 + 10 − 5x minutes
dT 2x 5 5x
b =5 5 −5= −5
dx 2 4+x 2
4 + x2

15
Chapter 15

c i
dT
dx
=0⇒
5 5x
4 + x2
( )
− 5 = 0 ⇒ 5 5 x = 5 4 + x 2 / ÷ 5 ⇒ 5x 2 = 4 + x 2 ⇒ 4 x 2 = 4 ⇒ x = 1

We have only one solution since the distance must be positive.

ii x = 1 ⇒ T = 5 5 4 + 1 + 10 − 5 = 30 minutes

iii In the case of a complicated rational expression, it is simpler to use the product rule.
dT 1
d 2T 1
 1 3
= 5 5 x (4 + x 2 ) 2 − 5 ⇒ 2 = 5 5 (4 + x 2 ) 2 + 5 5 x ×  −  (4 + x 2 ) 2 × 2 x
− − −

dx dx  2

=
5 5
3 (4 + x 2
−x2 = ) 20 5
3
(4 + x ) 2 2
(4 + x )
2 2

d 2T 20 5 20 5
(1) = 3 = = 4 > 0 ; therefore, the time found is a minimum.
dx2 ( 4 + 1)2 5 5
69 f ( x ) = x e 2 x ⇒ f (n) ( x ) = (2n x + n × 2n−1) e 2 x , n ∈ +
1
a f ' ( x ) = (2x + 1) e
2x
= 0 ⇒ 2x + 1 = 0 ⇒ x = −
>0 2
  1
1  1  2×  − 
f " ( x ) = ( 4 x + 4 ) e 2 x ⇒ f "  −  =  4 ×  −  + 4 e  2 = 2e −1 > 0 ; therefore, it is a minimum.
 2   2 

b f " ( x ) = ( 4 x + 4) e
2x
= 0 ⇒ 4 x + 4 = 0 ⇒ x = −1
>0

c Since the sign of the second derivative depends on the expression 4 x + 4 :

i We can see that the function is concave up when: f " ( x ) > 0 ⇒ x > −1.

ii The function is concave down when: f " ( x ) < 0 ⇒ x < −1.

d To be able to clearly sketch y = f ( x ) , we need to find the y-coordinates of points P and Q.


1 1
f  −  = − × e −1 ≈ −0.184 , f (−1) = −1 × e −2 ≈ −0.135
 2 2
2.5 y

Horizontal 2
asymptote:
y=f (x)
y=0 1.5

1
y-intercept:
(0, 0)
0.5

x
–2 2
Q –0.5 P
–1

e Basis step: n = 1 ⇒ f ' ( x ) = (2x + 1) e 2 x

Alternatively, we can differentiate the function and check the result:

f ( x ) = x e 2 x ⇒ f ' ( x ) = e 2 x + x e 2 x × 2 = e 2 x (1 + 2x )

We can conclude that the formula works for n = 1.


Inductive step: We assume that the formula works for n = k ⇒ f (k ) ( x ) = (2k x + k × 2k −1) e 2 x
We need to see whether the formula works for:
n = k + 1 ⇒ f (k +1) ( x ) = (f (k ) ( x )) ' = ((2k x + k × 2k −1) e 2 x ) ' = 2k e 2 x + (2k x + k × 2k −1) e 2 x × 2
= e 2 x (2k + 2k +1 x + k × 2k ) = e 2 x (2k +1 x + (1 + k ) × 2k ) = e 2 x (2k +1 x + (k + 1) × 2(k +1)−1)

16
This is the formula for n = k + 1.

The formula works for n = 1 and, from the assumption that it works for n = k , we determined that it works for
n = k + 1. Therefore, by the principle of mathematical induction, we conclude that it works for all n ∈ + .

70

In order to find the width, we need to find the intersection between the curve and the horizontal line y = −6 , since the
water depth is 10 m.
πx πx 13  πx  8 36 8
16 sec   − 32 = −6 ⇒ sec   = ⇒ cos   = ⇒x= arccos  
 36   36  8  36  13 π  13
72 8
The width is twice as long, so: w = arccos   cm.
π  13
In cases such as this, if we have enough time, it is advisable to check the answer by using a GDC.

71 By looking at the graph of the function, we can conclude that the function is a cubic polynomial; therefore, its first
derivative will be a quadratic polynomial with zeros at the x-coordinates of the stationary points and the minimum at the
x-coordinate of the point of inflexion. The reason for stating the minimum point of the first derivative is the positive cubic
coefficient of the original function. The second derivative will be a straight line with a positive slope and the zero at the
x-coordinate of the point of inflexion.
a) b)
y y
y=f '(x)

y=f (x)

x x

y=f(x)

y=f "(x)

17
Chapter 16
Practice questions

1 a We know that the parameter p is directly related to the amplitude; therefore, we can find that p = 3.
π π
π
b ∫ 2 3 cos x d x = [3 sin x ]02 = 3 sin   − 3 sin 0 = 3
0  2
Note: Even though you might not know how to find the parameter in part a, it is always advisable to proceed with part b
and attempt to write the definite integral.
x 0
2 a y = e 2 ⇒ y (0) = e 2 = 1; therefore, point P has the coordinates (0, 1).
2
ln 2  x2  ln 2
b V = π∫  e  d x = π ∫0 e d x
x
0

e x d x = π e x 0 = π (eln 2 − e 0 ) = π (2 − 1) = π


ln 2
π∫
ln 2
c
0

Solution Paper 1 type

a 1
∫ d x = 2 ⇒ [ln x ]1 = 2 ⇒ ln a − ln 1 = 2 ⇒ ln a = 2 ⇒ a = e 2
a
3
1 x

Solution Paper 2 type

Notice that we changed variable a into variable x, and that Solver accepts all of the features from the calculator’s menu
with only one variable parameter to be represented as a function in that variable. We could have used the graphical
mode, but, since there is no more than one solution, we were satisfied using Solver. At the end, we can check whether
the numerical result is a special value which we could have recognized.

1 1
4 a y = ln x ⇒ y ' = . At the point (e, 1) the slope of the tangent is: m = y '( e ) = . The tangent can be found by using
x e
the formula for the tangent: y = f ' ( x1) ( x − x1) + y1 , where ( x1, y1) is a particular point on the graph of the function.
1 1 1
y = ( x − e ) + 1 ⇒ y = x −1 +1 ⇒ y = x
e e e
Since the linear function has no y-intercept, it means that it passes through the origin. If we input the coordinates
1
of the origin into the equation, we get a true statement: 0 = × 0 ⇒ 0 = 0.
e
b For the first term, we need to apply the product rule.
1
( x ln x − x ) ' = ln x + x × − 1 = ln x + 1 − 1 = ln x
x
c The shaded region can be split into two. The first region is a triangle bounded by the tangent line,
1
1× 1
x-axis and the vertical line x = 1. Since that is a right-angled triangle, the area is calculated as: ATriangle = e = .
2 2e
In order to find the area of the second region, we need to evaluate the following integral:

1
Chapter 16

1e  dx  1 x
2

e
 1 e2  1
  1 12 0
 1 1
∫1  e
 x − ln x  = ×
e 2 − ( x ln x − x ) = e × − e ln e + e  −  × − 1 ln 1 + 1 = e − −1
  1  2  e 2  2 2 e
1 1 1 1
Now, the total area is the sum of those two areas; therefore, A = + e− − 1 = e − 1.
2e 2 2e 2
Solution Paper 1 type

5 a i s (t ) = 800 + 100t − 4t 2 ⇒ s (5) = 800 + 100 × 5 − 4 × 52 = 1200 m


ii v (t ) = s ' (t ) ⇒ v (t ) = 100 − 8t ⇒ v (5) = 100 − 8 × 5 = 60 m s−1
iii v (t ) = 36 ⇒ 100 − 8t = 36 ⇒ 64 = 8t ⇒ t = 8 s
iv s (8) = 800 + 100 × 8 − 4 × 82 = 1344 m
b Firstly, we need to find the time at which the plane stops after touchdown:
100 25
v (t ) = 0 ⇒ 100 − 8t = 0 ⇒ t = = 12.5 s
82
Now, we need to find the distance the plane will travel after touchdown:
2
 25 25  25
s   − s (0) = 800 + 100 50 × − 4 ×   −800 = 1250 − 625 = 625 m
 2 2  2
Using part a iv, the remaining runway length is 2000 − 1344 = 656 m; therefore, there is enough runway to stop the
plane if it makes a touchdown before point P.

Solution Paper 2 type

5 a i and ii

iii and iv b

6 a To draw the function, we input the function 3 y


into the calculator and then use Table to
plot the points (or, alternatively, use the trace
feature). 2

x
−3 −2 −1 1 2 3

−1

−2

−3

2
b

Note: Parts a and b cannot be solved without using a calculator.


x2
c ∫ (π sin x − x ) d x = − π cos x − 2 + c , c ∈ 
1  x2 
1
 − π cos 1 − 1  π cos
1

Area = ∫0 (π sin x − x ) d x = − π cos x −
 2 
0
= 
2

 

− 0 − 0

1
= π (1 − cos 1) − ≈ 0.944 , correct to three significant figures.
2
Otherwise:

7 Method I: From the direction of the x-axis


4
We split the shaded region into two: R1, the rectangle enclosed by the lines x = 0, y = , x = 1 and y = 2;
4 1 3
and R2, the region enclosed by x = 1, y = and y = 1 + . The area of the first region is:
 4 2 3 x
A1 =  2 −  × (1 − 0) = . To find the area of the second region, we need to do two things: firstly, we have to find the
 3  3 4 1
point of intersection of the line y = and the curve y = 1 + . By inspection, we see that the point of intersection
 4
3 x 4
is  3,  . Now, to be able to use the definite integral, we need to translate the graph vertically units down.
 3 3
3
3 1 4 3 1 1  1 
A2 = ∫  1 + −  d x = ∫  −  d x = ln x − x 
1  x 3 1 x 3  3 1
  1
0
2 2 2
= (ln 3 − 1) −  ln 1 −  = ln 3 − ⇒ A = A1 + A2 = + ln 3 − = ln 3
 3 3 3 3
Method II: From the direction of the y-axis
We need to express x in terms of y and calculate the integral with respect to the y-variable.
1 1 1
y = 1+ ⇒ = y − 1 ⇒ x =
x x y −1
1 1 1
d ( y − 1) = [ln y − 1 ] 4 = ln 1 − ln = 0 − (− ln 3) = ln 3
2 2
A = ∫4 d y = ∫4
2

3 y −1 3 y −1 3 3
8 a i and ii To draw the function, we input 1.5 y

the function into the calculator and then


use Table to plot the points (or, 1

(1.1, 0.55)
alternatively, use the trace feature).
0.5

(0, 0) (1.57, 0)
x

−2 −1.5 −1 −0.5 0.5 1 1.5 2 2.5

−0.5

−1

−1.5
(2, -1.67)

−2

3
Chapter 16

π
b x 2 cos x = 0, x > 0 ⇒ cos x = 0 ⇒ x =
2 π
c i
π
See the diagram in part a.
π
ii ∫
0
2
x 2 cos x d x
d ∫ 0
2
x 2 cos x d x = x 2 sin x + 2x cos x − 2 sin x  2
0

 
0  
π π π π π π2
2
        
=   sin   + 2   cos   − 2 sin   − (02 sin (0) + 2 × 0 × cos (0) − 2 sin (0)) = − 2 ≈ 0.467
 2   2  2  2  2 4
 

Or:

9 a f ( x + 2π ) = sin2 ( x + 2π ) cos ( x + 2π ) = sin2 ( x ) cos ( x ) = f ( x ) , so the fundamental period of f is 2π .


b

By looking at the graph, we estimate that the range would be [20.4, 0.4].
c i f ' ( x ) = (2 sin x cos x ) cos x + sin2 x (− sin x ) = 2 sin x cos2 x − sin3 x . We can also continue to express the whole
derivative in terms of sine only:
2 sin x cos2 x − sin3 x = 2 sin x (1 − sin2 x ) − sin3 x = 2 sin x − 3 sin3 x
ii In this problem, we use the first form of the derivative:
f ' ( x ) = 0 ⇒ 2 sin x cos2 x − sin3 x = 0 ⇒ sin x (2 cos2 x − sin2 x ) = 0
Since the value of sine cannot be equal to 0 at A, we can conclude that:
1
2 cos2 x − sin2 x = 0 ⇒ 2 cos2 x − (1 − cos2 x ) = 0 ⇒ 3 cos2 x − 1 = 0 ⇒ cos x =
3
1 1 2 2 3
iii f ( xmax ) = sin2 ( xmax ) cos ( xmax ) =  1 −  × = orr
 3 3 3 3 9
π π
d f ( x ) = 0 ⇒ sin x = 0 or cos x = 0 ⇒ x = 0, x = π or x = , so the x-coordinate of point B is .
2 2
sin3 x
e i ∫ sin ( x ) cos ( x ) d x = ∫ (sin x ) d (sin x ) =
2
2
+ c, c ∈ 
3
π
π
π
sin3  
 sin3
x  2  2  − sin 0 = 1
3
ii ∫ 2 f ( x ) d x =   =
0
 3 0 3 3 3
π
f f " ( x ) = 0 ⇒ 9 cos3 x − 7 cos x = 0 ⇒ cos x (9 cos2 x − 7) = 0 . Since the x-coordinate of C is less than , the
second factor must be equal to 0. 2
7 7  7
9 cos2 x − 7 = 0 ⇒ cos2 x = ⇒ cos x = ⇒ x = arccos   ≈ 0.491
9 3  3 

4
10 a
5 y

x
1 2 3 4 5 6

−1

 −1

b x = π ⇒ π + π cos π = π − π = 0
c

In this problem, if we use Solver, we need to use an estimated value that is further to the right of p, which we found
as the first zero. Our estimate was 4. So, the answer is 3.696 72.
d See the diagram above.
π
Area =∫ π(π + x cos x ) d x
0

Area = ∫ (π + x cos x ) d x = [π x + x sin x + cos x ] = (π + 0 − 1) − (0 + 0 + 1)


π
e 0
2
0

= π − 2 ≈ 7.869 044 01 . Or, by using a GDC, we get:


2

So, the answer, correct to six significant figures, is 7.869 60.


11 a i p = g ( x ) − f ( x ) = (10 x + 2) − (1 + e 2 x ) = 10 x + 1 − e 2 x
ln 5
ii p ' = 10 − 2e 2 x = 0 ⇒ 2e 2 x = 10 ⇒ e 2 x = 5 ⇒ 2x = ln 5 ⇒ x = ≈ 0.805 (3 s.f.)
2
1
b i x = 1 + e 2 y ⇒ e 2 y = x − 1 ⇒ 2 y = ln ( x − 1) ⇒ y = ln ( x − 1) = ln x − 1
2
ii f −1 ( x ) = ln x − 1 ⇒ f −1 (5) = ln 5 − 1 = ln 2

(1 + e ) d x
ln 2
c V = π∫ 2x 2
0

Bonus: Evaluate the integral in part c: ln 2


 e4x 
(1 + e ) d x = π ∫ (1 + 2e + e 4 x ) d x = π x + e 2 x +
ln 2 ln 2
V = π∫ 2x 2 2x
0 0
 4 0
 e 4 ln 2   e0    16 4 1 27
= π  ln 2 + e 2 ln 2 +  −  0 + e 0 +   = π  ln 2 + 4 + − 1 −  = π  ln 2 +  ≈ 23.4
 4   4    4 4   4

5
Chapter 16

12 It is not possible to solve this question with a GDC.

( )
V = π ∫ (a x + 2) − ( x 2 + 2) d x = π ∫ (a2 x 2 + 4 a x + 4 − x 4 − 4 x 2 − 4) dx
a 2 2 a

0 0

 (a 2 − 4 ) x 3
a
x5    a5 − 4 a3 a5   2a 5 2a 3 
((a
− 4) x 2 + 4 a x − x 4 ) d x = π 
a
= π∫ 2
+ 2a x 2 −   = π  + 2a 3 −  = π  +
0  3 5  0  3 5  15 3 
 1 
 u = x + 1 ⇒ x = 2u − 2 
 = (2u − 2) u × 2 du = 4  u 2 − u 2  du
3 1
1 2
13 ∫ x x + 1 dx = 
2  1  ∫ ∫  
 du = 2 d x ⇒ d x = 2 du 
 
5 3
 2 5 2 3 8 1 8 1
= 4  u 2 − u 2  + c =  x + 1 −  x + 1 + c , c ∈ 
2 2

5 3  
5 2  
3 2 
dv ds dv 3 (2s − 1) − 2 (3s + 2) 3s + 2 −7 (3s + 2)
14 a = × = ×v ⇒a = × =
ds dt ds (2s − 1)2 2s − 1 (2s − 1)3
−7 (6 + 2) 56
s=2⇒a= =−
( 4 − 1)3 27
 1 
k
π (e 2k − 1)
15 V = π ∫0 (e ) d x = π ∫0 e d x = π ×  e 2 x   =
k k
x 2 2x
 2  0 2

Solution Paper 1 type

k
k  1  1  1 k2 − 1 3
16 ∫1 1 + x 2  d x =  x − x  1 = k − k −1 +1 = k = 2 ⇒ 2k − 3k − 2 = 0 ⇒ (2k + 1) (k − 2) = 0
2

1
k = − or k = 2 , since k > 1 .
2

Solution Paper 2 type

16 We will use Solver on a GDC. We could have easily used the graphing menu too, in a very similar manner.

Note: The value of x, shown in the final screen, is not relevant and it can be any value.

1
17 We can deduce that a (t ) = − t + 2, v (0) = 0. So, we can now proceed with finding the distance travelled by the train.
20
 1  1
v (t ) = ∫  − 20 t + 2 dt = − 40 t
2
+ 2t + c , c ∈ 

1 2
v (0) = 0 ⇒ c = 0 ⇒ v (t ) = − t + 2t
40
60
 − 1 t 2 + 2t   t 
3
60
d= ∫0

40
 =  −
 120
+ t 2   = −1800 + 3600 = 1800 m
 0

6
18 Firstly, we need to find the zeros of the parabola.

y = a 2 − x 2 ⇒ y = ( a − x ) ( a + x ) ⇒ x1 = − a , x 2 = a

The area of the rectangle is AR = 2ah, where h is the height of the rectangle.

The area under the parabola is calculated by the following integral.


a
 2 x3    a3   a3  4
AP = ∫ (a − ) d =  a −   =  a 3 −  −  − a3 +  = a3
a
2
x 2
x x
−a  3  − a  3  3 3
Since the two areas must be equal, we can find the height of the rectangle:
4 2
2ah = a3 ⇒ h = a2
3 3
2
So, the dimensions of the rectangle are: 2a × a2 .
3
19 a fk ( x ) = x ln x − k x , x > 0 ⇒ fk ' ( x ) = ln x + 1 − k , x > 0

b If the function is increasing, the first derivative is positive; therefore:

ln x + 1 − k > 0 ⇒ ln x > k − 1 ⇒ x > e k −1, x ∈  e k −1, +∞ 

The question asks us to find the interval over which f(x) is increasing; therefore, the value of k is 0 and the interval is:
1 1
x > e −1 = , x ∈  , +∞  .
e e 
c i fk ' ( x ) = ln x + 1 − k = 0 ⇒ ln x = k − 1 ⇒ x = e k −1

ii fk ( x ) = x ln x − k x = 0 ⇒ x (ln x − k ) = 0 ⇒ x = 0 or ln x − k = 0

So, the other x-intercept is: ln x − k = 0 ⇒ ln x = k ⇒ x = e k

d Since the curve is below the x-axis, we need to take the absolute value of the integral.
ek
∫ 0
( x ln x − k x ) d x
To solve the first part of the integral, we need to use integration by parts.
 1 
 u = ln x du = d x 
x  = x ln x −  x × 1  d x = x ln x − 1 x d x = x ln x − x + c
2 2 2 2 2

∫ x ln x d x =  x2  2 ∫  2 x  2 2∫ 2 4
 dv = x d x v = 
 2 
ek
 x2 x2 x2   e 2k e 2k
( )
ek
∫0 ( x ln x − k x ) d x =  2 ln x − − k = 2 ln e k
− 1 − 2k = −
4 2  0 4 4
e 2k
So, the area enclosed by the curve and the x-axis is: .
4
e A (e k , 0) , m = fk ' (e k ) = ln
 e k + 1 − k = 1 ⇒ Equation of tangeent: y = 1 × ( x − e k ) + 0 ⇒ y = x − e k
k

f The y-intercept is −e k , so the area of the triangle enclosed by the tangent and the coordinate axes is:
1 e 2k e 2k
A = e k × (− e k ) = = 2× , which is twice the area enclosed by the curve.
2 2 4
g k = 1 ⇒ x1 = e , k = 2 ⇒ x1 = e 2 , k = 3 ⇒ x1 = e 3 , k = 4 ⇒ x1 = e 4 , ...

To verify the statement, we are going to take two consecutive x-intercepts, for k and k + 1:
x k +1 e k +1
= k = e . The ratio is constant and therefore the zeros form a geometric sequence.
xk e

7
Chapter 16

dy dy dy
= dx ⇒ ∫
1+ y2 ∫
20 = 1+ y2 ⇒ = d x ⇒ arctan y = x + c
dx 1+ y2
y = 0, x = 2 ⇒ arctan 0 = 2 + c ⇒ c = −2 ⇒ arctan y = x − 2 ⇒ y = tan ( x − 2)

dv x2 v2
21 k x = mv ⇒ k x d x = mv dv ⇒ ∫ k x d x = ∫ mv dv ⇒ k = m + c, c ∈ 
dx 2 2
x 2 m (v − v 0 )
2 2
v 02 mv 02 x2 v 2 mv 02
x = 0, v = v 0 ⇒ k × 0 = m + c ⇒ c = − ⇒k =m − ⇒k =
2 2 2 2 2 2 2
22 m (v − v 0 )
2 2
4k 4k
x =2⇒k = ⇒ = v 2 − v 02 ⇒ v = + v 02
2 2 m m
22 a 3 y

y=e^(x2)−1
2

y=e^(−x2)
x
–2 –1 1 2

–1

b Given that the point of intersection has an x-coordinate equal to p, we notice that the rectangle has dimensions 1 × p
and the triangle has a vertical base of length 1 and height p.
1 p
ATRIANGLE < AREGION < ARECTANGLE ⇒ p × 1 < AREGION < p × 1 ⇒ < AREGION < p
2 2
c

p = 0.6937 (correct to four decimal places).

(e ( ))
p

− x2 x2
d AREGION = − e − 1 dx
0

In a case like this, it is advisable to store the coordinates of the point of intersection in the GDC’s memory and then
work with this more accurate value.

So, the area of the region is 0.467 (correct to three significant figures).

Notice that the last two parts of the question cannot be done without using a calculator.
 u=x du = d x 
  = 1 x sin 3x − 1 sin 3x d x
23 a ∫ x cos 3 x d x =
 dv = cos 3x d x v = 1 sin 3x  3 ∫3
 3 
1 1 1 1 1
= x sin 3x − ×  − cos 3x  + c = x sin 3x + cos 3x + c , c ∈ 
3 3  3  3 9

8


1 1
x cos 3x d x =  x sin 3x + cos 3x  
6
b i ∫ π
6
6
3 9  π
6

   
 π  3π  1  3π    π  π 1  π 2π 2π
=  sin   + cos    −  sin   + cos    = − ⇒A=
6  2 9  2 18  2 9  2 9 9
           
 
−1 0 1 0

The value of the integral is negative, but, as we need to calculate the area, we simply take the absolute value of
the integral, since the function is always negative for the given integral.

5π    
 1 1   6
 5π  5π  1  5π    π  3π  1  3π   4 π
ii ∫3π x cos 3x d x =  x sin 3x + cos 3x   = 
6
sin   + cos    −  sin   + cos    =
3 9   3π   9  2   9  2
6
6  18 2     6 2    9
1 0 −1 0


1 1
x cos 3x d x =  x sin 3x + cos 3x  
6
iii ∫ 5π
6
6
3 9   5π
6

   
 7π  7π  1  7π    5π  5π  1  5π   6π 6π
= sin   + cos    −  sin   + cos    = − ⇒A=
       
 18 2  9  2  18 
  2  9  2

9 9
−1 0 1 0

Again, the integral was negative, so, for the area, we take the absolute value.

c The areas enclosed by the given boundaries form an arithmetic sequence with first term u1 = and common
9

difference d = . Therefore, the sum of the first n terms is given by:
9
2π n (n − 1) 2π 2nπ  n − 1 2nπ 1 + n n (n + 1) π
Sn = n × + × = 1 +  = × = , n ∈ +
9 2 9 9  2  9 2 9
π π
24 a v (t ) = 0 ⇒ t sin  t  = 0 ⇒ t = 0 or t = k π ⇒ t = 3k , k ∈ 
3  3
Using the restricted domain, we can calculate the values of t : t = 0 or t = 3 or t = 6.

b i In order to avoid a discussion of the positive or negative values of the parts of the integral, we will simply use the
absolute values.
π
Total distance travelled = ∫ t sin  t  dt
6

0 3 
ii

So, the total distance travelled is 11.5 m (correct to


three significant figures).

Note: If not using a calculator, we should split the integral into two parts, from 0 to 3 and from 3 to 6, where the
last one has a negative value and we take its opposite value. The anti-derivative can be found by using integration
by parts.
1
1 1 1  1  t  
25 a Distance travelled = ∫ v (t ) dt = ∫ dt =  n
arctan  
0 0 2 + t2 2  2   0

 1 1   1  1  1
= arctan    −  arctan0 = 2 arctan  2  ≈ 0.435 m
 2  2    2 
 
0

9
Chapter 16

dv −2t
b a= ⇒ a (t ) =
dt (2 + t 2 )2
dy 2x 2x 2
26 a y = 2x 1 + x 2 ⇒ = 2 1 + x 2 + 2x × = 2 1 + x 2
+
dx 2 1+ x2 1+ x2
 u = 1+ x2  2 23 2 3

b ∫ 2 x 1 + x 2
d x =  = ∫ u du = u + c = (1 + x 2 ) 2 + c , c ∈ 
 du = 2x d x  3 3
k
2 3
 2 3
2 2 3
2 3
5
2x 1 + x 2 d x =  (1 + x 2 ) 2   = (1 + k 2 ) 2 − ⇒ (1 + k 2 ) 2 − = 1 ⇒ (1 + k 2 ) 2 = ⇒
k
c ∫ 0 3  0 3 3 3 3 2
2 2 2
5 3 5 3 5 3
1 + k =   ⇒ k 2 =   − 1 ⇒ k =   − 1 ≈ 0.918
2
 2  2  2
This part can be solved directly by using a GDC.

Again, the value of x is irrelevant for this calculation. To be correct, we need to say that the value of x must be within
the domain of the function. In our case, the domain is the set of all real numbers.

∫ 6t − 6t dt = ∫ (6t − 6t ) dt + ∫ (6t − 6t ) dt
2 1 2
27 v (t ) = 6t 2 − 6t , t  0 ⇒ distance = 2 2 2
0 0 1

= (3t − 2t ) + (2t − 3t ) = (1 − 0) + (16 − 12 − 2 + 3) = 6 m


2 3 1 3 2 2
0 1

The curve is a parabola which opens upwards, with the zeros at 0 and 1; therefore, the function is negative from 0 to 1
and positive from 1 to 2.
28

The total distance travelled is 0.852 m (correct to three significant figures).


dT dT dT
= k (T − 22) ⇒ = k dt ⇒ ∫
T − 22 ∫
29 a = k dt ⇒
dt T − 22

⇒ ln T − 22 = kt + c ⇒ T − 22 = e kt + c , c ∈  ⇒ T = 22 + Ae kt , A ∈  +

b To find the constants A and k, we need to solve the simultaneous equations formed from the given information.
 A = 78  A = 78
 T (0) = 100  22 + Ae 0 = 100  A = 78  
i  ⇒ ⇒  ⇒  8 ⇒   8 ⇒
 T (15) = 70  e = 13  15k = ln  13
15 k
 78e = 48
15 k
 22 + Ae = 70
15k

 
 A = 78

 k = 1 ln  8  ≈ −0.324
  
 15  13

10
8
ln   t
8 8 8
ln  t ln  t ln  t
 13  13  13
18 3  13
ii T = 22 + 78e 15
⇒ 40 = 22 + 78e 15
⇒ =e 15
⇒ ln   = ⇒
78  13 15
8 3
ln   t 15 ln  
3  13  13
⇒ ln   = ⇒t = ≈ 45.3
 13 8
15 ln  
 13
dy dy dy dx
30 x − y2 = 1 ⇒ x = y2 + 1 ⇒ 2 = ⇒ arctan ( y ) = ln x + c , c ∈ 
dx dx y +1 x

y = 0, x = 2 ⇒ arctan (0) = ln 2 + c ⇒ c = − ln 2
 
0

x
arctan ( y ) = ln x − ln 2 ⇒ arctan ( y ) = ln
2
 x
y = tan  ln 
 2
x3 u= x+2  (u − 2)3 u3 − 6u2 + 12u − 8
31 ∫ (x + 2)2 d x = 
 du = d x 
 ∫
=
u 2
du = ∫
u2
du

12 8  u2 8 ( x + 2)2 8
= ∫  u − 6 + − 2  du = − 6u + 12 ln u + + c = − 6 ( x + 2) + 12 ln x + 2 + + c, c ∈ 
 u u  2 u 2 x+2

32 a 5 y

4 y=f(x)
x=−3
3

1
x
–4 –2 2 4 6
–1 y=g(x)
–2

–3

–4

–5

b i The logarithmic function g ( x ) = ln ( x + 3) − 2 has a vertical asymptote: x = −3.

ii y-intercept: x = 0 ⇒ g ( x ) = ln (3) − 2 ≈ −0.901

x-intercept: y = 0 ⇒ 0 = ln ( x + 3) − 2 ⇒ ln ( x + 3) = 2 ⇒ x + 3 = e 2 ⇒ x = e 2 − 3 ≈ 4.39
c

d i Refer to the diagram in part a.

∫ ((4 − (1 − x ) ) − (ln (x + 3) − 2)) d x


3.05 2
ii
0

11
Chapter 16

iii

So, the shaded region has an area of 10.6 (correct to three significant figures).

Note: We were able to store the x-coordinate of the intersection in the GDC’s memory since that was the last
calculation done before we found the integral.

e To find the maximum distance, we need to form a new function: h ( x ) = f ( x ) − g ( x ) .

So, the maximum distance between f(x) and g(x) is 4.63 (correct to three significant figures).
dx d y y x dy y dy dx dy dx
33 a x = e θ ⇒ d x = e θd θ ⇒ d θ = , =
x d θ e2θ + 1

dx
= 2
x +1

y
=
x ( x 2 + 1)
⇒ ∫ y
= ∫ x (x2
+ 1)
dx 1 x  1
∫ =  −  d x = ln y − ln ( x + 1) + c , c ∈ 
x ( x 2 + 1) ∫  x x 2 + 1
b 2

2
dy dx 1 1
c ∫ = ∫ x (x ⇒ ln y = ln x − ln ( x 2 + 1) + c ⇒ ln y = ln e θ − ln (e 2 θ + 1) + c , c ∈ 
y 2
+ 1) 2 2
1 1
θ = 0, y = 2 ⇒ ln 2 = ln
1 − ln 2 + c ⇒ c = ln 2
2 0 2
1 2e θ
ln y = ln e θ − ln (e 2 θ + 1) + ln 2 ⇒ y =
2 e2θ + 1

12
Chapter 17
Practice questions

1 Let X be the amount of savings ($), where X  N ( µ = 3000, σ 2 = 5002 ) .


3200 − 3000 
a P ( X > 3200) = P  Z >  = 1 − P ( Z < 0.4) = 1 − 0.66554 = 0.3446 = 34.5%
 500
2300 − 3000 3300 − 3000 
b P (2300 < X < 3300) = P  <Z<  = P (−1.4 < Z < 0.6)
 500 500

= P ( Z < 0.6) − (1 − P ( Z < 1.4)) = 0.7257 − (1 − 0.9192) = 0.6449


So, for two townspeople, we need to square the result: 0.64492 ≈ 0.416.
d − 3000  d − 3000
c P ( X < d ) = 0.7422 ⇒ P  Z <  = 0.7422 ⇒ = 0.6495 ⇒ d = 3325
 500  500
 5 1
2 a Let X be the number of black discs, where X  B  n = 8, p = =  .
 35 + 5 8 
a i–ii

b Now, we change the number of trials to 400. Hence, the expected number of black discs that would be drawn is
1
µ = 400 × = 50 .
8
3 a The area of the shaded region is 0.1.
b Since the areas are the same, P ( X  12) = 0.1 and P ( X  8) = 0.1. We can find the mean value as the average of the
12 + 8
two numbers: µ = = 10 .
2
c To find the standard deviation we need to use the tables.
2 2 2
P ( X  8) = 0.1 ⇒ P  Z  −  = 0.1 ⇒ P  Z   = 0.9 ⇒ = 1.2816 ⇒ σ = 1.56
 σ   σ  σ
11 − 10 
d P ( X  11) = P  Z   = P ( Z  0.641) = 0.739 (correct to 3 s.f.)
 1.56 
 1
4 Let X be the number of heads obtained, where X  B  n = 8, p = .
 2
 8 1
4
1
4
8 ×7× 6 ×5 1 35
a P ( X = 4) =   ×   ×   = × =
 4   2  2 1× 2 × 3 × 4 2 8 128 7

 8 1
3
1
5
8 ×7× 6 1 7
b P ( X = 3) =   ×   ×   = × =
 3   2  2 1× 2 × 3 2 8 32 5

c Since this binomial expression is symmetrical with respect to the middle observation of obtaining 4 heads, the
probability of obtaining 3 heads and the probability of obtaining 5 heads are equal.
7 35 56 + 35 91
P (3  X  5) = 2 × + = =
32 128 128 128

1
Chapter 17

5 Let X be the lifespan of an insect, where X  N ( µ = 57, σ 2 = 4.4 2 ) .


a We need to calculate the values of a and b by using the transformation to standard normal variable.
55 − 57 60 − 57
a= ≈ −0.455, b = ≈ 0.682
4.4 4.4
b i–ii

c i If 90% of the insects die after t hours, that means that 10% of the insects have a lifespan of up to t hours.

1 y 90%

x=−1.281558

x
−3 −2 −1 1 2 3

ii

50 − µ  50 − µ 
6 a P ( X > 50) = 0.3 ⇒ P  Z > 
 = 0.3 ⇒ P  Z <  = 0.7
 10  10 

b H1: The mean speed has been affected by the campaign.


c A one-tailed test is appropriate since the police are interested in decreasing the speed.
d To solve this part of the problem, we are going to use the Z-Test.

2
Since p = 0.0377 < 0.05, we reject H0.
7 Let X be a person’s IQ, where X  N ( µ = 100, σ 2 = 152 ) .
a–b

c H0: The average IQ of the group suffering from the disorder is 100.
H1: The average IQ of the group suffering from the disorder is less than 100.

Since p = 0.0548 > 0.05, we do not have enough evidence to reject the null hypothesis H0.
8 Let X be the weight of a bag, where X  N ( µ = 25.7, σ 2 = 0.52 ) .
a

c Since the probability of a bag weighing less than 25 kg is the same as the probability of a bag weighing more than
26 kg, the mean value must be µ = 25.5 kg.

3
Chapter 17

d With a mean of 26 kg, we lose 0.5 kg per bag; therefore, we lose 0.5 × 0.8 = 0.4 dollars per bag. For an initial cost of
5000
$5000, we need to sell = 12 500 bags to cover the investment.
0.4
9 Let X be the mass of the packets, where X  N ( µ = 750, σ 2 = 252 ) .
a–b c

10 Let X be the height of adults in Tallopia, where X  N ( µ = 187.5, σ 2 = 9.52 ) .


a b

11 Let X be the mass of a lion, where X  N ( µ = 310, σ 2 = 302 ) .


350 − 310  4
a P ( X  350) = P  Z  
 = 1 − P  Z <  = 1 − 0.9082 = 0.0918
 30 3
b Since a and b are symmetrical with respect to the mean, we can write:
a − 310 b − 310  b − 310 
P (a  X  b) = P  Z 
 = 2 × P  Z <  − 1 = 0.95 ⇒
 30 30  30 

b − 310  b − 310
⇒ P  Z <  = 0.975 ⇒ = 1.96 ⇒ b = 368.8 ⇒ a = 251.2
 30  30
12 Let X be the reaction time measured in seconds, where X  N ( µ = 0.76, σ 2 = 0.062 ) .
0.7 − 0.76 0.79 − 0.76
a a= = −1 and b = = 0.5
0.06 0.06
b i P ( X > 0.7) = P ( Z > −1) = P ( Z < 1) = 0.8413
ii P (0.7 < X < 0.79) = P (−1 < Z < 0.5) = P ( Z < 0.5) − (1 − P ( Z < −1)) = 0.6915 − 0.1587 = 0.5328
c i

c − 0.76  0.76 − c  0.76 − c


ii P ( X < c ) = P  Z < 
 = 0.03 ⇒ P  Z <  = 0.97 ⇒ = 1.8808 ⇒
 0.06  0.06  0.06
⇒ c = 0.76 − 1.8808 × 0.06 = 0.647 152

13 Let X be the number of faulty calculators, where X  B (n = 100, p = 0.02).


a E ( X ) = np ⇒ E ( X ) = 100 × 0.02 = 2

4
 100 
b P ( X = 3) =  0.023 × 0.9897 = 0.182 276
 3 
c Using the complementary event: P ( X  2) = 1 − P ( X  1)

14 We need to set up a system of two equations with two unknowns, but this time we will use a slightly different approach.
90 − µ   90 − µ 
P  Z >  = 0.15  P  Z <  = 0.885 
P ( X > 90) = 0.15   σ    σ 
⇒ ⇒ ⇒
P ( X < 40) = 0.12  40 − µ 40 − µ
P  Z <  0.12 
= P  Z <  = 0.12 
 σ    σ 
90 − µ 
= invNorm (0.85) 
σ  90 − σ invNorm (0.85) = µ 
⇒ 
40 − µ 40 − σ invNorm (0.012) = µ 
= invNorm (0.12) 
σ 

The x- and y-values are σ and µ respectively.

3 6 1 8 4
15 a E (X ) = ∑xp i i ⇒ µ=0×
10
+ 1×
10
+ 2× = =
10 5 10 5
i

b i ii
y 0 1 2
1 8 2
3
red 2
_
3\5 P(Y = y) ___ ___ __
5 5 15 15 5
2
4 22 
5
66 33  red green 4\15
4
__
15
4
5 4\15
22  1  green red 4
__
  15
66  33 

1
5 green 1\15
1
__
15
2 1 4 2 9 3
c P (2R) = × + × = =
6 10
   6 5
 30 10
1 or 6 Bag A 2 , 3 , 4 , 5 Bag B

21 1
×
6 3 10 1
d P (A 2R) = =
3 9
10

5
Chapter 17

16 Let X be the number of defective ball bearings, where X  B (n = 50, p = 0.04) .


a–b c E ( X ) = np ⇒ E ( X ) = 50 × 0.04 = 2

17 Let X be the number of functioning CDs in a pack, where X  B (n = 10, p = 0.98) .


a Using the complementary event, the probability that a package is returned is 0.1829 (see GDC screen below).
b Introducing a new binomial variable: Y  B (n = 3, p = 0.182 93)

18 a ∑ P (x ) = 1 ⇒ 2k + 2k 2
+ k 2 + k + 2k 2 + k = 1 ⇒ 5k 2 + 4k = 1 ⇒ 5k 2 + 4k − 1 = 0 ⇒
Imposssible
1 
⇒ (5k − 1) (k + 1) = 0 ⇒ 5k − 1 = 0 or k + 1 = 0 ⇒ k = or k = −1
5
Note: As a probability, k must be a positive number between 0 and 1.
2 2 6 7 35 7
b E (X ) = ∑xp i i ⇒ µ=0×
5
+ 1×
25
+2×
25
+3× = = = 1.44
25 25 5
i

19 a Let X be the number of small tomatoes, where X  B (n = 12, p = 0.023) .

b Let Y be the size of a tomato, where Y  N ( µ = 3, σ 2 = 0.52 ) .

20 X  N ( µ = 1.02, σ ) ⇒ P ( X < 1) = 0.017

Since the units in the calculation are kg, the standard


deviation is 9.4 g.

6
 e − ke k x 0  x  1
21 f ( x ) = 
0 otherwise


∫ (e − ke ) d x = 1 ⇒ (e x − e )
1 1
kx kx
a 0
= 1 ⇒ e − ek + 1 = 1 ⇒ e = ek ⇒ k = 1
0
1 1 1 1
1 1 1
b ∫ (e − e ) d x = (e x − e )
x x
2
1
2
1 = e − e2 − e + e4 = e − e + 4 e
4 4 2 4 4
c To find the two values, we need to refresh our memory regarding integration by parts.
 u=x du = d x 
∫ xe xex − ∫ ex d x = xex − ex + c, c ∈ 
x
dx =  x x =
 dv = e d x v = e 
 u = x2 du = 2x d x 
∫   = x 2 e x − 2 ∫ x e x d x = x 2 e x − 2 x e x + 2e x + c , c ∈ 
2 x
x e d x =
 dv = e d x
x
v = ex 
 
1
 x 2
 1
∫ (x e − x e ) d x =  e 2
1
µ= x
− e x ( x − 1)  = e − 1
0  0 2
1 2
 x3   1 
∫0 (x e − x e ) d x − µ =  e 3 − e (x − 2x + 2) 0 −  2 e − 1
1
σ2 = 2 2 x 2 x 2

2
1 2e e 2 e e2
=  − e + 2 −  e − 1 = 2 −
e
− + e − 1 = 1+ −
3  2  3 4 3 4
1 1
1 1
 e
P ( X  0.5) = 1 − P ( X < 0.5) = 1 − ∫ (e − e ) d x = 1 − (e x − e ) = 1 −  e − e 2 + 1 = e−
x x
d 2 2
0 0
2  2
 e
We can define a new variable: Y  B  n = 3, p = e − 
3
 2
 e
i P (Y = 3) =  e − 
 2
ii If two batteries have failed, exactly one has not failed.
2

P (Y = 2) = 3  e −   1 − e + 
e e
 2   2
0 y<0

22 f ( y ) =  −
y
 0.5e 2 y  0

a Since the time is measured in years, we need to calculate the following probability:
y y 0.5
0.5 −  − 
P (Y < 0.5) = ∫ 0.5e 2 d y = 0.5  −2e 2   = − e −0.25 + 1 ≈ 0.2212
0   0
b We can use a binomial variable: X  B (n = 3, p = 0.2212) . ‘At least two’ means either two or all three components fail.

P ( X  2) = 1 − P ( X  1) ≈ 0.125

23 a I  N ( µ = 60.33, σ 2 = 1.952 )

P (I > x ) = 0.8

So, the distance is 58.69 m.

7
Chapter 17

b K  N ( µ = 59.39, σ 2 )

P (K > 56.52) = 0.8

So, the standard deviation is 3.41 m.

c I  N ( µ = 60.33, σ 2 = 1.952 ) , K  N ( µ = 59.50, σ 2 = 32 )

i We are going to use the complementary event, which is that neither of them has a throw longer than 65 m in
those three throws.

1 − (P (I < 65)) = 0.0247 1 − (P (K < 65)) = 0.0968


3 3

So, Karl is more likely to qualify.

ii

24 a 2 y

1
y=1/6x(1+x2)

x
–1 1 2 3

b From the graph, we can see that the modal value is 2 since the probability density function has a maximum value of
5
at 2.
3 2
2 1 1  x3 x5   1 8 32 68
c ∫ x 2 (1 + x 2 ) d x =  +   =  +  =
0 6 6 3 5  0 6  3 5  45
m
1 1 1  x2 x4   1 2m2 + m 4
x (1 + x 2 ) d x = ⇒  +   = ⇒
m
d ∫ 0 6 2 6 2 4  0 2 4
= 3 ⇒ m 4 + 2m2 − 12 = 0

So, the median value is 1.614.

8
25 A  N ( µ = 1.56, σ 2 = 0.162 ) , B  N ( µ = 1.52, σ 2 = 0.162 )

a P (B < 1.52) = 0.242

So, the mean diameter of a bolt produced by manufacturer B is 1.63 mm.

b P (S ) = 0
.44 × P ( A < 1.52) + 0
.56 × P (B < 1.52) = 0.312
A B

0.56 × P (B < 1.52)


c P (B S ) = = 0.434
0.312

d If manufacturer B makes 8000 bolts, the following are the expected numbers of bolts produced.

So, we can say that we would expect 1936 smaller bolts, 5201 bolts of diameter 1.52–1.83 mm and 863 larger bolts.
Therefore, the expected profit is calculated as follows:

5201 × 1.5 + 863 × 0.5 − 1936 × 0.85 = 6587 (correct to the nearest dollar).

You might also like